Download as pdf or txt
Download as pdf or txt
You are on page 1of 320

MCQs

in
oral suruerv
MCQs
in
Oral suruerv

Babu S Parmar
MDS,FAOMSI
Professor and Head
Departmen t of Oral and Maxillofacial Surgery
Government Dental College and Hospital
New Civil Hospital Campus, Asarwa
Ahmedabad - 380016
Member: Den'tal Council of Ind ia, New Delhi
Hon. Dental Surgeon to HE The Governor of Gujarat

Foreword
Neelima A Malik

JAYPEE BROTHERS
MEDICAL PUBLISHERS (P) L TO
New Delhi
Published by
Jitendar P Vij
Jay pee Brothers Medical Publishers (P) Ltd
EMCA House, 23123B Ansari Road. DaryaganJ
New Delhi 110 002, India
Phones: +91-11-23272143, +91-11-23272703, +91-11-23282021 , +91-1 1-23245672
Fax: +91 - 11-23276490, +91-1 1-23245683
e-mail: jaypee@jaypeebrothers.com
Visit our webs ita: www.jaypeebrothers.com

Branches
• 2fB Akruti Society. Jodhpur Gam Road Satellite
Ahmedabad 380 015 Phone: +91-079-30988717
202 Batavia Chambers, 8 Kumara Krupa Road, Kumara Park East,
Bangalore 560 001, Phones: +91-80-22285971 , +91-80-22382956, +91-80-30614073
Tele Fax: +91-80-22281761
e-mail: jaypeemedpubbgl@eth.net
• 282 lllrd Floor, Khaleel Shirazi Estate, Fountain Plaza,
Pantheon Road, Chennal 600 008, Phones: +91-44-28262665, +91-44-28269897
Fax: +91-44-28262331
e-mail: jpchen@eth.net
• 4-2-106711-3, 1st Floor, Balaji Building, Ramkote Cross Road,
Hyderabad 500 095, Phones: +91-40-55610020, +91 -40-24758498
Fax: +91-40-24758499
e-mail: jpmedpub@rediffmail.com
• 1A Indian Mirror Street. Wellington Square.
Kolkata 700 013, Phones: +91-33-22456075, +91-33-22451926 Fax: +91-33-22456075
e-ma1l: jpbcal @cal.vsnl.nel.in
• 106 Amit Industrial Estate, 61 Dr SS Rao Road.
Near MGM Hospital, Parel, Mumbal400 012
Phones: +91-22-24124863, +91-22-24104532, +9 1-22-30926896
Fax: +91-22-24160828
e-mail: jpmedpub@bom7.vsnl.net.in
• "KAMALPUSHPA" 38, Reshimbag
Opp. Mohota Science College, Umred Road
Nagpur 440 009 (MS), Phones: +91-712-3945220, +91-712-2704275
e-mail: jpmednagpur@ rediffmail. com

MCQs in Oral Surgery

IC> 2006, Babu S Parmar

All rights reserved. No part of this publication should be reproduced, stored in a retrieval system,
or transmitted in any form or by any means: electronic, mechanical, photocopying, recording, or
otherwise, without the prior written permission of the author and the publisher.

This book has been published in good faith that the material provided by author is original.
Every effort is made to ensure accuracy of material, but the publisher, printer and author will
not be held responsible for any inadvertent error(s). In case of any dispute, all legal matters
are to be settled under Delhi jurisdiction only.

First Edition : 2006

ISBN 81 -8061-733-5

Typeset at JPBMP typesetting unit


Printed at Sana! Printers, Kundli.
To
My late beloved
Mother ...
Prof. Dr. (Mrs.) Neelima A. Malik
M.D.S. (Bom.), F.I.A.O.S.
ORAL & MAXILLOFACIAL SURGEON
HEAD OF THE DEPT. OF.
ORAL & MAXILLOFACIAL SURGERY
NAIR HOSPITAL DENTAL COLLEGE
DR. A. L. NAIR ROAD, MUMBAI-400 008.1NDIA.
Hosp. Tel.: 308 2714

IFOREWORD I
Since the advent of entrance examinations for various gradu-
ate courses in our country, the criterion for admission is solely
based on their performances in the MCQ based examinations.
The Postgraduate entrance examinations for MDS course,
though, began a bit later, however; have gained a momentum
presently all over India for most of the institutions . Prepara-
tion for these examinations is totally different from that pre-
paring for the other undergraduate examinations. Lot of hard
work with logical learning is required for high marks.
In such a competitive arena, there is a long felt need for good
standard MCQ books, which are more specific and compre-
hensive with regards to a particular subject. I personally feel
that such books are not only meant for the entrance
examination, but also useful in enrichment of the knowledge.
Professor Babu S Parmar wishes to publish a book on MCQs
in Oral Surgery. I have seen the manuscript on the lap top and
am pleased to endorse the work of Dr Babu S Parmar, whom I
know from his student days. He has risen in the field of Oral
and Maxillofacial Surgery through his hard and sincere work
as a dynamic individual. He is a dedicated teacher and
academician. His vast experience as a DCI member and an
Inspector for various dental colleges has aided in the upliftment
of the academic standards of the dental students across the
country.
viii MCQs in Oral Surgery

This book is a collection of MCQs which have been strati-


fied into the important topics pertaining to Oral and Maxillo-
facial Surgery. These topics have been thoroughly researched
upon and a very concise, yet easy to comprehend text has been
laid forth.
I being an author of a Textbook of Oral and Maxillofacial
Surgery, know that writing a book is a tedious and time con-
suming job and hence the efforts of Dr Parmar are commend-
able. His book has almost covered the entire syllabus in the
subject, providing organized, reliable information for the post-
graduate entrance examinations. I am sure that the students,
who aspire for success in the entrance examination will get high
yielding results.
I congratulate and compliment Dr Babu S Parmar for his
well thought, student friendly excellent work and wish him all
the best in his future endeavors. I also take this opportunity to
congratulate the publishers, who are also providing a platform
for all deserving authors through their constant encouragement,
and motivation. Once again congratulations to all involved and
best wishes.

Res. :55. 'RUTURAJ' 3rd Floor, Juhu Road, Associate Dean, NHDC
(Next to Lido Cinema) Santa Cruz (W), 1st Lady President AOMSI
Mumbai 400 049. India.
TEL: 648 2932. Fax: 91-22-649 0691
Preface

Life transforming ideas have always come to me through books.


-Bell Hooks

The field of Oral and Maxillofacial Surgery has been ever


changing since past SO years. The newer advances in the field
of reconstruction, repair, anesthesia and pain has made it a
branch of interest not only for the people belonging to the
fraternity but even other specialties.
In this gruesome world of tough competition I felt under
moral obligation to do something related to my field for those
struggling young minds who aspire to specialize and super-
specialize but are defeated by the system of entrance examina-
tions.
This is a comprehensive book comprising of about 1500
Multiple Choice Questions assorted from all the topics of Oral
Surgery. I have tried to include all possible questions which in
one or other form can be asked in the forthcoming entrance
examinations or will be of aid to undergraduate students in
their syllabus.
The book also tends to needs of Postgraduate students in
giving them a bird sight view of the subject and can aid them in
viva-voce. It's a matter of pride that the book has been under-
taken for publishing by Jaypee Brothers Medical
Publishers {P) Ltd., New Delhi, whose name in Publishing
parallels authenticity.
Undertaking of such a magnitude cannot be completed
without the cooperation and support of some individuals. I
sincerely thank to my postgraduate students for their timely
help and support during the writing of the book.
x MCQs in Oral Surgery
I am also indebted for life to my family for their belief in me
and their unconditional support when I could not spare even
minimal time for them.
I extend a token of gratitude to my Department, Dept. of
Oral and Maxillofacial Surgery, Government Dental College and
Hospital, Ahmedabad for their cooperation.
Last but not the least I thank almighty for his blessings.

Babu S Parmar
Contents

1. Surgical Anatomy ................................................................ 1


2.
3.
.
LA/GA/Pain Control .......................................................... 17
Sterilization and Instruments .......................................... 78
4. Exodontia/Impaction ......................................................... 85
5. Minor Surgical Procedure .............................................. 130
6. Cyst/Tumour/Surgical Pathology ................................. 146
7. Fascial Space Infection/Osteomyelitis ......................... 174
8. TMJ and Maxillary Sinus ............................................... 191
9. Maxillofacial Injury ........................................................ 205
10. Reconstructive and Orthognathic Surgery .................. 243
11. Medical Emergency., ....................................................... 253
12. Implant ............................................................................... 297
13. Miscellaneous ................................................................... 304
It I SURGICAL ANATOMY

1. The major blood supply of the dura mater is via


which of the following artery:
A. Internal carotid
B. Midd le cerebral
C. Accessory meningeal
D. Middle meningeal
2. The osteum of the maxillary sinus normally commu-
nicates with the nasal cavity via:
A. The ethmoid infundibulum
B. The infra temporal fossa
C. The nasolacrimal duct
D. Below the level of inferior turbinate
3. When attempting to ligate the lingual artery low in
the submandibular triangle, which of the following
nerve is most susceptible to damage?
A. Facial B. Lingual
C. Accessory D . Hypoglossal
4. The ramus mandibularis branch of the facial nerve
has a constant relation ship with which of the fol-
lowing veins in the region of the mandibular angle?
A. Internal jugular
B. External jugular
C. Deep lingual
D. Retromandibula r

1 D 2 A 3 D 4 D
2 MCQs in Oral Surgery

5. Which of the following artery is most commonly in-


volved in extradural haemorrhage?
A. Meningeal branch of the ascending pharyngeal
B. Anterior cerebral
C. Middlen meningeal
D. Accessory meningeal
6. When attempting veni puncture in the region of the
anti cubital fossa which of the following artery is
most likely to be encountered?
A. Deep brachial
B. Brachial
C. Recurrent radial
D. Radial
7. In a patient presenting with a dilating pupil , one
should suspect Involvement of the:
A. Ophthalmic division of the trigeminal nerve
B. Superior cervical ganglion
C. Ciliary ganglion
D. Maxillary division of the trigeminal nerve

8. Aphasia involves the lesion of which of the follow-


ing areas? ·
A. Cerebral cortex
B. Medulla oblongata
C. Spinal cord
D. Cerebellum
9. In removal of the submaxillary gland which of the
following nerves are most likely to be damaged?
A. Lingual and glossopharyngeal
B. Lingual and hypoglossal
C. Facial and glossophyaryngeal
D. Facial and accessory

5 c 6 B 7 c 6A 9 B
Surgical Anatomy 3

10. One of the earlier signs of cavernous sinus throm-


bosis is due to deficit in the function of:
A. Trochlear nerve
B. Abducent nerve
C. Oculomotor nerve
D. Ophthalmic division of trigeminal nerve
11. Which of the following best illustrates the symp-
toms of Horner's syndrome?
A. Myosis and ptosis of the upper eyelid
B. Mydriasis and ptosis
C. Myosis and exopthalmos
D. Ptosis and hydrosis
12. Ataxia involves the lesion of the:
A. Spinal cord
B. Cerebral cortex
C. Medulla oblongata
D. Cerebellum
13. The mandibular branch of the trigeminal nerve
passes through the:
A. Foramen ovale
B. Foramen rotundum
C. Foramen lacerum
D. Foramen spinosum
14. Roof of pterygomandibular space is formed by:
A. Cranial base
B. Temporal muscle
C. Medial pterygoid
D. Lateral pterygoid

10 B 11 A 12 0 13 A 14 0
4 MCQs in Oral Surgery

15. The source of motor innervation of the larynx:


A. Glossopharyneal
B. Facial
C. Hypoglossal
D. Vagus
16. Which of the following nerves are Involved In the
gag reflex?
A. Hypoglossal
B. Hypoglossal and glosopharyngeal
C. Glosopharyngeal and vagus
D. Accessory and vagus
17. Which are the following cranial nerves does not
make the cranial component of the parasympathetic
system?
A. Occulomotor
B. Accessory
C. Facial
D . Vagus
18. Which of the following innervates the buccal gin-
giva of maxillary second premolar?
A. Buccal nerve
B. Anterior palatine nerve
C. Posterior p alatine nerve
D. Middle superior alveolar nerve
19. Which is the only medial branch of the external ca-
rotid artery?
A. Ascending pharyngeal
B. Superior thyroid
C. Lingual
D . Internal maxillary

15 D 16 c 17 B 18 0 19 A
Surgical Anatomy 5

20. Which is not one of the main sensory branches of


the mandibular division of the trigeminal nerve?
A. Lingual nerve
B. Inferior alveolar nerve
C. Nerve to the mylohyoid
D. Auricula temporal nerve
21. Which duct provides drainage to the parotid
glands?
A. Bartholin's
B. Santorinis
C. Warthon's
D. Stenson's
22. In acute airway obstruction should it be necessary
to perform a coniotomy the entry should be made
at:
A. Cricoid cartilage
B. Thyroid notch
C. Cricothyroid ligament
D. Thyroid membrane
23. In the pterygomandibular space the inferior alveo-
lar nerve passes:
A. Medial to the pterygomandibular ligament
B. Medial to the medial pterygoid muscle
C. Lateral to the sphenomandibular ligament
D. Anterior to the deep tendon of temporalis muscle
24. During removal of the torus palatinus the mid por-
tion of the palatine process of the maxilla is
inadvertantely removed. One would expect to see:
A. An opening in to the nasal cavity
B. A vertical fracture of the maxilla
C. An opening i1~ to maxillary antrum
D. A horizontal fracture of maxilla

20 c 21 D 22 c 23 c 24 A
6 MCQs in Oral Surgery

25. Blood vessels· and nerves are generally scarce in


which component of TMJ?
A. Anterior portion of disc
B. Posterior portion of disc
C. Central portion of disc
D. Articular capsules
26. The facial nerve exits from skull from which fora-
men?
A. Ovale
B. Rotundum
C. Stylomastoid
D. Spinosum
27. Motor innervation of the muscle of facial expres-
sion is via :
A. Abducent nerve
B. Facial nerve
C. 1st division of the trigeminal nerve
D. 2nd division of the trigeminal nerve
28. The middle meningeal artery is associated with
which foramen?
A. Ovale
B. Rotundum
C. Stylomastoid
D. Spin.osum
29. A lesion of the lingual branches of the glossopha-
ryngeal nerve would result in the loss of taste from
which portion of the tongue?
A. Anterior third
B. Anterior two-third
C. Posterior third
D. Posterior two-third

25 c 26 c 27 B 28 D 29 c
Surgical Anatomy 7

30. Langer's line in the face usually are:


A. Parallel to the natural creases of the face
B. At right angle to the natural creases of the face
C. Due to subcutaneous interstitial fluid
D. Due to subcutaneous fat of the face
31 . Which of the following is not a structural part of the
temporomandibular joint?
A. Sigmoid notch
B. Condylar process
C. Articular disc
D. Capsular ligament
32. Which is not an anterior triangle of neck?
A. Diagastric
B. Muscular
C. Parotid
D. Subclavian
33. Which autonomic ganglion is not associated with
the function of the major salivary gland?
A. Otic
B. Ciliary'
C. Submaxillary
D. None of the above

34. The hypoglossal nerve provides:


A. Motor innervation to all of the muscles of the tongue
both intrinsic and extrinsic
B. Sensory innervation to the anterior two-third of the
tongue
C. Motor innervation to the s tyloglossus and
hypoglossus muscles only
D . Sensory innervation to the posterior third of the
tongue

30 A 31 A 32 0 33 8 34 A
8 MCQs in Oral Surgery

35. In phonation,the true vocal cords deal primarily with:


A. Expira tion
B. Inspiration
C. Both of the above
D. None of the above
36. In the inferior alveolar nerve block , the needle is
inserted adjacent and lateral to a raphe formed by
the buccinator and the :
A. Temporalis
B. Internal pterygoid
C. Superior constrictor of the pharynx
D. Middle constrictor of the pharynx
37. Classic Bell's palsy results from a lesion involving
which of the following nerves?
A. Trigeminal
B. Hypoglossal
C. Glossopharyngeal
D. Facial
38. The lingual artery is a branch of which of the fol-
lowing artery?
A. External carotid
B. Internal carotid
C. Internal maxillary
D. External maxillary
39. Which one of the following vein does not drain into
the internal jugular?
A. Vein
B. Posterior facial(retromandibular)
C. Anterior facial
D. Posterior auricular

35 B 36 c 37 D 38 A 39 A
Surgical Anatomy 9

40. The mesenteric artery arises from which of the fol-


lowing artery:
A. Midd le meningeal
B. Internal maxillary
C. External maxillary
D. Inferior alveolar
41 . The triangle of the lingual artery is:
A. Lesser's triangle
B. Carotid triangle
C. Submental triangle
D. Muscular triangle
42. Severe trismus resulting from acute dental pathol-
ogy indicates swelling in which one of the follow-
ing spaces:
A. Sublingual
B. Carotid
C. Retropharyngeal
D. Masticator
43. Which of the following muscle is an intrinsic muscle
of the tongue?
A. Chondroglossus
B. Glossopalatinus
C. Transverses lingual
D. Hypoglossus
44. Which of the following is not a branch of the inter-
nal maxillary artery?
A. Posterior auricular
B. Infraorbital
C. Posterior superior alveolar
D. Inferior alveolar

40 B 41 A 42 D 43 c 44A
10 MCQs in Oral Surgery

45. Which of the following is innervated by vagus


nerve?
A. Mylohyoid muscle
B. Tensor veli palatine
C. Levator veli palatine
D. Posterior belly of diagastric
46. Which muscle does not function in opening of the
mandible?
A. External pterygoid B. Diagastric
C. Hypoglossus D. Mylohyoid
47. The carotid triangle is separated from the submax-
illary triangle by:
A. The superior belly of omohyoid muscle
B. The hyoid bone
C. Anterior belly of diagastric
D. Posterior belly of diagastric
48. The hypoglossal nerve is:
A. Purely sensory nerve
B. Mixed nerve
C. Autonomic nerve
D . Purely motor nerve
49. Which are components of the posterior triangle of
neck?
A. Occipital triangle
B. Carotid triangle
c. Subclavian triangle
D. Muscular triangle
1. 2 and 3
2. 1 and 4
3. 1 and 3
4. 1 and 2

45 c 46 c 47 A 48 0 49 c
Surgical Anatomy 11

50. The trigeminal nerve is:


A. Mixed nerve
B. Purely motor nerve
C. Purely sensory nerve
D. Mixed and autonomic nerve
51. Which one of the following bones does not form
the part of the orbit?
A. Palatine B. Maxillary
C. Frontal D . Nasal
52. The no of branches of the internal carotid artery in
the neck:
A. None B. 1
c. 2 D. 3
53. Which one of the following nerves has a constant
relation to the occipital artery as It arises from the
.. external carotid artery?
A. Vagus B. Glossopharyngeal
C. Accessory D. Hypoglossal
54. The semilunar/gasserian ganglion is found in a
space known as:
A. Scarpa's space
B. Meckel's cavity
C. Antrum of Highmore
D. Rathke's pouch
55. The course of the lingual nerve in relation to the
submaxillary duct as it passes forward is sequen-
tially:
A. Below, medial, superior, lateral
B. Above, medial, inferior, lateral
C. Above, lateral, inferior, medial
D. Below, lateral, superior, lateral

:10 A :11 D 52 A 53 D 54 B 55 c
12 MCQs in Oral Surgery

56. The intrinsic muscles of the larynx deals with the


true and false vocal cords. Which cranial nerve
innervates these muscles?
A. Vagus
B. Spinal accessory
C. Hypoglossal
D. Glossopharyngeal
57. Which structure passes between two roots of the
auriculotemporal nerve?
A. Inferior alveolar artery
B. Internal maxillary artery
C. Middle meningeal artery
D. Sphenomandibular ligament
58. The ansa hypoglossal is composed of the:
A. Ascending cervical and ascending hypoglossal nerve
B. Ascending cervical and descending hypoglossal
nerve
C. Descending cervical and ascending hypoglossal nerve
D. Descending cervical and descending hypoglossal
nerve
59. Which syndrome consists of flushing, warmness,
perspiration over the cheek and pinna of the ear
on one side following ingestion of highly seasoned
foods?
A. Cushing's syndrome
B. Horner's syndrome
C. Auriculotemporal syndrome
D. Fanconi's syndrome

56 A 57 c 58 D 59 c
Surgical Anatomy 13

60. The common carotid artery usually divides into


external and internal carotid arteries at the level
of:
A. Hyoid bone
B. Above the level of hyoid bone
C. Superior border of thyroid cartilage
D. Inferior border of thyroid cartilage
61. The maxillary branch of trigeminal nerve passes
through the:
A. Foramen ovale
B. Foramen rotundum
C. Superior orbital fissure
D. Foramen lacerum
62. Which is not a branch of facial nerve?
A. Zygomatic B. Temporal
C. Buccal D. Greater auricular
63. Which artery is not a part of circle of Willis?
A. Internal carotid
B. Posterior communicating
C. Anterior cerebral
D. Vertebral
64. The schneiderian membrane lines the:
A. Maxillary sinus B. Oral cavity
C. Heart D. Inner ear
65. The motor component of the trigeminal nerve sup-
plies all of the following except:
A. Muscles of mastication
B. Anterior belly of digastric
C. Tensor tympani
D. Platysma muscle

60 c 61 B 62 D 63 D 64 A 65 D
14 MCQs in Oral Surgery

66. The maxillary sinus is also known as:


A. Meckels cave
B. Aantrum of Highmore
C. Burns space
D. Rathke's pouch
67. Which nerve supples sensory innervation for taste
to the anterior two-third of the tongue?
A. Hypoglossal
B. Lingual
C. Chorda tympani
D. Vagus
68. Which nerve is not a branch of the trigeminal
nerve?
A. Digastric
B. Mandibular
C. Maxillary
D. Lingual
69. Waldeyer's ring contains what type of tissue:
A. Muscle
B. Nerve
C. Lymphoid
D . None of the above
70. Motor innervation to the sternocleidomasto i d
muscle is supplied by:
A. Hypoglossal
B. Spinal accessory
C. Glossopharyn geal
D. Vagus

66 B 67 c 68 A 69 c 70 B
Surgical Anatomy 15

71. The facial nerve supplies all of the following


except:
A. Anterior belly of digastric
B. Posterior belly of digastric
C. Platysma
D. Scalp
72. The left common carotid artery usually arises from:
A. Left auricle of the heart
B. Costocervical trunk
C. Arc of aorta
D. Brachiocephalic artery
73. Greater palatine foramen is situated:
A. Between first a nd second maxillary molars
B. Between second and third maxillary molars
C. Between two central incisors
D. Between first and second maxillary premolars
74. Of the following which artery is most commonly
involved in stroke?
A. Lenticulostriate artery
B. Cerebral artery
C. Ophthalmic artery
D. None of the above
75. Any laceration of the scalp causes less bleeding:
A. Because of less blood supply
B. Because the vessels are bound loosely in the
connective tissue and retract readily
C. All of the above
D. Above statement is wrong

71 A 72 c 73 B 74 A 75 D
16 MCQs in Oral Surgery

76. Mental foramen opening is directed in:


A. Forward and medial direction
B. Backward and medial direction
C. Forward and lateral direction
D. Backward and lateral direction
77. In pterygomandibular space the inferior alveolar
nerve passes:
A. Medial to stylomandibular ligament
B. Anterior to the deep tendon of the temporal muscle
C. Lateral to the sphene mandibular ligament
D. Superior to lateral pterygoid muscle
78. Of the following which tissue has the least regen-
erating capacity after injury?
A. Bone
B. Liver
C. Peripheral nerve
0. Tendon
79. Sensory fibers of lingual nerve supplies to:
A. Tongue
B. Lingual surface of mandible
C. Floor of the mouth
0. All of the above
80. All of the following bones contain air sinuses ex-
cept:
A. Frontal
B. Nasal
C. Sphenoid
D. Ethmoid

76 D 77 c 78 D 79 D 80 B
LA/GA/PAIN
CONTROL

1. Epinephrine is added to local anesthetics because


it:
A. Decreases the rate of absorption of the local
anesthetic at the injection site
B. Prevents the rapid deterioration of the local
anesthetic solution
C. Increases the rate of destruction of the local anesthetic
D . Potentiates the action of all local anesthetics
2. The most likely cause of trismus after block anes-
thesia for surgery in the mandibular molar area is:
A. Excessive edema
B. Damage to the medical pterygoid muscle on injection
C. Stretching of the lateral pterygoid muscle
D. Submandibular cellulitis
3. The patient has received an injection of 1.8 ml of
local anesthetic containing 2 percent lidocaine with
1:100,000 epinephrine. Thirty seconds later he goes
into syncope. The most probable cause is:
A. Bradycardia
B. Tachycardia
C. Cerebral hypoxia
D. A toxic reaction to the lidocaine

1 A 2 B 3 c
18 MCQs in Oral Surgery

4. Nitrous oxide alone is not used as a general anes-


thetic agent because of the:
A. Difficulty in maintaining an adequate oxygen
concentration
B. Expense of the agent and its explosive hazard
C. Adverse effects on the liver
D. Poor analgesic properties
5. When attempting to achieve an intraoral palatal
second divison block anesthesia, the needle
should enter the:
A. Greater palatine foramen
B. Stylomastoid foramen
C. Nasopalatine foramen
D. Foramen rotundum
6. A deep level of general anesthesia is enhanced by
a:
A. · High alveolar concentration of anesthetic agent
B. Loose-fitting mask
C. Nonirritating drug
D. Muscle relaxant
7. A decrease in interstitial fluid pH will:
A. Decrease the effectiveness of a local anesthetic block
B. Increase the effectiveness of a local anesthetic block
C. Have no effect on the effectiveness of a local
anesthetic block
D. Decrease, them increase the effectiveness of a local
anesthetic block

4 A 5 A 6 A 7 A
LA/GA/Pain Control 19

8. Which of the following deep bony landmarks is im-


portant in performing a block of the second and
third divisions of the trigeminal nerve from the lat-
eral approach?
A. Mastoid bone
B. Styloid process
C. Temporal surface of the sphenoid
D. Lateral plate of the pterygoid process
9. A history of medication with which of the following
drugs requires special consideration prior to gen-
eral anesthesia?
A. Estrogen B. Cortisone
C. Meperidine D. Phenacetin
10. When administered in concentrations necessary to
produce sedation, nitrous oxide:
A. Does not produce nausea
B. Combines with hemoglobin
C. Involves no risk for the patient
D. Will depress the bone marrow and peripheral white
cell counts in man after prolonged use
11. Which of the following areas are anesthetized due
to the anterior (greater) palatine nerve block?
A. Incisors and canine on the injected side
B. Upper lip, nose and lower eyelid
C. Pos terior portion of the hard palate and overlying
structures up to the first premolar on the injected
side
D. Incisor to premolar on the injected side
12. Of the following which group of drugs eliminate all
sensations?
A. Anesthetics B. Analgesics
C. Narcotics D. Sedatives

8 D 9 8 10 0 11 c 12 A
20 MCQs in Oral Surgery

13. Of the following which causes anesthesia of the


lower lip?
A. Metastatic malignancy
B. CNS tumor
C. Fracture mandible body region
D. All of the above
14. For what purposes adrenaline is added to procaine?
A. Reduction of hemorrhage in the field of operation
B. Prevention of toxic ef.fects from too rapid absorption
C. Prolongation of anesthesia
D. All of the above
15. Following area of the brain is the "last" one de-
pressed by GA agents:
A. Medulla.
B. Cerebellum
C. Pons
D. None of the above
16. The presser effect of both epinephrine and nore-
pinephrine is increased by which of the following
local anaesthetic?
A. Procaine
B. Dibucaine
C. Cocaine
D. Lidocaine
17. A nerve is absolutely refractory during:
A. Depolarisation
B. After depolarisa tion
C. Hyperpolarisation
D. Firing level only

13 D 14 D 15 A 16 c 17 A
LAIGA/Pain Control 21

18. A nerve can be stimulated during relative refrac-


tory period by:
A. Stronger than normal stimuli
B. Sustained normal stimuli
C. Subthreshold stimuli
D. None of the above
19. Local anaesthetic agents act by:
A. Increasing the rate of depolarisation
B. Shortening the rate of repolarisation
C. Decreasing the threshold potential
D. Increasing the threshold potential
20. The local anaesthetic agent acts on:
A. Nerve membrane
B. Axoplasm
C. Epineurium
D. Perineurium
21. The most acceptable theory which explains the
actions of LA:
A. Surface charge theory
B. Calcium displacement theory
C. Membrane expansion theory
D. Receptor binding theory
22. The ultimate action of binding the receptor by agent
is brought about by its:
A. Hydrophilic component
B. Lipophilic component
C. Intermediary chain
D. RN of amide agents

18 A 19 0 20 A 21 0 22 A
22 MCQs in Oral Surgery

23. Amide group of local anaesthetic agents are dis-


pensed as salts of strong acids because:
A. They are not lipid soluble but stable in air
B. They are not wa ter soluble but stable in air
C. They are not water soluble and unstable in air
D. They are lipid soluble but stable in air
24. In acidic medium (during pyogenic infections) local
anaesthetics are less effective because:
A. More uncharged pa rticles a re released
B. Less uncharged particles a re released
C. Less charged particles are released
D. None of the above

25. Local anaesthetic agents with higher pka would


have:
A. Shorter onset of action
B. Longer onset of action
C. No affect on onset of action
D. None of the above
26. Which characteristic of a LA agent is responsible
for its penetration into the nerve?
A. Lipid solubility
B. Water solubility
C. Its ionisation
D . None of the above

27. Addition of a vasoconstrictor to LA agents:


A. Increases alkalinity of the solution
B. Increases acidity of the solution
C. Has no effect on the pH
D . None of the above

23 c 24 B 25 B 26 A 27 B
LA/GA/Pain Control 23

28. Sodium bisulfite has the following affect on the ac-


tion of LA solution:
A. Slows down its onset of action
B. Decreases its duration of action
C. Increases its pH
D. Has no affect
29. Increasing the concentration of LA from 2 to 5%
would have:
A. Rapid onset and prolonged action
B. Onset would not be affected but action would be
prolonged
C. No change on action
D . Rapid onset and duration not affected·
30. The efficacy of benzocaine in inflamed area would
be:
A. Decreased B. Increased
C. Not altered D. Prolonged
31. The main barrier for diffusion of LA into the nerve
is:
A. Epineurium
B. Perineurium
C. Endoneurium
D. Neural membrane
32. Which fibres of the nerve are anaesthetised first:
A. Mantle fibres
B. Core fibres
C. Both are anaesthetised at the same time
D. None of the above

28 A 29 D 30 c 31 c 32 c
24 MCQs in Oral Surgery

33. The inadequate pulpal anaesthesia in presence of


adequate soft tissue anaesthesia can be due to:
A. Faulty technique
B. Insufficient penetration of core fibres
C. Insufficient penetration of fasciculi
D. Insufficient penetration of mantle fibres
34. Prolonged duration of action of drugs like etido-
caine and bupivacaine can be attributed to their:
A. Increased lipid solubility
B. Increased water solubility
C. Increased protein binding
D. Vasoconstrictor action
35. Tachyphylaxis occurs due to :
A. Increased dose of LA
B. Increased dose of vasoconstrictor
C. Repeated use of LA
D. Allergy to sodium metabisulfite
36. Which of the following belongs to ester group of
local anaesthetics?
A. Bupivacaine B. Benzocaine
C. Etidocaine D. Mepivacaine
37. The only local anaesthetic with vasoconstrictor
properties is:
A. Cocaine B. Procaine
C. Benzocaine D. Lidocaine
38. Which of the following is not a ester local anaes-
thetic?
A. Propoxycaine B. Procaine
C. Prilocaine D. Piperacaine

33 B 34 c 35 c 36 B 37 A 38 c
LAIGA/Pain Control 25

39. Which of the following local anaesthetics crosses


the blood-brain barrier:
A. Lignocaine B. Etidocaine
C. Bupivacaine D. All of the above
40. All of the local anaesthetics cross the placenta:
A. Except mepivacaine
B. Statement is true
C. Statement is false
D. Except lignocaine
41. Ester type local anaesthetics are metabolised in the:
A. Liver only B. Kidney
C. Plasma D. Lungs
42. A patient who had a history of prolonged apnoea
during administration of muscle relaxant (suc-
cinylcholine) should not be given?
A. Ester local anaesthetics
B. Amide local anaesthetics
C. Vasoconstrictors
D. Sodium bisulfite
43. A patient complains of history of hepatitis one month
ago should be preferably given which local anaes-
thetic agent?
A. Lignocaine B. Bupivacaine
C. Procaine D. Procainamide
44. One of the complications of prilocaine LA is:
A. Agranulocytosis
B. Hepatic dysfunction
C. Methemoglobinemia
D. None of the above

39 D 40 8 41 c 42 A 43 c 44C
26 MCQs in Oral Surgery

45. Local anaesthetics are excreted mainly by:


A. Lungs B. Fecal route
C. Kidneys D. Uterus
46. Blood level of > 7 f.l.g/ml of LA produces:
A. Anticonvulsive activity
B. Tonic clonic seizure
C. CVS stimulation
D. None of the above
47. The anticonvulsant activity of-local anaesthetic
agents occurs at:
A. <4 J.l.g/ml
B. 4-7 ~tg/ml
C. 7-10 J.l.g/ml
D. 10-12 J.l.g/ ml
48. The differentiating factor between LA toxicity and
developing syncope would be:
A. CNS stimulation
B. CNS depression
C. Pallor of skin
D. Light reflex
49. The level of 2% lidocaine which reaches blood af-
ter use of one or two cartridges is:
A. 0.5-2 J.l.g/ ml
B. 1.5-5 J.l.g/ml
C. 5-10 J.l.g/ ml
D. No LA reaches blood if given safely after aspiration
50. A patient with known history of hyperthermia should
be given:
A. Lignocaine B. Procaine
C. Bupivacaine D. Mepivacaine

45 c 46 B 47 A 48 A 49 A 50 B
LAIGA/Pain Control 27

51 . With overdose of local anaesthetic agent one would


observe:
A. Hypertension
B. Hypotension
C. No change in BP
D . Card iac arrh ythmias
52. Epinephrine (Adrenalin) which is used in dental
cartridge of LA acts on:
A. a receptors only
B. Preceptors only
C. a and P receptors but p predominantly
D. a and P receptors but a predominantly
53. When local anaesthetic agent with adrenalin is in-
jected, the termination of activity of the vasocon-
strictor is brought by:
A. Adrenergic nerve endings
B. Blood enzymes COMT and MAO
C. Excretion in urine (80%)
D . A and B

54. Use of norepinephrine in dental practice is not rec-


ommended because it causes:
A. Bradycardia
B. Intense peripheral vasoconstriction
C. Hypertension
D . Sensitisation of myocardium
55. When one has to use the weakest vasoconstrictor
(e.g . in patient with history of angina) one should
consider:
A. Epinephrine
B. Norepinephrine ·
C. Phenylephrine
D. Levonordefrin

51 B 52 c 53 D 548 55 c
28 MCQs in Oral Surgery

56. Rebound phenomenon is most commonly seen with


use of:
A. Epinephrine
B. Norepinephrine
C. Phenyleplu·ine
D. Levonordefrin
57. The absolute contraindication for use of adrenalin
inLAis:
A. Myocardial infarction, 3-6 months ago
B. Angina pectoris
C. Hyperthyroidism
D. Pregnancy
58 . Adrenalin should not be used when halothane is
used during GA because halothane:
A. Sensitises the myocardium to adrenalin
B. Increases the heart rate
C. Increases the blood pressure
D. Interferes with AV conduction
59. A cartridge of LA contains 1:200,000 adrenalin, it in·
dicates that there is:
A. 0.005 mg/ ml of adrenalin
B. 0.065 mg/ ml of adrcnalin
C. 0.0125 mg/ ml of adrenalin
D. 0.02 mg/ ml of adrenalin
60. Maximum dose of adrenalin which can be given to
a patient with history of cardiovascular disease is:
A. 0.2 mg/ ml
B. 0.2 mg
C. 0.04 mg
D. 0.005 mg/ml

56 A 57 c 58 A 59 A 60 c
LA/GA/Pain Control 29

61. To a patient of 50 kg wt how many cartridges of LA


with 1: 200,000 adrenalin can be given (consider-
ing patient is normal, healthy and lignocaine toxic-
ity is not considered):
A. 10.5 B. 22
C. 32 D . 40

62. Lidocaine was first prepared by:


A. Nils Lofgren
B. Ekenstam
C. Einhorn
D. None of the above
63. The duration and depth of pulpal anaesthesia with
lignocaine (2%) added to 1:50,000 epinephrine as
compared to 1:100,000 epinephrine would be:
A. Longer and profound
B. Dura tion wou ld be two times longer bu t depth
would be same
C. No much difference
D. Duration would be 4 times
64. Three percent lignocaine indicates that there is:
A. 25 mg/ml of lignocaine
B. 30 mg/ ml of lignocaine
C. 54 mg/ml of lignocaine
D . 27 mg/ml of lignocaine
65. How many cartridges of 2% lignocaine can be given
to a 50 kg man (with adrenalin)?
A. 6
B. 12
c. 17
D. 25

61 B 62 A 63 c 64 B 65 A
30 MCQs in Oral Surgery

66. When vasoconstrictor is contraindicated, the ideal


local anaesthetic would be:
A. Lignocaine B. Mepivacaine
C. Cocaine D. Bupivacaine
67. A patient with respiratory disease presents for treat-
ment, which drug should not be used:
A. Lignocaine B. Adrenaline
C. Prilocaine D. Mepivacaine
68. Which of the following is least toxic LA?
A. Lignocaine B. Mepivacaine
C. Propoxycaine D. Bupivacaine
69. When injecting Into relatively highly vascular area
as in posterior superior alveolar nerve block, one
should use needle with:
A. Smaller gauge
B. Larger gauge
C. Gauge does not matter
D. None of the above

70. One should use a needle which has:


A. Greatest angle of bevel
B. Minimum angle of bevel with tip lying in the centre
of the lumen
C. No bevel at all
D. None of the above
71. Which of the following is a long acting LA agent?
A. Mepivacaine
B. Bupivacaine
C. Prilocaine
D. Propoxycaine

66 B 67 c 68 D 69 B 70 B 71 B
LA/GA/Pain Control 31

72. The safest local anaesthetic agent:


A. Cocaine B. Procaine
C. Chloroprocaine D. Propoxycaine
73. Which local anaesthetic agent when used topi-
cally interferes with sulphonamide action:
A. Lidocaine base B. Lidocaine
C. Benzocaine D. Propoxycaine
74. Self-aspirating syringes, provide aspiration by:
A. Pulling the thumb ring
B. Negative pressure created due to elasticity of rubber
diaphragm
C. Pressure release on thumb disc
D. All of the above
75. The jet injectors are used to obtain:
A. Pulpal anaesthesia
B. Topical anaesthesia
C. Regional block
D. Nerve block also
76. The gauge of needle used in dental syringes
refers to:
A. Internal diameter of the lumen
B. External diameter of needle
C. Diameter of bevel only
D. Diameter of hub
77. Sodium bisulphite used in dental LA cartridge acts
as:
A. Antioxidant for adrenalin
B. Antioxidant for lignocaine
C. Antibacterial for lignocaine
D. Not used any more

72 c 73 D 74 D 75 B 76 A 77A
32 MCQs in Oral Surgery

78. The allergic reactions commonly seen following use


of cartridge of LA is due to:
A. Lignocaine
B. Vasoconstrictor
C. Methylparaben
D. Sodium metabisulphite
79. Glass LA cartridge should be sterilised by:
A. Autoclaving
B. Dry heat
C. Cold sterilisation
D. None of the above
80. If the diaphragm of cartridge is soaked in isopropyl
alcohol for purpose of antisepsis, it may result in:
A. Reduced anaesthesia
B. No anaesthesia
C. Long-term paraesthesia
D. None of the above
81. Local infiltration should be:
A. Paraperiosteal
B. Subperiosteal
C. Transeptal
D. None of the above
82. Infiltration is not successful for anaesthetising buc-
cal roots of:
A. Maxillary 1st permanent molar
B. Maxillary 1st deciduous molar
C. Mandibular 1st permanent molar
D. Decidous maxillary 1st molar

78 c 79 D 80 c 81 A 82 c
LA/GA!Pain Control 33

83. For posterior superior alveolar nerve one should


use:
A. Long needle (40 mm)
B. Short needle (25 mm)
C. Length not a criterion
D. Only bevel should be considered

84. Greater palatine foramen is present:


A. Between 1st and 2nd maxillary molars
B. Between 2nd and 3rd maxillary molars
C. Distal to 3rd maxillary molar
D. Mesial to 1st maxillary molar
85. In 80% of patients infraorbital nerve block is effec-
tive for buccal aspect of:
A. Central incisors and canines
B. Central incisors to 1st premolars
C. Central incisors to mesiobuccal root of 1st maxillary
molar
D. Central incisors only
86. In greater palatine nerve block the needle should
be:
A. Parallel to mucosa
B. Perpendicular to mucosa
C. 45° inclined to mucosa
D. Parallel to roots of molars
87. The two techniques used for maxillary block are:
A. Greater and lesser palatine approach
B. Greater palatine and high tuberosity approach
C. Greater tuberosity and retromolar approach
D. Gow gates and Akinosi technique

83 B 84 B 85 c 86 B 87 B
34 MCQs in Oral Surgery

88. Inferior alveolar nerve block anaesthetises all


except:
A. Body of mandible lower part
B. Mandibular teeth
C. Mucous membrane anterior of first mandibular molar
D. Mucous membrane distal to 1st mandibular molar
89. The needle while giving inferior alveolar nerve
block passes through:
A. Buccinator muscle
B. Pterygomandibular raphe
C. Buccal fat
D. Stylomandibular raphe
90. While giving inferior alveolar nerve block the
needle is lateral to:
A. Lingual nerve
B. Sphenomandibular ligament
C. Medial pterygoid muscle
D. All of the above
91 . If bone is not contacted before injecting local
anaesthetic in inferior alveolar nerve block, there
are chances of:
A. Lingual nerve anaesthesia
B. Transient facial palsy
C. Transient maxillary anaesthesia
D. None of the above
92. Inferior alveolar nerve block at times is not very
effective because which of the nerves is not
anaesthetised:
A. Mental nerve
B. Lingual nerve
C. Mylohyoid nerve
D. Incisal nerve

88 D 89 A 90 D 91 8 92 c
LAIGA/Pain Control 35

93. Gow gates technique is for:


A. Mandibular nerve block
B. Inferior alveolar nerve block
C. Trigeminal ganglion block
D. v2 Vy block
I

94. The target of Gow Gates technique is:


A. Coronoid notch
B. Sigmoid notch
C. Foramen ovale
D. Neck of condyle
95. In patients with reduced mouth opening which tech-
nique of mandibular anaesthesia should be used:
A. Gow gates
B. Akinosis
C. Labyrinths
D. Williams
96. For extraoral maxillary nerve block the target area ·
is:
A. Posterior to lateral pterygoid plate
B. Anterior to lateral pterygoid plate
C. Pterygomandibular fissure
D. Pterygomandibular fossa
97. For extraoral mandibular nerve block the needle
should be inserted from:
A. Above the zygomatic arch
B. Below the zygomatic arch
C. Coronoid notch
D. None of the above

93 A 94 0 95 B 96 B 97 B
36 MCQs in Oral Surgery

98. If needle breaks during injecting LA and radio-


graphically it appears to be deep in tissues, the
advised management would be:
A. Removal of needle under LA
B. Removal of needle under GA
C. Leaving the needle in the tissue
D. None of the above
99. Use of which of the solutions relatively can have
more burning sensation?
A. Plain lignocaine
B. Isotonic solution
C. Lignocaine and adrenalin
D. Hypotonic solution
100. Persistent anaesthesia can result most often in
which of the nerves, after LA injection?
A. Inferior alveolar
B. Lingual nerve
C. Infraorbital nerve
D. Mental nerve
101. Aspiration should be carried out at least in:
A. One plane
B. Two planes
C. Three p lanes
D. Four planes
102. ECG changes can first be observed when level of
lignocaine is more than:
A. 5-6 J..Lg/ ml
B. 10-12 J..Lg/ ml
C. 2-4 J..Lg/ ml
D. 1-2 J..Lg/ ml

98 c 99 c 100 B 101 B 102 A


LA/GA/Pain Control 37

103. To control tonic clonic seizures following lignocaine


toxicity the drug of choice would be:
A. Pentobarbital
B. Diazepam
C. Succinylcholine
D. Antihistaminics
104. Succinylcholine can be used for control of tonic
clonic seizures but along with this:
A. Pentobarbitone should be used
B. Artificial respiration is must
C. Atropine should be given
D. Neostigmine should be given to terminate its action
105. Postictal phase, which follows CNS toxicity with li-
gnocaine should be managed by:
A. CNS stimulants
B. Analeptics
C. Supportive therapy only
D. Diazepam
106. During CVS depression in lignocaine toxicity, one
should administer:
A. Vasoconstrictors
B. Atropine
C. Crystalloids
D. All of the above
107. Gingival retraction cords contain adrenalln in con-
centration of:
A. 1:200,000/ inch
B. 0.3 J..tg/inch-1.0 J..tg/ inch
C. 300 J..lg/ inch-1000 J..lg/inch
D. 1000 J..lg/inch-2000 J..lg/inch

103 B 104 B 105 c 106 D 107 c


38 MCQs in Oral Surgery

108. Extraction of d~ciduous teeth in dental clinic can


be carried out in which stage of general anaesthe-
sia:
A. Stage I
B. Stage II
C. Stage III, plane II
D. Stage III plane III
109. Surgical plane for major surgery during general
anesthesia is. during:
A. Stage III plane I
B. Stage III plane II
C. Stage III plane III
D . Stage HI plane IV
110. Succinylcholine Is administered during GA for:
A. Better control
B. Intubation
C. Prevention of apnoea
D. Decreasing respiratory rate and thus decreasing GA
toxicity

111. Nowadays induction phase of GA has been reduced


because of use of:
A. Halothane
B. Ether, halothane combination
C. Thiopentone sodium
D. Ether, N 2 0, halothane combination
112. If long acting muscle relaxants are used during GA,
their action Is terminated by use of:
A. Neostigmine
B. Atropine
C. Ketamine
D. Succinylcholine

108 A 109 c 110 B 111 c 112 A


LA/GA/Pain Control 39

113. The N20 gas cylinder used in GA is:


A. White and black coloured
B. Blue coloured
C. Red and yellow coloured
D. White and blue coloured
114. For maxillofacial injuries one should always use
which endotracheal tube for GA?
A. Non inflatable
B. Inflatable cuffed
C. Catheterized
D. None of the above
115. The endotracheal tube should be placed for GA:
A. In right bronchus
B. In left bronchus
C. Above cricoid
D. In laryngopharynx
116. Rotameter on Boyle's trolley for GA is used to mea-
sure:
A. Pressure of gas in the cylinders
B. Pressure of halothane
C. Flow of gases in the tubes
D. None of the above
117. Goldman's vapourizer is used for:
A. N 20
B. 50% N 20 + 20% 0 2 mixture
C. 50% N 20 +50% 0 2 mixture
D. 50% either+ 20% 0 2 mixture

113 B 114 B 115 c 116 c 117 B


40 MCQs in Oral Surgery

118. In TMJ ankylosis patient, GA cannot be administered


by:
A. Oral intubation
B. Blind nasal intubation
C. Fiberoptic assisted intubation
D. Band C
119. In TMJ ankylosis patient, GA can be administered
by:
A. Oral intubation
B. Blind nasal intubation
C. Fiberoptic assisted intubation
D. Band C
120. A patient who is being operated under halothane
should not be given:
A. Lignocaine
B. Lignocaine + adrenalin
C. Propoxycaine
D. Mepivacaine
121. Glycopyrolate Is usually used during GA to:
A. Reduce heart rate
B. Increase BP
C. Reduce secretions
D. Control bleeding
122. In most surgical procedures, GA with N2 0 is given
as:
A. 70% N 20 + 30% 0 2
B. 70% N20 + 20-30% 0 2 + other GA agent
C. 50% N 20 + 50% 0 2 + other GA agent
D. 70% 0 2 + 20% N 20

118 A 119 D 120 8 121 c 122 D


LA/GA/Pain Control 41

123. Which of the following is used as a dissociative


anaesthetic agent:
A. Fentanyl B. Thiopentone
C. Ketamine D. Halothane+ either mixture
124. During GA oxygen concentration of blood should
not fall below:
A. 90% B. 60%
c. 40% D. 20%
125. Local anesthetics are most effective in tissue that
have what pH?
A. Below 7
B. Above 7
C. Below 4
D. Makes no difference what the pH of the tissue is
126. A 40-year-old male develops right facial and head
pain which is intermittent with abrupt onset and
cessation. It usually occurs at night awakening him
from sleep. The pain is associated with unilateral
flushing, sweating, rhinorrhea and increased
lactimation. What is the most likely diagnosis?
A. Tic douloureux
B. Vidian neuralgia
C. Sphenopalatine ganglion neuralgia
D. Histamine cephalgia
127. Which local anesthetic listed below may possibly
manifest its toxicity clinically by Initial depression
and drowsiness rather than stimulation and con-
vulsion?
A. Lidocaine B. Procaine
C. Benzocaine D. Tetracaine

123 c 124 A 125 8 126 0 127 A


42 MCQs in Oral Surgery

128. How will a larger than normal functional residual


capacity affect?
A. Nitrous oxide sedation
B. Nitrous oxide sedation will happen much quicker
C. Nitrous oxide sedation will take longer
D. Functional residual capacity does not effect nitrous
oxide sedation
129. A patient who is allergic to para-aminobenzoic acid
will also be allergic to which of the following:
A. Lidocaine B. Tetracaine
C. Prilocaine D. Carbocaine
130. Nitrous oxide works on which system listed below?
A. Peripheral nervous system (PNS)
B. Central nervous system (CNS)
C. Autonomic nervous system' (ANS)
D. None of the above
131. During an inferior alveolar block injection, the
needle passes through the mucous membrane and
the buccinator muscle and lies lateral to which
muscle listed below?
A. Masseter
B. Lateral pterygoid
C. Hypoglossus
D. Medial pterygoid
132. After receiving an injection of a local anesthetic
containing 2 percent lidocaine with 1:1,00,000 epi-
nephrine, the patient loses consciousness. Which
of the following is the most probable cause?
A. Acute toxicity
B. Allergic response
C. Syncope
D. Hyperventilation syndrome

128 B 129 B 130 B 131 D 132 c


LAIGA/Pain Control 43

133. How many milligrams of epinephrine are in each


cartridge (1.8 cc) of 2% lidocaine with 1:1,00,000
epinephrine?
A. 0.018 mg B. 18 mg
C. 0.036 mg D. 36 mg
134. Local anesthetics act directly on the nerve mem-
brane in what way?
A. Decrease the membrane's permeability to sodium.
B. Increase K+ flux
C. Increase the membrane's permeability to sodium
D. Increase membrane excitability
135. Phlebitis of a vein after administration of IV valium
is usually attributed to the presence of which of
the following in the mixture?
A. Hydroquinone
B. Water
C. Alcohol
D. Propylene glycol
136. Which drug listed below is most commonly used to
attain general anesthesia?
A. Valium
B. Chloral Hydrate
C. Phenargan
D. Methohexital
137. Which of the following Is a peculiar thermal alter-
ation that occurs during surgery in susceptible
persons?
A. Malignant hypothermia
B. Heat stroke
C. Malignant hyperthermia
D. None of the above

133 A 134 A 13~ D 136 D 137 A


44 MCQs in Oral Surgery

138. The maximum allowable do59 for 2% lidocaine with


1:1,00,000 epinephrine is what?
A. 2.0 mg lidocaine I per pound
B. 3.5 mg lidocaine/per pound
C. 5.0 mg lidocaine/ per pound
D. 7.5 mg lidocaine/ per pound
139. Which vein listed below is the optimum site for IV
sedation for an outpatient?
A. Median basilica
B. Median cephalic
C. Median antebrachial
D . Angular
140. What is usually the first clinical sign of mild
lidocaine toxicity?
A. Itching B. Nervousness
C. Vomiting D. Sleepiness
141. The CNS and respiratory depressant effects of bar-
biturates can be enhanced by concomitant use of:
A. Phenothiazines
B. Alcohol
C. Both of the above
D. None of the above
142. Productive cough, often with wheezing, is a
universal factor of which disease:
A. Emphysema
B. Congestive heart failure
C. Chronic bronchi tis
D. Hepatitis

138 B 139 B 140 B 141 c 142 c


LA/GA!Pain Control 45

143. The amount of vasoconstrictor in 1 ml or 2%


lignocaine solution with 1:2,00,000 adrenaline is:
A. 0.5 mg B. 0.05 mg
C. 0.005 mg D. 0.0005 mg
144. The latest drug of choice in the management of
trigeminal neuralgia is:
A. Valproic acid
B. Carbamazepine
C. Diphenhydantoin
D. Carbazepine
145. Medical oxygen is stored in steel cylinders painted:
A. Black B. Blue
C. White D. Green
146. How much lignocaine is present in 2.0ml of 2%
lignocaine solution?
A. 42.6 mg B. 21.3 mg
C. 32.1 mg D. 02.31 mg
147. Following a right inferior alveolar nerve block, the
patient experiences difficulty in closing the eyelid
of the right side. The problem cause is:
A. Anaesthesia of the motor branch of the mandibular
nerve
B. Anaesthesia of the facial nerve within the parotid
gland
C. Anaesthesia of an aberrant branch of the cervical calli
nerve
D. Anaesthesia of the infra-orbital nerve because the
injection was too high

143 c 144 0 145 A 146 A 147 B


46 MCQs in Oral Surgery

148. Local anaesthetics produce anaesthesia by:


A. Depolarising a nerve
B. Inhibiting the influx of sodium ions through the never
membrane
C. Creating negative after potentials
D. Increasing the outflow of Potassium ions through the
nerve membrane
149. When blocking a nerve containing both motor and
sensory fibers, the last functional property lost is?
A. Temperature B. Pain
C. Proprioception D. Touch
150. Sodium meta-bisulfite is added in local anaesthetic
solutions to:
A. Act as a preservative
B. Maintain the pH of the solution
C. Act as an anti-oxidant for adrenaline
D. Prevent the spontaneous dissociation of lignocaine
in solution
151. In tracheostomy, the entry into the trachea Is
through the:
A. Cricoid cartilage
B. First tracheal ring
C. Second and third tracheal ring
D. Fourth and fifth tracheal ring
152. The following Is true for percutaneous tracheo-
stomy, expect:
A. No pretracheal dissection is required
B. Can be carried out at the bedside
C. A transverse incision is used
D. Treatment of choice in intubated patient

148 B 149 c 150 c 151 c 152 c


LA/GA/Pain Control 47

153. Trismus after an inferior alveolar nerve block most


likely results from:
A. Damage to the inferior alveolar nerve
B. Damage to the Medial pterygoid muscle
C. Myositis of the buccinator muscle
D. Accidental blockade of a major branch of the
trigeminal nerve motor
154. The maximum safe dose of adrenaline that can be
given in a local anaesthetic solution to a healthy
dental out patient is:
A. 0.002 mg B. 0.2 mg
C. 2.0 mg D. 0.02 mg
155. The ideal rate If injection of a local anesthetic solu-
tion is:
A. 1.00 ml/min B. l.SOml/min
C. 1.80 ml/min D. 2.00ml/min
156. The aims of preanaesthetic medication Include the
following, except:
A. Allaying apprehension
B. Decreasing oral and gastric secretions
C. Inceresing reflex irritability
D. Increasing the pain threshold
157. Gustatory swelling is the chief symptom in:
A. Sjogren's syndrome
B. Frey's syndrome
C. Bell's palsy
D. Sialolithiasis

153 B 154 B 155 c 156 c 157 B


48 MCQs in Oral Surgery

158. The purpose of taping the eyes shut before a pa-


tient is draped for surgery under GA is to:
A. Prevent lacrimal secretions from contaminating the
field
B. Prevent corneal abrasions
C. Limit ocular motility
D. All of the above
159. During an oral surgical procedure under general
anaesthesia, the anaesthetic technique best suited
to avoid aspiration of blood or other debris is:
A. Open drop
B. Nasopharyngeal intubajion
C. Endotracheal intubation with cuffed tube
D. Oropharyngeal intubation
160. Allergic reactions in patients who receive amide
type local anesthetic solutions for tooth extractions
are most likely caused by a reaction to:
A. Lignocaine hydrochloride
B. Adrenaline bitartrate
C. Methylparaben
D. Sodium meta-bisuphite
161. Extra-oral injection for obtaining the anaesthesia
of the entire second division of the fifth cranial
nerve is given In/around the:
A. Infraorbital foramen
B. Foramen ovale
C. Pterygopalatine fissure
D. Foramen spinosum

158 8 159 c 160 c 161 c


LAIGA/Pain Control 49

162. After receiving a posterior superior alveolar block


the patient experiences:
A. Numbness of the molar teeth
B. Numbness of the upper lip
C. Numbness of the side of the face
D. Few subjective signs of anaesthesia
163. Which of the following drug is used to manage an
acute urticarial reaction after administration of a
local anaesthetic?
A. Intravenous injection of aminophylline
B. Inhalation of aromatic spirits of ammonia
C. Intravenous injection if hydrocortisone methyl
acetate
D. Intravenous injection of diphenhydramine
164. The most common postoperative complication of
outpatient general anaesthesia is:
A. Syncope B. Nausea
C. Atelectasis D. Shivering
165. The piercing of the infra orbital nerve with a 26
gauge needle while giving a block in the canine
fossa produces:
A. Lancinating pain in the lower eyelid
B. A mild, temporary paraesthesia of the upper lip
C. Destruction of the nerve
D. Haematoma formation in the region
166. Vascular compression of the gasserin ganglion by
the following blood vessel is thought to be
responsible for tic:
A. Superior cerebellar artery
B. Superior cerebral artery
C. Middle cerebral artery
D. Antero-inferior cerebellar artery

162 D 163 0 164 B 165 B 166 A


50 MCQs in Oral Surgery

167. A patient with upper motor lesions of the facial


nerve presents with the following clinical appear-
ance except:
A. Sagging of the corner of the mouth
B. Flattening of the nasolabial furrow
C. Loss of eyebrow and forehead w rinkles
D. Incompetent lip seal
168. Endotracheal intubation is only possible in the
following stage of general anaesthesia:
A. Stage I
B. Stage III, Plane 1
C. Stage III, Plane 2
D. Stage III, Plane 3
169. The important sign a patient exhibits when the cor-
rect level of sedation is reached while using diaz-
epam sedation Is?
A. Blurring of vision
B. Slurring of speech
C. Loss of gag reflex
D . 50% ptosis of the eyelids
170. In general anaesthesia the loss of eyelash reflex
occurs In:
A. Stage I
B. Stage II
C. Stage III, Plane 1
D . Stage III, Plane 4

167 c 168 c 169 D 170 c


LA/GA/Pain Control 51

171. Guedel's criteria for classification of the depth of


general anesthesia are based on:
A. Respiration, eyeball movement and presence and
absence of various reflexes
B. Response to skin incision
C. Pharyngeal and laryngeal reflexes
D. Respiration, eyelid reflex and secretion of tears
172. According to the gate-control theory of pain, im-
pulses traveling in the A fiberes tend to
A. Inhibit the substansia geletinosa cells thereby
blocking the pain stimuli transmitted by the C fibers
B. Excite the substansia gelatinosa cells thereby blocking
the pain stimuli transmitted by the nonmyelinated
fibers
C. Be associated with pain of inflammatory origin
D. Bypass the central control process
173. The "burning" felt at the site of venipuncture after
administration of diazepam can be attributed to:
A. Propylene glycol in the mixture
B. Benzoic acid in the mixture
C. Use of a blunt or large diameter needle
D. Poor venipuncture technique
174. Nitrous oxide-oxygen inhalation sedation is con-
traindicated in patients with:
A. Nasal obstruction
B. Emotional instability
C. Emphysema
D. All of the above

171 A 172 B 173 A 174 D


52 MCQs in Oral Surgery

175. Dissociative analgesia is produced using the


following drug combination:
A. Ketamine and scopolamine
B. Fentanyl and droperidol
C. Pentobarbital and meperidine
D. Diazepam and atropine
176. The minimum distance a nerve should be covered
by the anaesthetic solution to ensure thorough
blockade of the nerve Impulse, is:
A. 3-Smm
B. 5-8 mm
C. 8-10 mm
D. 10-14mm
177. The absolute contraindication to the use of a local
anesthetic solution with epinephrine Is in a patient
with:
A. Uncontrolled hypertension
B. Untreated hyperthyroidism
C. Diabetes mellitus
D. Ventricular fibrillation
178. The pH of the tissues does not effect the effective-
ness of this anesthetic agent:
A. Bupivacanie B. Procaine
C. Benzocaine D. Mepivacaine
179. The most potent vasodilator local anaesthetic agent
is:
A. Cocaine
B. Lignocaine
c. Tetracaine
D. Procaine

175 A 176 c 177 B 178C 179 D


LAIGA/Pain Control 53

180. The following incision is used to gain assess to the


medial surface of the mandibular ramus, for infe-
rior alveolar neurectomy:
A. Risdon's incision
B. Ginwall's incision
C. Bayonet incision
D. Craw's foot incision
181. Of the following which is most suitable anaesthe-
sia for the extraction of a deciduous molar?
A. Inferior alveolar nerve block
B. Surface anaesthesia
C. Local infiltration
D. None of the above
182. Most reactions following injection of LA are caused
by:
A. Allergy B. Anxiety
C. Toxicity D. Over dosage
183. Hematoma formation after the posterior superior
alveolar nerve block is due to the damage to:
A. Internal maxillary artery
B. Small capillaries and arterioles in the region
C. Pterygoid venous plexus
D. Facial artery
184. Two minutes following cessation of a two hour treat-
ment session using 80% N2 0 and 20% 0 2 and local
anesthesia. The patient becomes cyanotic and
tachycardiac. Most probable cause Is:
A. Diffusion hypoxia
B. Over oxygenation
C. Under oxygenation
D. Malignant hyperthermia

180 B 181 A · 182 B 183 c 184 A


54 MCQs in Oral Surgery

185. Level of analgesia is best monitored by:


A. Eye movements and absence of corneal reflex
B. Eye movements and absence of conjunctival reflex
C. Respiration, rate, rhythm and type of respiratory
movements
D. Verbral response
186. Among the following which is the optimum site for
intravenous sedation in an outpatient?
A. Jugular vein
B. Median cephalic vein
C. Dorsal vein back of the hand
D. Popliteal vein
187. Which respiratory conditions is most alarming dur-
ing patient sedation in dental hospital?
A. Tachypnoea
B. Dyspnoea
C. Apnoea
D. Hyperpnoea
188. The common anesthetic complication occurring
within fir.s t day after surgery under GA is:
A. Cardiac failure
B. Renal failure
C. Atelectasis
D. Hypotension
189. According to Gate control theory of pain, large fi-
ber impulses tend to:
A. By pass the central control process
B. Inhibit the effect of painful stimuli transmitted by
small fibers
C. Transmit severe .chronic pain only
D. Potentiate the effect of painful stimuli

185 0 186 B 187 c 188 c 189 B


LAJGA/Pain Control 55

190. In case of mild lidocaine toxicity, the first clinical


sign is:
A. Nervousness
B. Lethargy
C. Convulsions
D. Tachycardia
191. Which of the following is the cause of allergies to
local anesthetics?
A. Slow elimination
B. Improper technique
C. Slow detoxification
D. Antigen and antibody reactions
192. The following drugs potentiates the action of seda-
tive drugs:
A. Phenothiazines B. Propanol
C. Methyldopa D. Digitalis
193. Each cartridge contains how many mi. of anesthetic
solution:
A. 0.9 ml B. 1.2 ml
C. 1.8 ml D . 2.2 ml
194. How many mg of 2% lidocaine present in the
cartidge?
A. 36 mg B. 18 mg
C. 180 mg D. 360mg
195. How many mg of adrenaline are In each cartridge
of 2% lignocaine with 1:1 ,00,000 adrenaline:
A. 0.36 mg
B. 0.036 mg
C. 0.018 mg
D. 0.15 mg

190 A 191 0 192 A 193 c 194 A 19:5 C


56 MCQs in Oral Surgery

196. Among the following in which patient GA is contrain-


dicated?
A. An adult male over 65 years of age
B. Patient with acute respiratory infection
C. Child under 6 years of age
D. Patient with wheel compensated heart disease
197. The following two l aboratory tests should be
considered before deciding whether to use
general anesthesia or not?
A. Total WBC count and prothrombin time
B. Total WBC count and urine analysis
C. Bleeding time and clotting time
D. Complete blood count and urine analysis
198. Nerve membrane stabilization action of local anes-
thetic agents is due to:
A. Increasing conductance of potassium ions
B. Creating negative after potentials
C. Preventing conductance of Na ions from the exterior
of the nerve to the interior
D. Preventing conductance of Na ions from the interior
to the exterior of the nerve
199. Thirty seconds after the injection of 1.8 ml of local
anesthetic containing 2% lidocaine with 1:1,00,000
adrenaline patient loses consciousness. The most
probable diagnosis is:
A. Toxic reaction to the adrenaline
B. Cerebral hypoxia
C. Tachycardia
D. Toxic reaction to the lidocaine

196 B 197 D 198 c 199 B


LAIGA/Pain Control 51

200. In which conditions N2 0 - 0 2 sedation in contraindi-


cated?
A. Emphysema
B. Upper respiratory obstruction
C. Emotional instability
D . All of the above
201 . Members of the sterile professional team include
A. Surgeon, assistan ts and scrub nurse
B. Surgeon, scaib nurse, and circulating nurse
C. Surgeon, scrub nurse, circula ti ng nurse and
anesthesiologist
D. A and B
202. One wants to anesthetise the inferior alveolar
nerve at the point at which it exists from the
mandibular canal, so the injection site most often
located in
A. Between first and second molars inferior to the root
apices
B. Between first and second molar su perior to root
apices
C. Between first and second premolars inferior to root
apices
D . Between first and second premolars superior to root
apices
203. Accidental intravenous injection of LA which con-
tains a vasoconstrictor may cause:
A. Convulsions and palpitations
B. Unconsciousness and depressed respiration
C. Increased respiration
D. Any of the above may occur

200 D 201 A 202 c 203 D


58 MCQs in Oral Surgery

204. Compared to inhalation, IV sedation:


A. Produces more; predictable amnesia
B. May produce more serious complications
C. May result in deeper sedation
D. All of the above are correct
205. Among the following which is least likely to block
reflexes for surgery completely when used alone?
A. Nitrous oxide
B. Halothane
C. Ethane
D. Thiopentone sodium
206. Likely cause of trimus after inferior alveolar block
anesthesia is:
A. Damage to medial pterygoid muscle during injection
B. Myositis of the external pterygoid muscle
C. Submandibular cellulitis
D. Stretching of the pterygomandibular raphe
207. Most common cause of death occurring under GA
is due to:
A. Reflex cardiac standstill
B. Over dosage of anaesthetic agent
C. Excessive carotid sinus stimulation
D. Airway obstruction with improper ventilation
208. Three common symptoms indicating the correct
level of sedation after diazepam administration is:
A. Blurring of vision, slurring of speed and loss of GAG
reflex
B. 50%ptosis eyelids, blurring of vision, slurring of
speech
C. Paresthesia of lips, tongue and fingers
D. Sweating over the face, itching of the nose and loss
of gagreflex

204 D 205 A 206 A 207 D 208 B


LA/GA/Pain Control 59

209. Preferable patient position in recovery room after


ambulatory general anesthesia is:
A. Supine ·B. Sitting
C. Prone D. Lateral
210. Among the following which nerve innervates the
buccal gingiva of the maxillary second premolar?
A. Anterior palatine nerve
B. Buccal nerve
C. Middle superior alveolar nerve
D. Posterior palatine nerve
211. Paraesthesia during a mandibular nerve block is
due to:
A. Sodium conductance through the nerve
B. Acid pH of the anaesthetic solution
C. Contact of the needle with the nerve trunk
D. Poor technique
212. Of the following in which condition local anaesthe-
sia is ineffective?
A. Edema
B. Localised infection
C. Hematoma
D. Anemia
213. The following anatomical land marks are useful In
the closed mouth approach of mandibular nerve
block except:
A. Occlusal plane of occluding teeth
B. Anterior border of ramus
C. Mucogingival junction of the mandibular teeth
D. A and B

209 D 210 c 211 c 212 B 213 c


60 MCQs in Oral Surgery

214. A contraindication to local Infiltration technique:


A. Hypertension
B. Diabetic patient in whose urine sugar is present
C. Infection in the arch
D. Kidney problems
215. The purpose of gas filled bubbles in anesthetic
cartridge is:
A. Indicate the level of anaesthetic agent
B. Prevent deterioration of the vasoconstrictor
C. Formed during the manufacturing process
D. A and C
216. Anterior palatine nerve is anaesthetized by ante-
rior palatine nerve block. The subjective symptoms
include:
A. Feeling of numbness in the posterior palate
B. Feeling of numbness in the anterior part of the palate
near the incisors
C. Feeling of numbness of whole palate
D. Little or no subjective symptoms
217. The primary site of local anaesthetic action is:
A. Medulla oblongata
B. Substantia gelatinous
C. Nerve membrane
D. Nodes of ranvier
218. Which is best monitor of the level of analgesia?
A. Eye movements
B. Respiratory movements
C. Muscle tone
D. Verbal response

214 c 215 B 216 A 217 c 218 D


LAIGA/Pain Control 61

219. When comparing to the nonmyelinated n·erve,


myelinated nerve requires:
A. Higher concentration of anaesthetic agent
B. Lower concentration of anaesthetic agent
C. Same concentration of anaesthetic agent
D. None of the above because no correlation
220. Local anaesthetic cartridges contain small bubbles.
These bubbles contain:
A. Oxygen gas
B. Carbon dioxide gas
C. Nitrogen gas
D. Carbon monoxide gas
221. Which of the following statements is true?
A. More concentration of H• ions in an area potentiate
anaesthetic agent action
B. Less concentration of H+ ions in an area potentiate
anaesthetic agent action
C. A little is known about the relation between H+ ion
concentration and anaesthetic activity
D. No correlation between H• ion concentration and
anaesth~tic activity

222. A surgical procedure required bilateral block. A bi-


lateral inferior alveolar nerve block is:
A. Not contraindicated
B. Should rarely be performed
C. May cause space infection
D. Dangerous because patient may swallow the tongue

219 A 220 c 221 B 222 A


62 MCQs in Oral Surgery

223. Risk of death in more in some patients with the use


of local anaesthesia with adrenaline. Which of the
following group of patients are called as walking
time bombs?
A. Cocaine abusers
B. H ypertensive
C. Hyperthyroidism patients
D . Patients under MAOI [MAO inhibitors] treatment
224. Subjective symptom of buccal nerve block is:
A. N umbness and tingling sensation of the buccal
m ucosa
B. Numbness of the ling ual mucosa near molars
C. No subjective symptoms
D . A and B

225. Ester group of local anaesthetics mainly inactivated


by:
A. H ydroxylation B. Oxidation
C. H ydrolysis D. None of the above
226. Among the following, which condition is a contrain-
dication to the use of local anaesthetic agent?
A. Parkinson's disease
B. Liver damage
C. Pregnancy (3rd trimerister)
D. Hypersensitivity to the d rug
227. Amide type of local anaesthetic agents undergo
biotransformation primarily in the:
A. Kidney
B. Liver
C. Plasma
D. No biotransformationoccursexcreted in all unaltered
from

223 A 224 c 225 c 226 D 227 B


LA/GA/Pain Control 63

228. Ester type local anaesthetic agents undergo


biotransformation in the:
A. Kidney
B. Liver
C. Plasma
D. B and C

229. Of the following in which condition general anaes-


thesia is contraindicated?
A. H em oglobinopathy
B. Allergy
C. Diabetes mellitus
D. All of the above
230. The following is least toxic of all local anaesthetics:
A. Tetracaine
B. Mepivacaine
C. Prilocaine
D. 2-chloro procaine
231. Cocaine Is:
A. Vasodilator in action
B. Vasoconstrictor in action
C. Synthetic
D. Synthetic and vasodilator in action
232. Adrenaline is added to local anaesthetic agents
because it:
A. Potentiate the action of all local anaesthetic agents
B. Incre ases the ra te of des tr u cti on of the local
anaesthetic agent
C. Decreases the r a te of absorption of the loca l
anaesthetic at the injection site
D. Prevents deterioration of the an aesthetic solution

228 D 229 A 230 D 231 B 232 c


64 MCQs in Oral Surgery

233. Sedation by which of the following routes can be


reversed rapidly?
A. Intravenous
B. Oral
C. Inhalation
D. Intramuscular
234. Only nitrogen oxide alone is not used as a general
anaesthetics agent because of the:
A. Poor analgesic properties
B. Expense of the agent and its explosive hazard
C. Difficulty in maintaining an adequate oxygen
concentration
D. Hepatotoxic nature
235. After the general anaesthesia, vomiting may occur.
The immediate postoperative complication follow-
ing aspiration of liquid vomitus into trachea and
bronchi is:
A. Lung abscess B. Bronchitis
C. Pleurisy D. Atelectasis
236. Which of the following side effects is seen com-
monly with the administration of nitrous oxide and
oxygen?
A. Hallucinations and dreams
B. Tachycardia
C. Tremors
D. Nausea
237. In which stage of general anaesthesia surgery is
performed?
A. Stage I B. Stage II
C. Stage III D . Stage IV

233 c 234 c 235 B 236 D 237 c


L.A/GA/Pain Control 65

238. The following areas are anaesthetized by the ante-


rior palatine nerve block:
A. Part of the upper lip, lateral surface of nose and lower
eyelid
B. Posterior portion of hard palate and overlying tissues
upto the first premolar
C. Incisors canine and first premolar
D. Both hard and soft palates completely
239. The following deep bony landmarks is important in
performing a block of the II and Ill divisions of the
trigeminal nerve from the lateral approach:
A. Temporal surface of sphenoid
B. Perpendicular plate of the palatine bone
C. Lateral plate of the pterygoid process
D. Styloid process
240. For a therapeutic procedure bilateral mandibular
nerve block is required. Bilateral mandibular nerve
block is:
A. Should perform rarely
B. Not contraindicated
C. Dangerous because tongue may fall back and patient
D. May cause sever trismus due to space infection
241. Facial paralysis after Inferior alveolar nerve block
is due to:
A. Injection into the parotid gland
B. Aberent facial nerve
C. Sympathetic stimulation
D. All of the above

238 B 239 c 240 B 241 D


66 MCQs in Oral Surgery

242. Among the following items in the past medical his-


tory, which would most affect the choice of local
anaesthetic agent?
A. RHD
B. Hyperthyroidism
C. Adrenal insufficiency
D. Asthma
243. The main function of atropine in preoperative medi-
cation is to:
A. Depress gastrointestinal mobility and relieve anxiety .
B. Reduce the salivary secretion and sedation
C. Reduce secretions and depress vagal reflex
D. Reduce bronchial secretions and enhance vagal reflex
244. In the extraoral infraorbital nerve block the follow-
ing nerve is not involved:
A. Anterior superior alveolar
B. Middle superior alveolar
C. Sphenopalatine
D. Lateral nasal
245. After giving an inferior alveolar nerve block patient
develops paralysis of the muscles of the forehead,
eyelids and upper and lower lips on the same side
of the face. This is probably due to:
A. Anaesthesia of the motor branches of the mandibular
nerve
· B. Diffusion of the anaesthetic solution involving the
ophthalmic division of the trigeminal
C. Injection of the solution into the capsule of the
parotid gland and consequent block of the facial
nerve
D. Diffusion of the solution through the inferior orbital
fissure and anaesthetised the oculomotor nerve

242 B 243 c 244 c 245 c


LAIGA/Pain Control 67

246. Among the following which of the following are po-


tential immediate post extraction complications for
a surgical patient treated under GA?
A. Respiratory embarrassment
B. Circulator depression
C. Aspiratory of blood or gastric contents
D. All of the above
247. The fate of nitrous oxide when administered for
analgesia is:
A. Detoxified in liver
B. Exhaled by Iungs
C. Excreted by kidney
D. Chemically changed into NO and C02
248. Of the following in which nitrous oxide-oxygen se-
dation is contraindicated?
A. Mental retardation
B. Sickle cell anemia
C. History of congestive heart failure 3 months
previously
D. None of the above
249. The sensation of "tissue tearing" during inferior
alveolar nerve block indicates:
A. Passage through buccinator muscle
B. Passage through an area of infection
C. Barb on the needle
D. Large bore needle such as 19 no. needle

246 D 247 B 248 0 249 c


68 MCQs in Oral Surgery

250. Of the following which is main cause of toxic


manifestations following injection of a vasocon-
strictor is?
A. Intramuscular injection
B. Intravascular injection
C. Injection into a nerve trunk
D. Allergic reaction
251. Of the following which is most common postopera-
tive complication of out patient of out patient gen-
eral anaesthesia?
A. Pneumonia B. Lung abscess
C. Nausea D. Atelectasis
252. In a dental clinic after the administration of local
anaesthesia depression of respiration occurred. It
is a manifestation of:
A. Trauma to a nerve trunk by the syringe needle
B. Toxic effects of the solution
C. Puncture of a blood vessel
D . Use of an isotonic solution
253. Among the following which is most common com-
plication associated with nitrous oxide sedation?
A. Vomitus aspiration
B. Behavioral problem
C. Respiratory depression
D. Vomiting
254. Among the following which is advantage of oral
adminstration of drugs to achieve conscious seda-
tion?
A. It is highly reliable in producing sedation
B. It is easiest to administer
C. Over dosage is ':lnlikely
D. None of the above

250 B 251 c 252 B 253 8 254 8


LAIGA/Pain Control 69

255. Which of the following is immediate post operative


complication of aspiration of liquid vomitus into the
trachea and bronchi?
A. Atelectasis
B. Pneumothorax
C. Chemical pneumonia
D. Pleuricy
256. The following factors play a role and cause varia-
tions in susceptibility of sensory nerve fibers to
local anaesthesia:
A. Diameter of the nerve fiber
B. Distance between nodes or Ranvier
C. Length of nerve fiber
D. All of the above
257. Which of the following are contraindications to
nitrous oxide oxygen sedation?
A. Emotional instability
B. Upper respiratory tract obstruction
C. Emphysema
D. All of the above
258. Which of the following would be indicated if one
expected a long surgical procedure and a general
anaesthetic to be administrated?
A. Sodiurri thiopental
B. Vinyl ether, open drop method
C. Intubation with individually selected anaesthetic
agent
D. None of the above

255 c 256 D 257 D 258 c


70 MCQs in Oral Surgery

259. Intraoral Injection to block second division of the


trigeminal nerve should be given In the:
A. Foramen spinosum
B. Foramen ovale
C. Pterygopalatine fossa
D. Infraorbital foramen
260. Which is the main route of elimination of nitrous
oxide?
A. Kidneys B. Liver
C. Lungs .D. Skin
261. Frequently which of the following In amide type lo·
cal anaesthesia causes allergic reaction?
A. Lignocaine hydrochloride
B. Methyl paraben
C. Adrenaline
D. Contaminants in the· solution
262. Usually hydrolysis of amide type of local anaesthetic
salts is facilitated by:
A. Tissue pH above 7.0
B. Tissue pH between 5.0 and 7.0
C. Tissue pH below 5.0
D. There is no relation ship between pH and hydrolysis
in amide type local anaesthesia
263. Respiration In stage IV general anaesthesia is
characterised by:
A. Thoracoabdominal in nature
B. Abdominothoracic in nature
C. Deep and irregular
D. Absent

259 c 260 c 261 B 262 A 263 D


LAIGA/Pain Control 71

264. A patient is allergic to PABA. Most probably he will


also be allergic to:
A. Lignocaine B. Carbocaine
C. Prilocaine ·D. Tetracaine
265. Nitrous oxide cylinders are usually in colour:
A. Black B. White
C. Blue D. Red
266. Which of the following should be included in the
preoperative orders?
A. NPO from mid night
B. Take written informed consent
C. Void on call to the OR
D. All of the above
267. In which stage of anaesthesia patient becomes
hyperactive?
A. Stage I B. Stage II
C. Stage III D. Stage IV
268. Toxic dose of lignocaine with adrenaline is:
A. 5 mg/ kg body weight
B. 10 mg/kg body weight
C. 15 mg/kg body .weight
D. 20 mg/kg body weight
269. In gate control therapy of pain. Pain is mainly modu-
lated by:
A. Substantia gelatinosa
B. 'B' fibers
C. 'C' fibers
D. Transmission system

~-z_M__o____z_s_s_c____2_ss__o____2_s7__s ___2_s_s_A____2s_s_A__~I .
72 MCQs in Oral Surgery

270. Morphine scopalamine premedication produces:


A. Amnesia and decreased salivation
B. Psychic sedation
C. Additive effects with anaesthetics
D. All of the above
271. The following which produces an unconscious
state?
A. Regional analgesia
B. Nitrous oxide and oxygen
C. Neuroleptic analgesia
D. None of the above
272. Maximum recommended dosage of lignocaine with
1:1,00,000 adrenaline subcutaneously is:
A. 100 mg B. 30 mg
C. 500 mg D. 700 mg
273. Among the following which stage of anaesthesia
describes the level of conscious sedation?
A. Stage I
B. Stage II
C. Stage III plane 1
D. Stage Ill plane 2
274. The initial stage of paralysis of facial nerve:
A. Tongue deviate to the same side on protrusion
B. Tongue deviate to the opposite side on protrusion
C. No deviation of the tongue
D. None of the above
275. "Tic douloureux" treatment includes:
A. Dilantin sodium in all cases
B. Car~amezapine in all cases
C. Mandibular nerve block in all cases
D. Trichloroethylene in all cases

270 0 271 c 272 c 273 A 274 A 275 B


LA/GA!Pain Control 73

276. Risus sardonlcus is one the sign of the tetanus.


This involves:
A. Spasm of the glottis causing altered voice
B. Spasm of the facial muscle causing a fixed smile
C. Spasm of the pterygoid muscles causing trismus
D. Spasm of the chest wall causing difficulty in breathing
277. Which of the following structure is associated with
"Bell's palsy"?
A. Submandibular gland
B. Seventh cranial nerve
C. Temporomandibular joint
D. Glossopharyngeal nerve
278. A decrease in the pH of interstitial fluid will:
A. Increase the effectiveness of a local anaesthetic block
B. Have no effect on the effectiveness of a local
anaesthetic block
C. .Decrease the effectiveness of the LA solution
D. Initially increases than decreases the effectiveness
of LA solution
279. Maximum recommended dosage of lignocaine and
1:1,00,000 adrenaline subcutaneously is:
A. 100 mg
B. 300 mg
C. 500 mg
D . 700mg
280. The following clinical disease process affect the
TMJ directly:
A. Ankylosis
B. Arthritis
C. Dislocation
D. All of the above

276 B 277 B 278 c 279 c 280 D


74 MCQs in Oral Surgery

281. Best treatment for a small opening of a disease free


maxillary sinus is:
A. Not treating socket but advising the patient proper
home care
B. Packing tile socket with a hemostatic agent to
encourage clotting
C. Leaving the socket undisturbed but prescribing nasal
vasocontrictor and antibiotics
D. Employ primary closure of the socket using sutures.
282. Among the following which factors influence the
pain perception threshold?
A. Emotional state.
B. Apprehension and fear
C. Age
D. All of the above
283. The drug of choice to counteract the central ner-
vous system stimulation caused by accidental IV
injection of local anesthesia:
A. Adrenaline
B. Pentobarbital
C. Caffeine
D. None of the above
284. Under GA, hemolysis resulting from a transfusion
reaction will exhibit which of the following symp-
toms?
A. Chills, fever, dyspnoea, headache, pain in the back
B. Hypotension, skin flush, tachycardia, abnormal
hemorrhage, hyperpyrexia
C. Urticaria, angioneurotic edema, asthma
D . Utricaria and pruritus

281 A 282 D 283 B 284 B


LA/GA/Pain Control 75

285. Following group of drugs used for premedication


produce a detached serenity without clouding con-
sciousness. Can you identify them?
A. Antiemetics
B. Ataractics
C. Analeptics
D. Psychodissociates
286. Among the following which results in sudden swell-
ing in the area of a dental injection?
A. Drug incompatibility.
B. Paravascular injection
C. Infection
D. Vascular penetration
287. Among the following which has least hypnotic ac-
tion?
A. Chloral hydrate
B. Pentobarbital
C. Phenobarbital
D. Codeine
288. Which of the following is not a transmitter substance
and it has no role .i n impulse transmission?
A. Dopamine
B. Noradrenaline
C. Succinyl choline
D. GABA
289. Preanesthetic medication commonly used to
achieve the following goals except:
A. Decreasing nervous irritability
B. Decreasing secretions
C. Providing analgesia
D. Desensitizing tile respiratory centre to C02

285 8 286 D 287 D 288 c 289 D


76 MCQs in Oral Surgery

290. When is the appropriate time to administer an anal-


gesic to control postoperative pain?
A. Only after the return of sensation
B. When pain becomes moderate to severe
C. Before the anesthetic effect wears off
D. None of the above
291. Sedation by which of the following routes can be
reversed most rapidly?
A. Inhalation B. Intravenous
C. Intramuscular D. Oral
292. Among the following which drug is drug of choice
in management of the acute allergic reaction involv-
ing hypotension?
A. Aminophylline
B. Dexamethasone
C. Diphenydramine
D. Adrenaline
293. One cannot determine the level of N2 0 per tank with
N2 0 pressure gauge on a sedation machine
because:
A. The gas is explosive
B. The pressure drops in a sporadic fashion
C. The pressure remains constant until the tank
D. It is nearly empty
294. Toxic effects seen after administration of a local
anesthetic with epinephrine are probably due to
anesthetic agent if the following sign is present:
A. Tachypnoea
B. Tachycardia
C. Drowsiness
D. Sweating

290 c 291 A 292 0 293 c 294 c


LA/GA/Pain Control 77

295. While receiving IV diazepam, patient has upper eye-


lid ptosis (verrill's sign) the dentist should immedi-
ately (
A. Assist respiration
B. Consider the patient is adequately sedated
C. Place the patient in Trcndelenberg position.
D . Consider the anesthesia is not complete and
administer one more dose of diazepam
296. Among the following which way best protect from
the toxic aspects of a local anesthetic?
A. Take a thorough medical history
B. Have oxygen available
C. Use an aspirating technique
D. Use the lowest possible concentration of the local
anesthetics ·
297. Trigeminal neuralgia is characterized by:
A. Paralysis of the one side of the face due to paralysis
of facia) nerve ·
B. Dull pain when pressure is applied over the affected
area (Trigger zones)
C. Sharp pain when pressure is applied to the affected
area
D. Prolonged episodes of the pain w hich is constant in
nature

295 B 296 c 297 c


STERILIZATION AND
3 INSTRUMENTS

1. Scrub technique refers a method of scrubbing the:


A. Surgical instruments after a contaminated case
B. Patient body surface before placing surgical incision
to decrease bacterial flora.
C. Operating room.
D. Opera tor's hands and forearms before donning gown
and gloves.
2. The sterilising units should be monitored at which
of the following intervals?
A. Daily
B. Weekly
C. Biweekly
D. Monthly
3. Scrub technique refers to the proper method of:
A. Washing hands and arms before gloving for surgery
B. Cleaning the operating room after a contaminated case
C. Washing the patient's body surface prior to incision
D. Washing the instruments prior to sterilization
4. Bi-beveled chisels are used primarily to:
A. Remove bone
B. Split teeth
·C. Make purchase points
D. All of the above

1 D 2 B 3 A 4 B
Sterilization and Instruments 79

5. Chromic catgut sutures are packed in alcohol to


prevent:
A. Atmospheric contamination
B. Enzymatic degradation
C. Loss of tensile strength
D. All of the above
6. Catgut sutures are packed in this type of alcohol:
A. Ethyl alcohol
B. Methyl alcohol
C. Isopropyl alcohol
D. Absolute alcohol
7. The typical "railroad track" scar is the result of:
A. Inadequate approximation of wound edge during
suturing
B. Delayed removal of sutures
C. Delayed wound closure
D. Excessive fibrous tissue formation
8. The "Wicking" effect will be more commonly seen
when the following suture material is used:
A. Nylon
B. Plain catgut
C. Silk
D. Polypropylene
9. Steam autoclaving:
A. Kills microorganisms by RNA and DNA breakdown
B. Destroys cotton or cloth products
C. Is non corrosive to carbon steel instruments
D. Utilises 249.8°F at 5 psi and 15 to 30 minutes

5 B 6 c 7 c 8 c 9 A
80 MCQs in Oral Surgery

10. Glutaraldehyde is activated to achieve its full anti-


microbial capability by adding:
A. Sodium sulphate
B. Sodium hypochlorite
C. Sodium bicarbonate
D. Sodium hydroxide
11. A vertical mattress suture is used in wound closure:
A. To prevent ischemic necrosis of wound edges in areas
with poor blood supply
B. To close flaps that are one tissue layer thick
C. To evert the wound margins
D. To provide a water tight closure
12. Single use hypodermic needles have coloured hubs
for ease of indentification of the needle gauge. 25'G
needle hubs are:
A. Blue
B. Red
C. Yellow
D. Green
13. To reduce thrombophlebitis in patients on parentral
fluid therapy, IV catheter made up of the following ma-
terials are used:
A. Silicone
B. Stainless steel
C. Teflon
D. Plastic
14. The main objective for efficacy of sterilization prac-
tice is destruction of:
A. Cocci B. Viruses
C. Bacteria D. Spores

10 c 11 A 12 A 13 c 14 0
Sterilization and Instruments 81

15. Most postoperative infection results from :


A. Faulty surgical technique
B. Inadequate asepsis and disinfection
C. None of the above
D. Both of the above
16. Is a chemical, applied to living tissues,
such as skin or muc·o us membrane to reduce the num-
ber of microorganisms present , by inhibition of their
activity or by destruction.
A. Antiseptic
B. Disinfectant
C. Antiseptic and disinfectant
D. Cleaning agent
17. is the process of destruction or removal
of all microbial forms:
A. Antisepsis
B. Sterilization
C. Disinfection
D. Cleaning
18. The mechanism by which the disinfectant and anti-
sepsis act on microorganisms?
A. Coagulation of bacterial protein
B. Alteration in properties of bacterial w all
C. Binding of sulphydryl groups
D. All of the above
19. Alcohol shows maximum antiseptic activity at:
A. 10% B. 50%
c. 70% D. 99%

15 D 16 A 17 B 18 D 19 c
82 MCQs in Oral Surgery

20. In autoclave at 15 psi pressure and 121 degree C


temperature time required to kill all living organ-
isms is:
A. 3'min B. 6min
C. 15min D. 60 min
21. Organisms used for test of efficiency for heat steril-
ization
A. S sanguis
B. T pallidum
C. Beta hemolytic streptococci
D. Bacillus sterothermophyllus
22. Basic action of Dry Heat sterilization is
A. Dehydration B. Oxidation
C. Reduction D. A and B
23. Which of the following relation of temperature and
time is for dry heat sterilization?
A. 60°C-2 hrs
B. 100°C-11/2 hrs
C. 170°C-1 hr
D. 200°C-15 mins
24. Glass bead sterilizer is used for sterilization of:
A. Glass instruments
B. Plastic instruments
C. Endodontic files and burs
D. Long metal instruments
25. The operation theatres are disinfected by:
A. Alcohol
B. Heat sterilization
C. Fumigation
D. None of the above

20 c 21 D 22 D 23 c 24 c 25 c
Sterilization and Instruments 83

26. Excretions and secretions should be disposed by:


A. Buried in deep pit"
B. Powder and lime
C. Inceneration
D. Deep burial
27. Laboratory specimen should be kept in:
A. 10% formalin
B. 10% phenol
C. 100% alcohol
D. 0.2% Chlorhexidine
28. Plaque from gingival crevice contains around how
many bacteria:
A. 1000
B. 2500
C. 2.5 X lQII
D. 2.5 X 1017
29. At birth oral cavity contains:
A. S. sanguis
B. S. hemolytiws
C. M. tuberculae
D. Oral cavity is sterile
30. Most commonly detected organisms in mouth of new
born is:
A. S. sanguis B. T. pallidum
C. S. salivarius D. C. albicans
31. Shape of treponema is:
A. Spiral ·B. Cocci
C. Bacilli D. Dumb-bell

26 B 27 A 26 c 29 0 30 c 31 A
84 MCQs in Oral Surgery

32. Candida albicans is:


A. Virus
B. Bacteria
c. Fungi
D. Yeast
33. A hot air oven cannot be used for sterilization of:
A. Glassware
B. Talcum powder
C. Cotton
D. Culture media
34. A gas compound used in special autoclaves for ster-
ilizing materials that cannot be taken to higher tem-
peratures is:
A. Water vapour
B. Ethylene oxide
C. Hydrogen
D. Cyanide
35. Criteria that have been used to distinguish pathogenic
from non-pathogenic bacteria are known as:
A. Gram stains
B. Koch's postulates
C. Randolph's rules
D. Pasteur's principles

32 0 33 D 34 B 35 B
EXODONTIA/
IMPACTION

1. A patient develops facial edema one day after removal


of a tooth. Instructions to the patient should be to use:
A. Warm, wet applications outside and inside the mouth
B. Cold applications outside and inside the mouth
C. Intraoral cold applications only
D. Intraoral hot applications only
2. Of the following the most common postoperative com-
plication after surgical removal of mandibular teeth
is:
A. Myositis
B. Paresthesia
C. Loss of the blood clt
D. Postoperative hemorrhage
3. The most serious complication which may occur fol-
lowing surgery about the maxillary canine is:
A. Iritis
B. Cellulitis
C. Localization of pus
D. Cavernous sinus thrombosis
4. The elevators used in exodontia are functionally:
A. Class I levers only
B. Class I and II levers
C. Class III levers only
D. Only wedge-shaped

1 A 2 c 3 D 4 B
86 MCQs in Oral Surgery

5. To extract a tooth the following type of grip Is most


desirable:
A. The whole of the inner surface of the forceps blade
should fit the root surface
B. Only edges of the forceps blade should contact the
tooth (Two point contact)
C. Only single lever contact between forceps blade and
root surface (one point contact)
D. Grip plays little role in extraction
6. Exodontia elevators are based on principle of:
A. Levers B. Wedge
C. Wheel and axle D. All of the above

7. While removing mandibular 3rd molar which part of


bone should be used as a fulcrum?
A. Lingual cortical bone
B. Mesial inter-radicular bone
C. Buccal alveolar bone
D. Distal surface of adjacent crown
8. Mechanical advantage would be maximum for an el-
evator when:
A. Effort arm is greater than resistance arm
B. Resistance arm is greater than effort arm
C. When fulcrum is in the centre
D. When fulcrum is near the point of effort
9. When maxillary lateral incisor has to be extracted the
first movement of forcep should be:
A. Labially then apically
B. Palatally then apically
C. Labially or palatally
D. Apically then palatally

5 A 6 D 7 c SA 9 D
Exodontia/Impaction 87

10. Mead forceps are used for extraction of:


A. Maxillary molars B. Mandibular incisors
C. Maxillary canines D. Maxillary premolars
11. To extract mandibular premolars the first movement
of forceps should be:
A. Apical and labial
B. Apical and lingual
C. Apical and mesial
D. Labial and no apical movement
12. When maxillary and mandibular teeth are to be ex-
tracted simultaneously the order of extraction should
be:
A. Maxillary teeth followed by mandibular teeth
B. Mandibular teeth followed by maxillary teeth
C. It does not matter which teeth are removed first
D. For anterior region mandibular first and for posterior
region maxillary first
13. When simultaneous extractions have to be carried out
in one maxillary segment the order of extraction
should be:
A. Incisors, canine, premolars, molars
B. 3rd molar, second molar, 2nd premolar, 1st molar,
1st premolar, incisors, canine
C. Molars, premolars, canine, incisors
D. Molars, premolars, incisors, canine.
14. The Stobis method of extraction is indicated when:
A. Two adjacent teeth have to be removed
B. When central incisor is to be extracted
C. When isolated molar is to be extracted and adjacent
teeth are absent
D. Upper and lower 3rd molars have to be extracted
simultaneously

10 B 11 B 12 A 13 B 14 A
88 MCQs in Oral Surgery

15. 'White line' described by George Winter is a line


drawn:
A. Along occlusal surfaces of erupted mandibular molars
extending to impacted third molar region
B. From crest of bone lying distal to third molar to crest
of the interdental septum between 1st and 2nd molar
C. From perpendicular to 'amber line'
D. From perpendicular to 'red line'
16. 'Red Line' as described by George Winter Is demar-
cated as a line:
A. Drawn from bone distal to third molar to crest of the
interdental septum between 1st and 2nd molar
B. Which is a perpendicular dropped from 'amber line'
to point of application of elevator
C. Which is a perpendicular dropped from 'white line'
to point of application of elevator
D. None of the above
17. If in a mesioangular mandibular impaction there is
apical notch visible on a radiograph the tooth
should:
A. Not be removed
B. Be removed by lingual split .technique
C. Be removed by splitting the tooth
D. None of the above
18. Which type of mandibular impactions are most com-
monly associated with 'apical notch' formation:
A. Mesioangular
B. Distoangular
C. Vertical
D. A and B

15 A 16 B 17 c 18 D
Exodontia/Impaction 89

19. The disadvantage of lingual split technique is/are:


A. Injury to lingual nerve
B. Opening up of facial spaces on lingual side and floor
of mouth
C. Chances of dislodging tooth or root in sub- lingual
space
D. All of the above
20. The basic advantage of lingual split technique for
extraction of mandibular impacted teeth is/are:
A. Bone loss is minimal
B. Easy and quick method
C. Tissue trauma is minimal
D. All of the above
21 . Lateral trepanation technique of Bowdler Henry is
indicated for:
A. Extraction of impacted canines
B. Removal of impacted premolars
C. Removal of partially formed unerupted third molars
D. Treating dentigerous cysts with enclosed third molars
22. The chisel should be used with :
A. Bevel towards the bone which is to be saved
B. Bevel towards the bone which is to be sacrificed
C. Bevel d irection is not important
D. Flat surface parallel to direction of grains of bone
23. While making vertical incision for flap for mandibular
third molar impactions one can injure:
A. Buccal pad of fat
B. Branches of lingual nerve
C. Branches of facial nerve
D. Branches of facial artery /vein

19 D 20 D 21 c 22 8 23 D
90 MCQs in Oral Surgery

24. Dry socket commonly occurs after:


A. 24 hours
B. 2 days
C. 3-4 days
D. 10-15 days
25. The treatment of localized osteitis is:
A. Debridement, curettage and sedative packing
B. Curettage, irrigation and sedative packing
C. Irrigation and sedative packing
D. None of the above
26. In case of extraction of maxillary molars if maxillary
tuberosity also fractures the fractured bone:
A. Should be removed
B. Should be replaced and allowed to heal by secondary
intention
C. Should be replaced and retained by primary suturing
of soft tissues
D. Should be fixed by transosseous wiring or bone plating
27. The complication of using air rotor at 30,000 RPM
for impacted molars is:
A. Necrosis of cone
B. Dehiscence
C. Tissue laceration
D. Emphysema
28. The most important suture while closing Ward's inci-
sion for impacted mandibular third molar is:
A. Suturing of vertical limb
B. Suturing of retromolar limb
C. Suture of area immediately distal to 2nd molar
D. None of the above

24 c 25 c 26 c 27 D 28 c
Exodontia/Impaction 91

29. Dull ache in the jaw and ear three days after a den-
tal extraction may indicate:
A. Pulpitis of an adjacent tooth
B. Need for a stronger prescription for discomfort
C. Need for an antibiotic
D. Loss of the alveolar clot
30. The scalpel blade most commonly used for oral sur-
gical procedures is:
A. No. 22 B. No. 12
C. No. 11 D. No. 15
31. The "red line" in Winter's classification of impacted
teeth represents:
A. The depth at which the impacted tooth is located
B. The angulation of the 2nd molar
C. The summit of the alveolar bone covering the
impacted tooth
D. Relation of the 3rd molar to the ramus
32. The most serious complication of an abscess of the
maxillary anterior teeth is:
A. Pain
B. Paraesthesia
C. Cavernous sinus thrombosis
D. Orbital cellulitis
33. The beaks of an extraction forceps should be placed:
A. On the anatomical crown
B. At the junction of the clinical and anatomic crowns
C. On the root surface as far apically as possible
D. At the junction of the apical and middle thirds of the
root

29 D 30 D 31 A 32 c 33 c
92 MCQs in Oral Surgery

34. A mucoperiosteal flap can be repositioned accu-


rately because the:
A. Periosteum is inelastic
B. Periosteum is very elastic
C. Periosteum contains sharpey's fibers
D. All of the above

35. The root most likely to be pushed into the maxillary


sinus during a tooth extraction is:
A. Palatal root of the maxillary second molar
B. Palatal root of the maxillary first premolar
C. Palatal root of the maxillary first molar
D. Mesiobuccal root of the maxillary first molar
36. A 55-year-old lady has a lone standing carious maxil-
lary first molar tooth on the right side. The tooth has
served as an abutment, and now has to be extracted.
Proper care is to be taken during extraction to
prevent:
A. Alveolar osteitis
B. Pushing the palatal root in the maxillary sinus
C. Removing the floor of the maxillary sinus with the
tooth
D. Displacement into the infratemporal fossa
37. Which of the following muscles is pierced by the
needle while giving an inferior alveolar nerve block?
A. Medial pterygoid
B. Superior constrictor
C. Temporalis
D. Buccinator

34 A 35 c 36 c 37 D
Exodontia/Impaction 93

38. Primary healing of a mandibular fracture is seen fol-


lowing fixation with:
A. Gunning splints
B. Compression plates
C. Trans-osseous w ires
D. Champy plates
39. In the maxilla, a compression bone plate can be safely
applied along the:
A. Infraorbital margin
B. Anterolateral wall of the maxillary sinus
C. Pronto-zygomatic suture
D. Zygomaticomaxillary suture
40. To provide 'absolute stability' of the fractured ends
by a compression bone plate, the minimum number
of screws that have to be placed on both side of the
fracture line is:
A. Six .B. Two
C. Three D. Four
41. A potential complication following tooth extraction
from areas, having undergone irradiation is:
A. Excessive bleeding
B. Alveolar osteitis
C. Osteoradionecrosis
D. Fracture of the underlying bone
42. A straight elevator is property used to advantage when
the?
A. Buccal cortical plate is used as a fulcrum
B. Adjacent tooth is not to be extracted
C. Interdental bone is used as a fulcrum
0. When the bone surrounding the tooth to be extracted
is sclerosed

38 B 39 A 40 D 41 c 42 c
94 MCQs in Oral Surgery

43. The ideal order of extraction of teeth is:


A. First ~olar, canine, lateral incisor, second molar, first
premolar and third molar
B. Third molar, second molar, second premolar, first
molar, first premolar, lateral incisor and canine
C. La teral incisor, canine, first premo lar, second
premolar, first molar, second molar and third molar
D. Canine, first molar, third molar, second molar, 2nd
.. premolar, 1st premolar and lateral incisor
44. The 'cow-horn' (no. 16) forceps should not be used
for the extraction of mandibular deciduous molars as
the:
A. Sharp beaks can damage the unerupted premolar
teeth
B. Beaks cannot engage the furcation area properly
C. Forcep is not intended for extraction of deciduous
molars
D. None of the above
45. Prophylactic removal of unerupted third molars in a
teenager should ideally be carried out:
A. When crowding/ rotationoccurs in the anterior teeth
B. When the root is one-third formed
C. When the root is two-thirds formed
D. Before the root formation
46. The most common contributing factor to pericoroni-
tis of an impacted mandibular third molar is:
A. Bacterial infection
B. Trauma from the opposing maxillary third molar
C. Trauma from the eruptive forces exerted by the tooth
D. Food impaction

43 B 44A 45 c 46 B
Exodontia/Impaction 95

47. A palatal flap for the removal a maxillary impacted


canine near the midline should be:
A. Reflected around the necks of the teeth
B. Started with vertical incisions in the opposite canine
region
C. Started with a vertical incision in the midline
D. Semilunar in design
48. Epinephrine packs are to be avoided in controlling
bleeding from extraction sockets as:
A. It interferes with the coagulation process
B. It causes vasoconstriction with resultant alveolar
osteitis
C. It causes reflex vasod ilatation with severe pos t-
operative bleeding
D. It can be lead to a hypertensive crisis
49. In the open beak technique of extraction, the tip of
the forceps is placed at:
A. The cementoenamel junction
B. 2/3rd on the root surface
C. Marginal alveolar bone
9. Two nun apical to the DE junction
50. While removing an impacted mandibular third mo-
lar buccal leverage is applied when?
A. Tooth is inclined mesially
B. Tooth tapers towards the root
C. Tooth is very close to the 2nd molar
D. Space exists between the neck of the tooth and the
second molar

47 A 48 c 49 c 50 c
96 MCQs in Oral Surgery

51 . The extraction of an impacted upper third molar


with SA is best carried out using:
A. A straight elevator
B. Miller's elevator
C. Cryer's elevators
D. Apexo elevators
52. Blood loss during full mouth extractions of teeth and
alveoplasties, can be expected to be in the range of:
A. 50-200 ml B. 200-800 ml
C. 800-1000 ml D. 1000-1200 ml
53. Facial oedema following surgical extracion of an im-
pacted mandibular third molar, can be best reduced
by:
A. Postoperatoive administration of dexamethasone
B. Preoperative administration of d iclopfenace sodium
C. Careful retraction and manipulation of the soft tissue
flap during surgery
D. Regular use warm saline mouthrinses after surgery
54. Generally, the treatment of choice for cementoma is:
A. Extraction of all teeth
B. Endodontic treatment of involved teeth
C. No treatment
D. Block resection of involved area
55. The healing process that takes place following the non
surgical extraction of a mandibular 1st molar may be
described as:
A. Healing by first intention
B. Healing by second intention
C. Granulomatosis
D. A and C

51 8 52 8 53 c 54 c 55 B
Exodontia/Impaction 97

56. Best treatment for pericoronitis associated with im-


pacted mandibular third molar is:
A. Irrigating under the operculum
B. Antibiotic and analgesic therapy
C. Extraction of impacted third molar
D. Operculectomy
57. A palatal flap is required for removal of a maxillary
impacted canine near the midline. Palatal flap should
be:
A. Started with a vertical incision in the midline
B. Reflected from the necks of the teeth
C. Made with a semilunar incision
D. None of the above palatal flap should be avoided
58. During the tooth extraction a known cardiac patient
experienced angina. This was most likely precipitated
by:
A. Allergy to the anesthetic agent
B. The stimulus of pain and anticipation
C. Adrenaline in the drug
D. The upright position in the chair ·
59. Three days after the extraction of maxillary first mo-
lar patient developed a fluctuant palatal abscess. The
treatment of choice is:
A. Caldwell-Luc procedure
.... B. Heat and cold therapy
C. I and D and antibiotic therapy
D. Only antibiotic therapy after culture and sensitivity
tests

56 c 57 B 58 B 59 c
98 MCQs in Oral Surgery

60. Early sign of syncope is:


A. Epistaxis B. Pupillary constriction
C. Pallor D. Rapid pulse and increased BP
61. Among the following, the factor of least importance
in alveolar flap design is:
A. Linear incision
B. Closure over the bone
C. Tension free closure of the flaps
D. Base relationship to length
62. The following are indication for suturing following
exodontia except:
A. Blood clot protection
B. Reapproximation of mucoperiosteal flaps
C. Immobilisation of tissues
D. Hemostasis
63. In the extraction of mandibular third molars, the main
reason why the posterior incision should be placed
more buccally is:
A. To prevent damage to lingual nerve
B. Incision should be on the sound bone
C. To prevent damage ~o retromolar artery
D. All of the above
64. In multiple extractions, the order of extraction
usually:
A. Maxillary teeth before mandibular
B. Mandibular teeth before maxillary
C. Posterior teeth before anterior
D. Anterior teeth before posterior
1. 1 and 3 2. 2 and 4
3. 2 and 3 4. 1 and 4

60 c 61 A 62 A 63 D 64A
Exodontia/Impaction 99

65. Limited accessibility is one of the most disadvan-


tage of the following flap:
A. Envelop flap
B. Semilunar flap
C. Three cornered flap
D. Four cornered flap
66. According to Nitzin's theory, the following microor-
ganisms are responsible for dry socket:
A. Treponema denticola
B. Leprospira pyrogen
C. Pseudomonas
D. Bacillus subtilus
67. Most common site for dry socket:
A. Lower incisor area
B. Upper incisor area
C. Lower molar area
D. Upper molar area
68. Few days after a dental extraction patient came with
marked pyrexia and complaining of impairment of la-
bial sensation and marked tenderness on extra oral
palpation. Most probable diagnosis is:
A. Dry socket
..... B. Acute osteomyelitis
C. Postoperative haematoma
D. None of the above
69. Advantage of chisel and mallet over bur in removing
bone is:
A. Less skill is needed
B. Psychologically more acceptable
C. No coolant is necessary
D. Chisel and mallet are good instruments to remove
dense bone

65 B 66 A 67 c 68 D 69 c
100 MCQs in Oral Surgery

70. The following design parameters should follow to


prepare a mucoperiosteal flap except:
A. Flap sh ould provide adequate visibility and
accessibility
B. Free margin broader than base
C. Ragged margins should be avoided
D. Flap should be a full thickness mucoperiosteal flap
71. Most common cause of post-extraction bleeding is:
A. Bleeding disorders
B. Failure of the patient to follow post extraction
instructions
C. Due to the analgesics such as aspirin
D. Patient with the anticoagulant therapy
72. A deep level of general anaesthesia is enhanced by a:
A. Muscle relaxant liked-tubocurarine
B. Non-irritating drug
C. Loose fitting mask
D. High alveolar concentration of anaesthetic agent
73. "Postage stamp" method is:
A. A method of bone removal in transalveolarextraction
B. A method of extraction of maxillary canines by intra-
alveolar method extraction
C. A method of bone grafting
D. None of the above
7 4. "Line of withdrawal" of a tooth Is mainly determined
by:
A. The size of the crown
B. The root pattern
C. Shape of the crown
D. Size of the roots

70 B 71 B 72 D 73 A 74 B
Exodontia/ Impaction 101

75. "Odontectomy" is synonymous to:


A. Transalveolar extraction
B. Intra-alveolar extraction
C. Extraction under local anaesthesia
D. Extraction under general anaesthesia
76. The best example of an elevator which works on wheel
and axle principle?
A. Howartli's periosteal eleva tor
B. Winter cross bar elevator
C. Millers apexoelevator
D. None of the above
77. Impacted mandibular third molar classification is usu-
ally based on :
A. Relation of the tooth to the ramus of the mandible
B. The position of the tooth in relation to the long axis of
the second molar
C. The relative depth of the third molar in bone
D. All the above
78 . Ten teeth have been removed for a patient who was
"j" premedicated . The proper position of the pa-
tient in Pfy the recovery room is:
A. Head elevation 30° w ith patient on his back
B. Trendelenburg's position patient head is almost
parallel to the floor
C. Reverse Trendelenburg's position
D. Supine position
79. A swelling over lies an extraction wound and it crack-
les on palpation. Most likely diagnosis is:
A. Ecchymosis B. Cellulitis
C. Emphysema D. Empyema

75 A 76 B 77 D 78 A 79 c

-~--
102 MCQs in Oral Surgery

80. During reflecting a palatal mucoperiosteal flap for


removal of an impacted canine structures passing
through the incisive canal are averted. The sequela
to this is:
A. Of no clinical significance
B. Loss of a triangular mucosal flap behind the incisors
and canine due to loss of blood s upply
C. Resorbption of bone around the canal
D. Anaesthesia of the anterior palate
81. Which of the following is the principle action of am-
monia in syncope?
A. Respiratory stimulant
B. Vagal stimulant
C. Vasomotor stimulant
D. Inhibitor of vasomotor tone
82. After the surgical removal of chronically infected teeth
in a controlled diabetic patient, which of the follow-
ing is of utmost importance?
A. Postoperative check of patient for bleeding
B. Return of patient to diet of a normal individual
C. Revaluation of insulin dosage being taken by the
patient
D. Medication for marked post operative pain and
discomfort
83. The following nerves should be anesthetized for the
removal of maxillary first molar:
A. Anterior palatine
B. Posterior palatine
C. Anterior superior alveolar
D. Middle superior alveolar
E. Posterior superior alveolar
1. 1,3,5 2. 1,4,5
3. 2, 3,5 4. 2, 4, 5

80 A 81 A 82 c 83 B
Exodontia/ Impaction 105

92. Recontouring of the residual ridge in the early part


of extraction of period occurs primarily by resorp-
tion of the:
A. Buccal-lingual cortical bone
B. Labial-buccal cortical bone
C. Lingual cortical bone
D. Crestal bone
93. Three days after extraction of teeth for an immediate
denture, patient complains of a diffuse, non-pain-
ful yellow, submandibular and supra sternal discol-
oration of the skin. The dentist should:
A. Advise antibiotics
B. Apply heat and advise analgesics
C. Apply cold compressions
D. None of the above
94. Of the following which tooth is most difficult to anes-
thetize by infiltration technique alone: ·
A. Maxillary canine
B. Maxillary central incisors
C. Maxillary first molar
D. Maxillary first premolar
95. To luxate a tooth with a forceps the movements should
be:
A. Finn and deliberate primarily to the facial surface with
secondary movements to the lingual surface
B. Sharp and d efinite so periodontal ligament tears easily
C. Gentle w iggles so patient get used to the pressure
D. Figure of "eight" motion

92 B 93 0 94 c 95 A
104 MCQs in Oral Surgery

88. A 75-years-old woman whose weight is approxi-


mately 100 pounds requires multiple extractions
under local anesthesia. For premedication, the fol-
lowing dose of diazepam is enough to allay the ap-
prehensi on?
A. 20 mg B. 30 mg
C. 10 mg D. Smg
89. The beak of an extraction forceps is designed such a
way that the extraction pressure is transmitted to the:
A. Crown of th e tooth
B. Alveolar bone
C. Root of the tooth
D. Cementoenamel junction of the tooth
90. During the extraction of mandibular third molar it is
noted that the distal root is missing. The root tip is
most probably in the:
A. Submen tal space
B. Submandibular space
C. Pterygomandibular space
D. Parapharyngeal space
91. A 45-year-old women insists on extraction of pain-
ful man-dibular first molar. She has not rested for 2
days and nights because of pain. Her medical his-
tory reveals she took 30 mg of prednisone daily for
one year but stopped taking the medication three
months age. The dentist should:
A. Relieve the symptoms by g iving a ntibiotics and
analgesics
B. Give steroid s upplements and extract the tooth under
local anesthesia
C. Avoid procaine anesthetic
D. Extract the tooth under loca l anesthesia in a usual
manner

88 B 89 c 90 B 91 B
Exodontia/Impaction 107

100. Among the following which is most common compli-


cation after surgical removal of a mandibular tooth?
A. Post extraction hemorrhage
B. Paresthesia of lingual nerve
C. Loss of blood clot
D. Difficulty in swallowing
101. A severe dull ache in the jaw three or four days after
an extraction of tooth usually indicates:
A. Exposure of cementum of an adjacent tooth
B. Acute pulpitis of an adjacent tooth
C. Prescription of an antibiotic agent
D. Loss of alveolar blood clot

102. Hemorrhage after dental extraction procedure can be


prevented by:
A. Applying pressure
B. Ligating bleeding major vessels
C. Properly designing and carefully reflecting
mucoperiosteal flaps
D. All of the above
103. Trismus after extraction most likely results from:
A. Passing the needle through medial pterygoid muscle
while injecting
B. Injecting the solution near a branch of facial nerve
C. Injecting the solution near a branch of trigeminal nerve
D. Systemic administration of LA

100 c 101 0 102 D 103 A


106 MCQs in Oral Surgery

96. When a root is accidentally displaced into the max-


illary antrum, the dentist should?
A. Enlarge the socket to gain access
B. Place a drain into the socket
C. Refer to an experienced surgeon for root removal
D. Pack the socket with gelfoam to prevent fistula
formation
97. The left hand of the operator should serve the follow-
ing purpose when extracting a mandibular tooth?
A. Support the mandible
B. Protect other teeth
C. Retract soft tissues
D. All of the above
98. Patients with end-stage renal disease are of increased
risk when undergoing extraction of teeth because
they?
A. Have an increased tendency to bleed
B. Are susceptible to infections
C. Are often on steroid therapy
D. All of the above
99. During extraction of max. third molar the maxillary
tuberosity is fractured and remains attached to
periosteum with an intact blood supply. The treat-
ment of choice is to:
A. Refer the case to an oral surgeon
B. Remove the fractured tuberocity
C. Reposition and stablize loss of the tuberosity
D. Send the patient to preventive dentistry with
instructions to prevent

96 c 97 D 98 0 99 c
Exodontia/Impaction 109

109. Contributing factor to pericoronitis of an impacted


mandibular third molar is:
A. Trauma by opposing tooth
B. Previous radiation therapy
C. Systemic disease
D. Infected follicular cyst
110. After the posterior superior nerve block patient should
experience:
A. Numbness of the lip
B. Numbness of the palate till the first premolar
C. Few symptoms of anesthesia
D. Numbness of that side efface
111. The tips of anatomic forceps should be placed:
A. On the crown portion
B. At the junction of clinical and anatomical crowns
C. Near junction of apical and middle thirds of clinical
crown
D. On the root surface as far apically as possible
112. In designing the flap the following is correct:
A. The flap should be as small as possible
B. Free margins should be wider than the base
C. Sutured margins should be supported by a blood clot
D. It should be a m:ucoperiosteal flap
113. Appropriate time to administer analgesic to reduce
postoperative pain associated with removal of two
... mandibular molars:
A. When pain becomes severe
B. Before the effect of LA wears off
C. The morning of surgery
D. Only after the return of sensation

109 A 110 c 111 D 112 D 113 8


108 MCQs in Oral Surgery

104. Local factor for postponing tooth removal is:


A. Periodontitis
B. Hemangioma in the region of the tooth
C. Hypercementosis
D. Malignancy
105. Number 16 cowhorn forceps are specifically
designed to extract:
A. Maxillary first and second premolars
B. Mandibular central incisors
C. Mandibular molars
D. Maxillary molars

106. A small opening is made into the maxillary antrum


during extraction, immediate treatment is:
A. Pack the socket with gauge
B. Allow the clot to form No special treatment is
necessary
C. Place the patient on antibiotics
D. Raise a big mucoperiosteal flap and close the antrum
107. 'Rubber band extraction' is a method of extraction
in patient having:
A. Hemangioma and heamophilia
B. Periocoronitis and impacted molars
C. Supernumerary teeth and ankylosed teeth
D . Myocardial infraction and angina pectoris
108. A straight elevator is properly used to advantage when
the :
A. Adjacent tooth is the fulcrum
B. Tooth is isolated tooth
C. Interdental bone is fulcrum
D. Adjacent tooth is not to be extracted

104 B 105 c 106 B 107 A 108 c


Exodontia/Impaction 111

118. The following maxillary third molar impaction is most


likely to be displaced in to the in.f ratemporal fossa the
antrum if improper technique is used:
A. Mesioangular
B. Vertical
C. Distoangular
D. Mesiolingual
119. The most common complication after surgical re-
moval of a mandibular tooth is:
A. Paresthesia of ling ual nerve
B. Paresthesia of inferior alveolar nerve
C. Postoperative hemorrhage
D. Loss of blood clot
120. Which maxillary impactions are most likely to be dis-
placed into the antrum or the infratemporal fossa with
incorrect technique during the extraction?
A. Distoangular
B. Mesioangular
C. Vertical
D. Facioling ual
121 . Paresthesia of the lower lip is most commonly the
result of:
A. A benign tumor of the mandible
B. Removal of mandibular third molar
C. Periapical abscess of first molar
D. Fracture of the mandible
122. Following may cause postoperative haemorrhage:
A. Liver disease
B. Aspirin and other salicylates medication
C. Prolonged use of broad spectrum antibiotics
D. All of the above

118 c 119 D 120 A 121 B 122 D


110 MCQs in Oral Surgery

114. While extracting mandibular teeth the patient


should be positioned so that the occlusal plane of
the mandible should be:
A. Parallel to the floor when jaws are closed
B. Form some angle to the floor when the mouth is wide
open
C. Parallel to the floor when the mouth is wide open
D. Perpendicular to the floor when the mouth is wide
open
115. While extracting a multirooted tooth, the crown frac-
tures, the prudent next step is to?
A. Divide the roots with a bar
B. Reflect a flap
C. Reapply the forceps in a more apical d irection
D. Leave the roots in place and observe
116. During the extraction of mandibular third molar, it is
noted that distal root of the mandibular third molar
The root is most probably in the:
A. Submandibular space
B. Submental space
C. Inferior alveolar canal
D. Pterygomandibular space
117. A patient came for extraction of his four anterior teeth
all with grade 3 mobility. He is under prednisone 25
mg for the past three months. The dentist should:
A. Take the patient off prednisone therapy
B. Consult the physician who will probably take the
patient off prednisone therapy for at least five days
before the procedure
C. Be concerned with adrenal insufficiency
D. Proceed normally

114 c 115 A 116 A 117 c


Exodontia/Impaction 113

127. A 48-year-old, diabetic patient wants extraction of 2


teeth which are in grade I mobility. He takes 40 units
of NPH insulin daily. Which of the following manage-
ment procedure in indicated to remove the teeth?
A. Defer treatment, have him take breakfast, but not
take insulin to avoid hypoglycemia
B. Defer treatment, have the patient take his insulin but
tell him to avoid breakfast
C. Treat the patient in the usual manner without much
alteration in the diet and drugs
D. Admit the patient into a hospital for the procedure
128. A20-year-old patient requires extraction of all erupted
teeth because of extensive decay. He has two maxil-
lary third molar impactions. X-ray film shows bony
impactions. The molars should be:
A. Extracted along with other teeth
B. Retained because no changes of eruption
C. Retained Lmder a denture until they erupt
D. Exposed surgically and can be used for prosthetic
retention
129. Four days after multiple dental extractions, an elderly
fair skinned women complains of black and blue
marks over her neck. The most probable diagnosis
is:
A. Polycythemia
B. Leukemia
C. Postoperative ecchymosis
D. Manifestation of basal cell carcinoma

127 c 128 c 129 c


112 MCQs in Oral Surgery

123. The difficult tooth to be anesthesized by infiltration


technique is the maxillary:
A. Central incisor
B. First molar
C. Third molar
D. Canine
124. Infiltration in maxillary first molar region is not effec-
tive because:
A. Of zygomatic buttress bone in the region
B. Of the presence of parotid opening in that region
C. Of three roots
D. It is supplied by two nerves
125. The following mandibular teeth can be removed with-
out pain after the inferior alveolar and lingual nerve
block:
A. All molars and second premolars
B. First molar and premolars
C. Canine and first premolar
D. All teeth in that quadrant
126. The following history and laboratory value contrain-
dicate extraction of mandibular first molar:
A. Hb% is 15 gm% and hematocrit value is 42 percent
B. Five months pregnant lady without any complications
C. Rheumatic patient taking penicillin
D. Prothrombin time is 45 seconds

123 B 124 A 125 c 126 D


Exodontia/Impaction 115

134. A mandibular molar was extracted because of ad-


vanced caries. After one month patient developed an
extra oral indurated swelling near the previous ex-
traction site. Multiple draining sinuses appeared and
drained pus with yellow granules. The most probable
diagnosis is :
A. Histoplasmosis
B. Tuberculosis
C. Actinomycosis
D. Streptococcal infection
135. Most serious potential complication after extraction
from areas previously irradiated is:
A. Osteoradionecrosis
B. Alveolar osteitis
C. Prolonged healing
D. Prolonged suppression of salivary secretion
136. A patient complains of persistent bleeding five hours
after the extraction. Initial treatment is to:
A. Advise bleeding time and clotting time and other
laboratory investigations
B. Inject a local anesthetic solution into the .area to
provide vasoconstriction
C. Remove clots and examine the area to locate the source
of bleeding
D. Have the patient bite on a gauge socked with
adrenaline
137. After a tooth extraction under local anesthesia, a pa-
tient with a known cardiac problem experiences an-
gina. This is most likely precipitated by:
A. Allergy to the anaesthetic agent
B. Allergy to preservative agent in the anaesthetic
solution
C. The stimulus of pain and anticipation
D. The epinephrine in the anaesthetic solution

134 c 135 A 136 c 137 c


114 MCQs in Oral Surgery

130. A 45-year-old man requires a tooth extraction. His


medical history indicated myocardial infarction two
years ago. This patient is receiving anticoagulant
drugs. The dentist should:
A. Never operate on this patient
B. Avoid performing surgery until the anticoagulant
had been discontinued for at least three days
C. Perform surgery if prothrombin time is 18 seconds
D. To avoid postoperative haemorrhage advice vitamin
K
131. A patient who is receiving anticoagulants requires
extraction of a tooth. Laboratory data indicating pro·
thrombin time is 21 seconds with a control time of
15 seconds. The treatment should consist of:
A. Regulation of the anticoagulant level with heparin
B. Injection of vitamin K and immediate extraction
C. Injection of vitamin K and extraction after half an
hour
D. Extraction of the tooth and use of sutures and local
hemostatic agent
132. Persistent pyrexia after full mouth extraction results
from:
A. Wotmd infection
B. Dehydration
C. Endocarditis
D . Any of the above
133. Prophylactic antibiotic coverage before dental ex-
traction is indicated for patients with each of the
following conditions except:
A. Kidney damage needing hemodialysis
B. Prosthetic aortic. valve
C. Rheumatic heart disease
D. Coronary artery bypass

130 c 131 D 132 D 133 D


Exodontia/lmpa_ction 117

142. Osteotome is:


A. Monobeveled
B. Bibeveled
C. Tetrabeveled
D. Not contain any bevels
143. The following nerves should be anaesthetized for
extraction of a maxillary lateral incisor:
A. Posterior superior alveolar
B. Nasopalatine
C. Anterior palatine
D. Anterior superior alveolar
144. During tl)e administration of an inferior alveolar nerve
block which of the following muscles is usually pen-
etrated with the needle:
A. Buccinator B. Medial pterygoid
C. Lateral pterygoid D. Masseter
"!45. For the removal of root apex from the maxillary sinus,
one ~hould block the following nerves:
A. First division of trigeminal nerves
B. Second division of trigeminal nerve
C. Third division of trigeminal nerve
D. Infraorbital nerve
146. Treatment of choice for a localised osteitis or dry
socket following extraction of a tooth is:
A. Topical antibiotic application in the socket
B. Systemic antibiotic administration
C. Debridement of the wound by irrigation and
placement
D. Curetting the bony walls of the socket to induce
bleeding and clot formation

142 B 143 B 144 A 145 B 146 c


116 MCQs in Oral Surgery

138. During the extraction of maxillary first molar, the


palatal root tip of molar slipped Into the maxillary
sinus. Proper way to apprach to recovery is through
the:
A. Same socket by enlarging the opening through which
the root entered the sinus
B. Maxillary incisive fossa
C. Maxillary incisive fossa medial to canine
D. Lateral nasal wall in the middle meatus of the nose
139. A mandibular left second molar is unerupted in a 14-
year-old boy. Radiological examination reveals a den-
tigerous cyst surrounding the unerupted tooth. The
treatment of choice is:
A. Observe for at least 2 years
B. Aspirate and send the contents of the cyst to
biochemical analysis
C. Uncover the crown and keep it exposed
D. Extract the tooth under local anesthesia
140. The prime purpose of Blbevel chisel is:
A. Split teeth
B. Sharpen the angles
C. Remove bone
D. Engage point establishment
141. Ammonia inhalation in the treatment for syncope acts
by producing:
A. Bronchodilatation
B. Vasoconstriction
C. Sedation
D. Mucosal irritation

138 c 139 c 140 A 141 D


Exodontia/Impaction 119

151. The elevator can be used to advantage when?


A. Multiple adjacent tooth are to be extracted
B. The interdental bone is used as a fulcrum
C. The tooth to be extracted is isolated
D. The adjacent tooth is not to be extracted
152. Post-extraction localized alveolitis (or) "dry socket"
is usually characterized clinically by:
A. A brawny, indurated swelling with hyperpyrexia
over 100" F
B. Continuous dull pain with little or no adjacent
swelling
C. Attacks of sharp, lancinating pain with swelling
D. Local tenderness and ecchymosis of the skin and
mucous membrane
153. Extraction of an isolated residual maxillary molar of-
fers the technical hazard of:
A. The development of maxillary sinusitis
B. Displacement into the infratemporal fossa
C. Fracture of the tuberosity or floor of the antrum
D. Producing a nasal oral fistula
154. Prophylactic removal of unerupted teeth is accom-
plished with best results at certain stage of tooth
development. The best time to remove such a tooth
is:
A. When two-thirds of the root is formed
B. When the root is completely formed
C. Before any of the root is formed
D. When the crown of the tooth is above the
cementoenarnel junction of adjacent tooth

151 A 152 8 153 c 154 A


118 MCQs in Oral Surgery

147. A patient returns several days after extraction of


three adjacent mandibular molars and complains of
pain similar to dry socket. The first step is to:
A. Currete the three socket area thoroughly in an
attempt to establish new blood clot
B. Explore the sockets gently with curet
C. Irrigate the socket and place the dressing
D. Radiographic examination of the area
148. The following muscle is most frequently encountered
in an incorrect infraorbital nerve block:
A. Orbicularis oris
B. Triangularis
C. Quadratus labi superiors
D. Risorious
149. A patient who is under steroid therapy and has need
for extraction of chronically infected teeth. Premedi-
cation includes:
A. Antihypertensive drugs to prevent shock
B. Atropine to prevent cardiac arrest by vagal
stimulation
C. Antihistamines
D. Antibiotics
150. Forty eight hours after the extraction of impacted
mandibular third molar patient returns to your office
complaining of moderate pain radiating to the ear. His
temperature is ggo F and swelling is minimal. The most
probable diagnosis is:
A. Trauma to the inferior alveolar nerve during the
injection
B. Postoperative infection of parotid space
C. Postoperative infection of masticatory space
D. Post-extraction alveolitis

147 B 148 c 149 D 150 0


Exodontia/Impaction 121

159. The immediate step following the forceps extrac-


tion of a maxillary central Incisor with a periapical
radiolucency is:
A. Examine the root apex for soft tissues pathology
B. Pack the socket with a medicament
C. Currets the alveolus, if tissue was not removed with
extraction
D. AandC
160. "Ash forceps" is commonly used for:
A. Extraction of lower incisor
B. Extraction of upper canines
C. Holding tissues prior to biopsy
D. Holding vascular areas during surgery
161. If there is visible bleeding from an Isolated major ves-
sel, it is best controlled by:
A. Systemic administrations of Vitamin K
B. Electrocoagulation
C. Clamping and ligation
D. BandC
162. Of the following most difficult impaction to extract
is:
A. Mesioangular B. Vertical
C. Distoangular D. Horizontal
163. Rongeur is commonly used to:
A. Cut the bone
B. Extract third molars
C. Place the sutures in the areas where access is limited
D. None of the above

159 D 160 A 161 D 162 c 163 A


120 MCQs in Oral Surgery

155. Following a maxillary tuberosity injection, within a


few seconds, patient face becomes extremely dis·
tended and swollen on the injected side treatment.
is:
A. Incision and drainage
B. Antibiotic administration
C. Use of cold packs and p ressure on the side
D. Antihistamine administration
156. The maxillary first molar is extracted by forceps
method. The healing of the socket can be described
as:
A. Healing by primary intention
B. Granulomatosis
C. Healing by secondary intention
D. Epithelialization
157. The healing process that takes place following
approximation of the flaps are described as:
A. Healing by first intention
B. Healing by second intention
C. Healing by granulomatosis
D. Hematoma formation followed by organization
158. A tooth is resistent to luxation with forceps. Its
removal is best achieved by:
A. Fracture the crown with forceps
B. Fracture the crown with chisel and roots with forceps
C. Transalveolar method
D. Split tooth technique

155 c 156 c 157 A 158 c


Exodontia/Impaction 123

16~. A bony opening in the canine fossa Caldweii-Luc


procedure Is commonly used to:
A. Expose canine root for apicoectomy
B. Enter the maxillary sinus
C. Uncover and marsupilize a cyst
D. Establish drainage of an alveolar abscess
170. Ankylosis of the TMJ is best treated with:
A. Exercise and massage
B. Sclerosing solutions
C. Antibiotics
D. Condylectomy
171. Alveolectomy is one ofthe commonly used procedure.
It is:
A. Closure of an oroantral fistula
B. Removal of alveolar bone
C. Reconstruction of mucoperiosteum and alveolar bone
D. None of the above
172. Which is best treatment for pericoronitis involving an
impacted mandibular third molar?
A. Antibiotic and analgesic therapy
B. Operculectomy
.. C. Extraction of the involved third molar
D. Gently irrigating under the operculum
173. Postoperative edema can be minimized by:
A. Careful surgical manipulation of hard and soft tissues
B. Administration of antihistamines preoperatively
C. Application of heat immediately after the procedure
D. Application of heat and cold simultaneously

169 B 170 D 171 B 172C 173 A


122 MCQs in Oral Surgery

164. Under general anesthesia patient needs oropha-


ryngeal:
A. Lap pad
B. 4 x 4 gauge
C. Ferguson mouth gag
D. Pick up forceps
165. One decided to use forceps for removal of a tooth.
The first direction for the force to be applied is:
A. Buccally ·B. Lingually or palatally
C. Apically D. Occlusally
166. After the extraction of maxillary first molar, a communi-
cation is found between the palatal socket and a dis-
ease free maxillary sinus which measure 0.2 em. The
best treatment ·is:
A. Allow the clot to form advise proper home care
B. Primary closure and antihistamine
C. Gold foil closure
D. Caldwell-Luc operation
167. An approach to an impacted mandibular molar is
achieved by:
A. Envelop flap
B. Bayonetflap
C. L shaped flap
D. All of the above
168. The muscles that aid in displacement of maxillary frac-
tures are:
A. Masseter
B. Temporalis
C. Upper part of orbicularis and lower part of orbicularis
occuli
D. None of the above

164 B 165 c 166 A 167 D 168 D


Exodontia/Impaction 125

179. A 30-year-old female patient with a BP of 150/80 re-


quires treatment. You would:
A. Recommend generai anesthesia
B. Seek medical treatment to lower BP
C. Refer the patient to an oral surgeon
D. Use sedation preoperatively
180. Among the following which condition will require
specific management prior to emergency extrac-
tion?
A. Hemophilia
B. Glaucoma
C. Carcinoma tongue
D. Mixed tumor of parotid
181. Elective dental extractions and minor oral surgical
procedures on a patient who has had a myocardial
infarct two months prior are best?
A. Performed using an adrenaline free local anaesthetic,
because of adrenaline action on heart muxcle
B. Performed under oral diazepam
C. Postponed until at least six months have elapsed
D. Performed with normal
182. Among the following, which is true about mucoperi-
osteal flap design?
A. Flaps should be narrower at the base
B. Flap margin should not rest on sound bone
C. Flap should be wider at the base
D. Flaps should be avoided

179 D 180 A 181 c 182 c


124 MCQs in Oral Surgery

174. During the dental procedure suddenly patient be-


came "light headed, diaphoretic" then became un-
conscious. The most diagnosis is:
A. Shock
B. Syncope
C. Cerebrovascular accident
D. Hyperventilation
175. One day after the extraction, patient came with the
complaint of a rubbery, nontender midfacial swelling.
The most likely diagnosis:
A. Abscess "B. Fibroma
C. Hematoma D. Inflammatory edema
176. Among the following which is most appropriate pre-
operative instruction for a diabetic patient undergo-
ing extraction of his teeth:
A. Maintain normal diet and normal medication
B. Increase medication preoperatively
C. Increase sugar intake prior to surgery
D. Do not eat or take any medication before the surgery
177. A patient who is a grand mal epileptic requires emer-
gency extraction. One should:
A. Advise the patient that procedure require admission
to the hospital
B. Proceed with extraction using xylocaine without
adrenalin
C. Proceed with extraction if anti-convulsive medication
is employed by patient
D. Inform the patient that in his condition no extraction
should be performed
178. Which of the following medication require preopera-
tively for a patient with mitral valve replacement?
A. Ampicillin B. Cloxacillin
C. Benzathine penicillin D. Procaine penicillin
174 B 175 c 176 A 177 c 178 D
Exodontia/Impaction 127

188. A mandibular second molar is extracted. It is found


that the distal root is fractured and the tip is missing.
When the dentist attempts to recover it and finds that
his instrument will pass downward and into the
lingual soft tissue, the tip is most likely to be in the:
A. Submental space
B. Sublingual space above the mylohyoid muscle
C. Submandibular space below the mylohyoid muscle
D. Parapharyngealspace
189. 48-72 hours after extraction patient developed fever.
The problem may result from:
A. Wound infection
B. Endocarditis
C. Cellulitis
D. Any of the above are corrected
190. Wilkinson extractions are:
A. A type of extractions done with wilkinson elevator
B. Done in order to permit eruption of 3rd molars in the
proper position
C. Usually done in the patients above 45 years
D. None of the above
191. The best time of extraction in pregnancy:
A. First trimester
B. Second trimester
C. Third trimester
D. None of the above

166 c 169 D 190 B 191 B


126 MCQs in Oral Surgery

183. Among the following which condition is contraindi-


cated to the use of local anesthesia?
A. Pregnancy
B. Hypersensitivity to the drug
C. Hepatic damage
D. Parkinson's disease
184. Post extraction complications are most often caused
by:
A. Inadequate diet
B. Inadequate preoperative testing
C. Inadequate patient instructions
D. Poor surgical technique
185. Among the following which is most important in pre-
vention of post extraction bleeding problems?
A. Transfusion of blood
B. Tight suturing of flaps
C. Ice to face
D. Local pressure
186. Five days after extraction of an Impacted mandibu-
lar third molar, your patient presents with hard, pain-
ful swelling of 12 hours duration near angle of the
mandible. This is most likely due to:
A. Angioneurotic edema B. Hematoma
C. Infection D. Surgical trauma
187. Inadvertent displacement of the teeth into inacces-
sible areas of head and neck and be effectively pre-
vented by:
A. Use of prudent surgical technique
B. Use of gauze pharyngeal curtain
C. Evaluation of radiography prior to surgery
D. All of the above

183 B 184 D 185 D 186 c 187 D


Exodontia/Impaction 129

197. Among the following which muscle is most fre·


quently encountered In an incorrect infraorbital
nerve block?
A. Quadratus labi superioris
B. Mentalis
C. Rizorius
D. Orbicularis oris
198. Management of dry socket (localised alveolar
osteitis) include all of the following except:
A. Curetting the socket and promote bleeding
B. Normal routine irrigation lo flush out the debris
C. Placing a sedative dressing lo protect the exposed
bone
D. Advising analgesics as an 'ld juncti\·e l:eatment

197 A 198 A
128 MCQs in Oral Surgery

192. Of the following which teeth need primarily rota-


tory movement to extract when:
A. Maxillary first premola r
B. Mandibular second p remolar
C. Mandibular central incisor
D. Mandib ular la teral incisor
193.. Bone removal with aeroter is usually contra indicated
because of the danger of developing:
A. Hematoma
B. Emphysema
C. Abscess
D. Edema
194. Which of the following nerves should be blocked to
remove a root tip from the antrum?
A. Anterior superior alveolar nerve
B. Posterior superior alveolar nerve
C. Second division of the trigeminal nerve
D. Third division of trigeminal nerve
195. Which of the following procedures eliminate hemor-
rhage after dental extraction procedures?
A. Ligating bleeding vessels
B. A p p lying press u re a nd car efully re fl ecting
mucoperiostema
C. Eliminating systemic bleed ing d isord ers
D. All of the above
196. Cavernous sinus thrombosis following infection of
maxillary and anterior teeth most often results from
spread of infected .emboli along the:
A. P terygoid plexus
B. Ophthalmic vein
C. Facial artery
D. Angular a rtery

192 B 193 B 194 c 195 D 196 B


Minor Surgical Procedure 131

5. An excisional biopsy is characterized by:


A. Using a trephine
B. Exfoliative cytologic techniques
C. Including normal tissues and most of the lesion
D. Including normal tissue and all of the lesion
6. In oral surgery, cold pac Epulis Fissuratum is most
commonly due to:
A. ill fitting dentures over resorbed ridges
B. Infection by Candida albicans
C. Poorly performed 'Z' plasty
D. None of the above
7. The recurrent ranula is best treated by:
A. Electrosurgery
B. Cryosurgery
C. Marsupialization
D. Sub-lingual gland excision
8. lncisional biopsy of an ulcer is taken:
A. At the center of the lesion
B. Edge of the lesion
C. Edge of the lesion along with normal tissues
D. At any part of the lesion

9. Which one of the following should not be done dur-


ing in incisional biopsy of soft tissue?
A. Place suture through the intended specimens before
removing it
B. Infiltrate local anaesthetic solution around the
intended site
C. Place the specimens in saline if 10% formalin is
unavailable
D. Obtain some adjacent normal tissue if possible

5 D 6 A 7 D 8 c 9 c
MINOR SURGICAL
Is I PROCEDURE

1. Repeat biopsies are indicated:


A. For blastomatoid conditions
B. When ulcers fail to heal in four days
C. In bone defect lesions that fail to heal in six weeks
D . When clinical course differs from a reported benign
process
2. Hyperplastic palatal ·tissue known as papillomatosis
is seen often beneath dentures. This condition is best
treated by:
A. Supraperiosteal dissection or electrosurgery
B. Not allowing the patient to wear his denture
C. Radical excision because it is premalignant
D. Chemical cautery
3. A soft tissue flap consisting of mucosa and perios-
teum can be repositioned accurately because the:
A. Periosteum is very elastic
B. Mucosa is mostly collagen fibers
C. Periosteum sends filiform-like projections into the
haversian canals
D. Periosteum is relatively inelastic
4. Treatment of a mucocele on the lower lip is by:
A . Incision
B. Excision
C. Electrosurgery
D. Excision, including adjacent minor salivary glands

1 D 2 A 3 D 4 0
Minor Surgical Procedure 133

15. How long should one wait before obtaining a biopsy


of an oral ulcer?
A. 4 days B. 7 days
C. 14 days D. 30 days
16. To prevent tearing of the mucosal edges during wound
closure, the following needle should be used:
A. Round bodied B. Cutting
C. Reverse cutting D. Five edged

17. The following principles should be adhered to when


performing alveoplasties after extractions?
A. Conservation of bone
B. Use only absorbable sutures
C. The soft tissue margins should be tightly
approximated
D. All of the above
18. Damage to the following structure should be avoided
while performing an apicectomy on tooth no. 7 (12):
A. Nasal floor
B. Inferior alveolar canal
C. Anterior superior nerve
D. Incisive canal
19. A suitable fixative for a routine biopsy specimen is:
A. 10 % formalin
B. 40 % formalin
C. 20 % formalin
D. 10% alcohol

15 c 16 A 17A 18 A 19 A
132 MCQs in Oral Surgery

10. The purpose of aleveoloplasty is to


A. Eliminate the undercuts that interfere with seating of
the denture
B. Avoid rounding off the sharp bony projections as they
will round off in a few days during the healing process
C. Retain the width of the alveolar ridge even though
undercuts exists
D. None of the above

11. Packs. are applied:


A. Immediately prior to surgery
B. Immediately after surgery
C. To prevent the spread of infection
D. To produce vasodilation
12. During full mouth extraction of teeth and
alveloplasties, blood loss can be expected to be in
the range of:
A. 0-100 ml B. 200-800 ml
c. 800-1000 ml D. 1000-1500 ml
13. The most frequently used approach to temporoman-
dibular joint surgery is:
A. Endural B. Submandibular
C. Preauricular .D. Postauricular
14. The primary Intent of an extra-oral surgical dressing
is to :
A. Support the incision
B. Protect the surgical site from trauma
C. Absorb drainage
D. Keep the surgical field free from infection

10 A 11 B 12 B 13 c 14 D
Minor Surgical Procedure 135

25. Among the following which is logical explanation that


explains how an abscessed maxillary canine causes
swelling beneath the eye?
A. Lymphatics drain upward in this region
B. Apex of caine lies above the attachment of the caninus
and levator labi superioris mucles
C. Infection spread through the angular vein which has
no valves
D. Bone is less porous above the apex of the canine which
facilitates the spread of infection
26. Most tuberosity reductions before denture construc-
tion requires:
A. Primarily soft tissue or fibrous tissue removal
B. Large amoung of bone removal
C. Relief from pterygoid hamulus
.. D. Some bony reductions in all instances
27. Incision and drainage {I and D) in an area of acute
infection should be performed when?
A. Induration has occurred
B. Localization has occurred
C. Acute pain is present
D. There is fever which is above 102° F
28. During the apicectomy on a maxillary lateal incisor,
one should take care not to perforate or damage the:
A. Maxillary sinus
B. Labial frenum
C. Floor of the nose
D. Contents of the incisive canal

25 B 26 A 27 B 28 c
134 MCQs in Oral Surgery

20. To achieve surgical haemostasis In a patient suffer-


ing from Haemophilia A, preoperative administration
of the following blood product should be preferred:
A. Whole blood only
B. Fresh frozen plasma
C. Factor VIII cryoprecipitates
D. Factor VIII concentrates
21. A skin graft vestibuloplasty prevents relapse by:
A. Physically maintaining the depth of the vestibule
B. Preventing the reattachement of the muscle to the
periosteum
C. Causing an inhibitory effect on fibroblasts in the
underlying tissue
D. All of the above
22. Pre-prosthetic surgery on the mandible requires the
preservation of the:
A. Labial frenum B. Palatal tori
C. Mylohyoid ridge D. Genial tubercles
23. Chromic catgut sutures are preferred over plain cat-
gut as they exhibit:
A. Greater strength
B. Less tissue inflammatory response
C. Delayed resorption
D. Ease of manipulation
24. Removal of mylohyoid ridge is planned in an edentu-
lous patient. Which of the following nerves should be
protected?
A. Mental B. Mylohyoid
C. Lingual D. Inferior alveolar

20 D 21 A 22 D 23 A 24 c
136 MCQs in Oral Surgery

29. A mucocele of the lower lip should be treated by:


A. Incision
B. Electrocautery
C. Excision including adjacent minor salivary gland
D. ·Incision and drainage followed by antibiotics
30. Among the following which definite sign/symptom
Indicate infection of masticator space
A. Difficulty in swallowing
B. Trismus
C. Decreased salivary secretion (Xerostomia)
D. Fever above 100°F
31. Antibiotic medication for pericoronitis should be
advised:
A. If extraction is delayed or postponed
B. Before surgery
C. Routinely to treat pericoffcnitis
D. If trismus and fever present
32. Among the following which stage of anaesthesia
describes the level of conscious sedation:
A. Stage I
B. Stage II
C. Stage III plane 1
D. Stage III plane 2
33. Alveolectomy for prosthetic reasons should be per-
formed:
A. During the time of extraction of teeth
B. One month after the extraction
C. 2 months after the extraction
D. At the time when the complete dentures are to be
constructed

29 c 30 B 31 D 32 A 33 A
Minor Surgical Procedure 137

34. lrritational fibroma which is asymptomatic is best


treated by:
A. No treatment is necessary because it is asymptomatic
B. Simple excision
C. Radiation therapy
D. Wide excision followed by chemotherapy
35. Commonly employed approach for the removal of for-
eign body or root tip from the maxillary antrum is:
A. Through the canine fossa
B. Through the socket by enlarging it
C. By nasal antrostomy
D. Palatal approach
36. Nasal antrostomy usually done from this:
A. Middle concha
B. Inferior concha
C. Middle meatus
D. Inferior meatus
37. Commonest complication with Risdon and subman-
dibular incision is:
A. Damage to marginal mandibular nerve
B. Damage to cervical branch of facial nerve
C. Damage to inferior alveolar nerve
D. Damage to lingual nerve
38. To remove an undercut maxillary tuberosity, the fol-
lowing flap design criteria is true:
A. Incision should be made at the crest of the ridge
B. A "wedge" should be done first
C. Incision may extend to premolar canine area
D. All of the above

34B 35 A 36 D 37 A 38 D
138 MCQs in Oral Surgery

39. In sutured skin incisions of the face, the edges of the


skin should be:
A. Everted B. Inverted
C. Edge to Edge D. Separated
· 40. The following methods are commonly used methods
of ridge extension procedure of the mandible:
A. Secondary epithelialization
B. Mucosal graft vestibuloplasty
C. Skin graft vestibutoplasty
D. All of the above
41. Maxillary tori are usually removed by:
A. Burs and chisels as indicated to section and remove
tabulation
B. Bone forceps to hold torus while it is being sectioned
C. Rongeurs to crush bone
D. All of the above
42. The 'Z' plasty procedure is commonly used to cor-
rect:
A. Oro-antral fistula
B. Ankyloglossia
C. Interfering labial frenum
D. BandC
43. At the time of extraction of upper third molar, the tu-
berosity is fractured, but still attached to the perios-
teum. Which of the following treatment should be
employed?
A. Remove the tuberosity and file the area and suture
the soft tissue
B. Remove the tuberosity and fill the defect with
osteogen and suture it
C. Leave the tuberosity and stabilize it
D. None of the above

39 A 400 41 A 42 D 43 c
Minor Surgical Procedure 139

44. To drain pus from an abscess, the surgeon should:


A. Cut .an eclipse from the abscess surface to allow for a
drain
B. Aspirate the contents
C. Penetrate into abscess cavity and probe with an artery
to allow for flow of pus
D. Cut only the mucosa and skin,
45. Two em highly suspicious lesion of the floor of the
mouth is to be studied microscopically. A specimen
is best obtained:
A. Inscisonal biopsy
B. Excisional biopsy
C. FNAC
D. None of the above
46. The severe complication of the cani~Je space infec-
tion is:
A. Erosion of internal carotid artery
B. Erosion of external carotid artery
C. Cavernous sinus thrombosis
D. Respiratory paralysis
47. The treatment of choice for traumatic bone cyst is:
A. Marsupialization and leaving the cyst lining
B. Opening the cavity and inducing bleeding
C. Enucleation of the cyst lining
D. Packing with BIPP and prevent bleeding and clot
formation
48. Treatment of choice to a localized infection with pus
is:
A. Antibiotic administration
B. Establish drainage
C. Apply cold to the area
D. Advise hot mouth washes

44C 45 A 46 c 47 B 48 B
140 MCQs in Oral Surgery

49. Of the following which is most frequently Indicated


treatment for odontogenic cysts:
A. Fulguration
B. Enucleation
C. Marsupialization
D. Incision and drainage
50. A patient came to dental clinic with a growth on the
inside of the cheek that is approximately 0.5 em in
diameter and not painful. He noticed the swelling 3
months back. The best treatment Is to:
A. Remove all of growth and send for biopsy
B. Observe periodically
C. Start antibiotic medication
D. Remove part of the growth and send for biopsy
51 . Among the following which Is treatment of choice for
infection with fluctuation in an afebrile patient?
A. Administration of antibiotics
B. Application of hot packs to the area
C. Incision and drainage
D. Antibiotics administration followed by I and D
52. Among the following which Is a problem with
reconstruction ofthe atrophic ridge with hydroxy apa-
tite granules:
A. Displaced material
B. Mental nerve dysesthesia
C. Poor ridge form
D. Any of the above
53. Which muscle should be detached to lower the floor
of the mouth?
A. Mylohyoid B. Genio/hyoid
C. Genioglossus ·D. A and C

49 B 50 c 51 c 52 D 53 D
Minor Surgical Procedure 141

54. Of the following in which condition a biopsy should


not be performed on a tissue. When the tissue?
A. Responds to local treatment
B. Is associated with paresthesia
C. Suddenly enlarges
D. B andC
55. Marsupialization is a surgical procedure which com-
monly used to treat a/an? ·
A. Cystic odontoma
B. Periapical cyst
C. Large cyst
D. Oro antral fistula
56. Of the following movement should be employed to
reduce a dislocation of the mandible:
A. Downward and forward
B. Downward and backward
C. Upward and backward
D. Upward and forward
57. Papillomatosis or hyperplastic palatal tissues most
often seen beneath dentures. This condition is best
treated by:
A. Not allowing the patient to wear the dentures
B. Supraperiosteal dissection or electrosurgery
C. Chemical cautery
D. Radical excision because this has malignant potential
58. Which of the following represent excisional biopsy?
A. Exfoliative cytologic technique
B. Including normal tissue and all of the lesion
C. Including normal tissue and most of the lesion
D. Removing a representative section of the pathological
tissue or lesion and some adjacent normal tissue for
comparison

54 A 55 c 56 B 57 B 58 B
142 MCQs in Oral Surgery

59. Of the following conditions where acrylic splints are


advisable during the postoperative phase of manage-
ment:
A. Torus palatinus reduction
B. Mental tubercle reduction
C. .Mandibular alveoplasty
D. None of the above
60. During the removal of a maxillary torus, the mid por-
tion of the palatine process of the maxilla is acciden-
tal~y removed. One might see:
A. Vertical fracture of the maxilla
B. An opening into the nasal cavity
C. Opening into the maxillary antrum
D. Horizontal fracture of the maxilla
61. In the usual approach to mandibular tori the incision
is usually placed:
A. Parallel to the internal oblique ridge
B. Over the alveolar crest
C. Over the prominence of the tori
D. Below the mylohyoid line
62. Of the following, where antrostomy is commonly
used?
A. To remove high maxillary third molars
B. To expose a tooth root for apicectomy
.C. To treat an alveolar abscess
D. Following closure of an oroantral fistula
63. Which of the following may cause paresthesia of the
lower lip?
A. Malposed maxillary third molar
B. Removal of a mandibular third molar
C. Removal of torus mandibularis
D. None of the above

59 A 60 B 61 B 62 D 63 B
Minor Surgical Procedure 143

64. Of the following which is best method to treat a large


radicular cyst of the maxilla with apical involvement
of four vital teeth?
A. Marsupialization
B. Aspiration
C. Enucleation and primary closure
D. Enucleation and packing the cavity with BIPP
65. All of the following are safely excised in preparing
complete dentures except:
A. A pendulous tori
B. Genial tubercles
C. Feather edged ridges
D. Mylohyoid ridge
66. - - - - - is best to restore a missing mandibular
portion.
A. Homogenous bone
B. Autogenous bone
C. Silastic
D. Tantalum
67. Which of the following criteria should be considered
before a third molar transplantation?
A. The root is atleast half formed
B. The width of the crown approaches the width of the
extracted tooth
C. A and B
D. None of the above
68. Most objective way to detect sharp ridges of bone
while performing alveoloplasty Is:
A. Place a finger over the body ridge and palpate
B. Place a finger over the soft tissue flap and palpate
C. See the area for sharp bony projections
D. None of the above

64A 65 B 66 B 67 c 68 B
144 MCQs in Oral Surgery

69. Minimum volumes of local anaesthetic agent is ad-


vised for infiltration when excising frenum because
larger volumes may?
A. Cause tissue slough
B. Distort the tissues
C. Cause rebound bleeding
D. Cause toxic effects
70. Of the following conditions, where an incisional bi·
opsy is indicated?
A. 4 em hemangioma of the tongue
B. 3 em leucoplakia of the soft palate
C. 0.5 em papillary fibroma of the gingiva
D. 1.5 em exostosis of jaw
71. A negative histopathology report of a highly suspi-
cious oral lesion suggest?
A. No malignant potential of the lesion
B. Lesion should be stained with toludine blue like stains
C. Periodical recalls are necessary to assess the nature of
the lesion
D. That another biopsy is necessary in view of the clinical
impression
72. Skin graft vestibuloplasty prevents relapse by:
A. Causing an inhibitory effect on fibroblasts
B. Forms a barrier to reattachment of muscle to
periosteum
C. Promoting osteogenesis and bony barrier is
established
D. Relapse is common in skin graft vertibuloplasty

69 B 70 B 71 0 72 B
Minor Surgical Procedure 145

73. Among the following, where excisional biopsy is in-


dicated?
A. 2 em hemangioma of the tongue
B. 3 em leukoplakia of the left palate
C. 0.5 em fibroma of the gingiva
D. 3.5 em area of Fordyce spot of the check
74. The soft tissue incision used to remove mandibular
torus should be placed:
A. Directly over the torus
B. Inrerior to the torus in the area oi the floor of the
mouth
C. Over the edentulous alveolar crest in the gingival
crevice aroung the dentition
D. A and B
75. After the incisional biopsy of a lesion in the floor of
the mouth and placement of sutures, the patient ex-
periences severe bilateral swelling of the submental
and submandibular spaces. This is ca!Jsed by:
A. Ludwig's·infection
B. Too much fluid intake postoperatively
C. Injury to slingual artery with haemorrhage
D. Extravasation into these compartments

73 c 74 c 75 D
CYST/TUMOUR/
SURGICAL
PATHOLOGY

1. Marsupialization is a surgical procedure which may


be used in treating?
A. A large cyst
B. A cystic odontoma
C. A periapical cyst of 1 em
D. On oral antral fistula
2. A painless, fluid-filled retention cyst appearing in the
area of recent dental treatment may be the result of:
A. Failure of absorption of the anesthetic agent
B. Allergic reaction to some agent employed
C. Infection ancurred during treatment
D. An injury obstructing or blocking a minor salivary
gland
3. A swelling on the anterior floor of the mouth is soft
and painless. It has been present for several months.
The overlying mucosa has a bluish tingue. The most
likely diagnosis is:
A. A retention cyst
B. A mixed salivary gland tumor
C. An infected periodontal abscess
D. A carcinoma of the floor of the mouth

1 A 2 D 3 A
Cyst/Tumour/Surgical Pathology 147

4. Sialoliths are most commonly found in the:


A. Parotid duct
B. Parotid gland
C. Sublingual gland
D. Submandibular duct or gland
5. Bifid mandibular canal is somewhat common in the
following disease:
A. Pagets disease
B. Fibrous dysplasia
C. Neuro fibromatosis
D. A and B
6. Of the following which nevus has the maximum
potential to become melanoma?
A. Junctional nevus
B. Blue nevus
C. Hairy nevus
D. Intradermal nevus
7. Ofthe following which group of lymph nodes one most
commonly involved in tubercular lymphadenitis?
A. Axillary B. Cervical
C. Submental D. Submandibular
8. Which of the following is correct regarding rodent
ulcer?
A. Common site of occurrence is lateral border of the
tongue occationally seen on the ventral surface of the
tongue
B. More common in blacks
C. Antibiotics particularly penicillin is effective in the
treatment of the disease
D. None of the above are correct

4 0 5 c 6 A 7 8 8 D
148 MCQs in Oral Surgery

9. Of the following areas, where retention cysts com-


monly occur?
A. Floor of the mouth B. Upper lip
C. Lower lip D. Buccal mucosa
10. Marsupialization is associated with the treatment for:
A. Abscess B. Pericoronal infection
C. Cyst D. Cellulitis
11. Mixed tumors of major salivary glands are most fre-
quently found in the:
A. Tip B. Palate
C. Tongue D. Parotid gland
12. The swelling of bone usually seen with a cyst is due to:
A. Ballooning of cortical plates
B. New subperiosteal deposition
C. Soft tis'sue reactive swelling
D. Cortical plates being reabsorbed and cystic lining
causes the swelling
13. In an otherwise asymptomatic cystic swelling there
is sudden neurapraxia in inferior alveolar nerve re-
gion it can be due to:
A. Infection of cyst
B. Expansion of periosteum due to cyst
C. Neuritis
D. Neuralgia
14. A patient presents with a non vital and swelling in the
labial sulcus. On aspiration straw coloured. fluid is·
present a tentative diagnosis would be:
A. Nasopalatine cyst B. Solitary bone cyst
C. Keratocyst D. Periapical periodontal cyst

9 c 10 c 11 D 12 B 13 A 14 D
Cyst/Tumour/Surgical Pathology 149

15. With infected large cyst the adjoining teeth give a


negative vital response, it is:
A. Non reversible
B. Reversible after decompression
C. Extraction of the teeth is a must
D. None of the above
16. In following situations an artefact may simulate a cys-
tic lesion (except in):
A. Radiolucent area in periapical region of central
incisors
B. Radiolucent area apical to mandibular premolars
C. Radiolucent area apical to maxillary canines
D. Radiolucent area in ramus of the mandible below
sigmoid notch
17. To obtain better informative radiographic view of a
cyst which has eroded the cortex considerably:
A. Exposure time should be reduced
B. Exposure time should be increased
C. Voltage should be increased
D . Milliampere should be increased
18. Partsch operation is same as:
A. Marsupialisation
B. Decompression with enucleation
C. Only enucleation
D. Enucleation with marsupialisation
19. Fissural cysts should be ideally treated by enucleation
because:
A. The cysts are smaller in size
B. They never enlarge to a size where enucleation cannot
be done
C. The bone does not always regenerate from margins
and fissural areas
D. Marsupialisation would leave the cavities open and
these areas are more susceptible to infection

15 0 16 0 17A 18 A 19 c
150 MCQs in Oral Surgery

20. Bohn 's nodules are:


A. Cystic swellings in neonates
B. Cysts associated with soft palate
C. Cysts of gingiva "in growing children
D. Warts on the tongue
21. The choice of treatment for keratocyst should be:
A. Marsupialisation
B. Marsupialisation with second ary closure
C. Enucleation
D. Marsupialisation with p rimary closure
22. The high recurrence rate of keratocysts is incriminated
to:
A. Its fragile thin lining
B. Presence of d aughter cysts in the cystic lining
C. Presence of d augh ter cysts in the capsule of the cyst
D. All of the above
23. The protein content of periapical and dentigerous
cysts is:
A. 5-11 gm/dl
B. Less than that of serum
C. More than tha t of serum
D. Less than 4 gm/ dl
24. The window for marsupialisation shows maximum
contracture when :
A. It consists of sulcus mucosa supported by loose
connective tissue
B. It consists of sulcus mucosa w ith firm periosteal bed
loosely adherent to bone
C. It consists of mucoperiosteum firmly adherent to
underlying bone
D . Cyst is large

20 A 21 c 22 D 23 A 24 A
Cyst/Tumour/Surgical Pathology 151

25. Complete enucleation of cyst in palatal area carries


danger of:
A. Excessive bleeding from nasopalatine artery
B. Severing of nasopalatine nerve
C. Tear of nasal mticosa
D. Alteration of speech
26. In cysts of maxilla involving maxillary sinus
marsupialisation pack should be changed from:
A. Nasal antrostomy
B. Caldwellluc operation
C. Palatal opening
D. From tooth socket
27. In a 48-year-old patient the treatment of dentigerous
cyst with impacted molar lying near lower border of
mandible, would be:
A. Enucleation with primary closure and IMP
B. Marsupialisation with extraction of molar
C. Marsupialisation with IMF
D. Enucleation with secondary closure
28. In which ofthe situation(s) the cystic lining would have
become thick and adherent?
A. Infection
B. Already decompressed earlier
C. Tooth has been extracted without treating the cyst
D. All of the above
29. During enucleation the incision should be placed on:
A. Firm bony base
B. Mucosa only
C. Cystic lining
D. Cystic capsule

25 c 26 A 27 c 28 0 29 A
152 MCQs in Oral Surgery

30. If during enucleation of a cyst the apices of adjacent


normal teeth become exposed:
A. Retrograde filling should be done with calcium
hydroxide
B. They should be treated endodontically with
apicectomy
C. Dressing and secondary closure should be done
D. If vital they should be observed for three months
31. One of the main reasons for break-down of suture line
after primary closure is:
A. Dead space
B. Haematoma formation
C. Saliva seepage
D. Retaining adjacent teeth
32. The best material to be packed in large bony cavity
after enucleation of cyst is:
A. Allogenic bone
B. Hydroxy apatite·crystals
C. Autogenic medullary bone chips
D. Autogenic cortical pieces
33. Cysts from the following teeth usually expand pala-
tally (except):
A. Maxillary lateral incisors
B. Roots of maxillary premolars
C. Upper molars
D. Upper central incisors
34. An early stage cementoma may be erroneously diag-
nosed as a periapical cyst but for the:
A. Position of lesion
B. Vitality of tooth
C. Radiopacity of the margin
D. Age of the patient

30 D 31 B 32 c 33 D 34 B
Cyst/Tumour/Surgical Pathology 153

35. Lateral developmental periodontal cyst is typically


present:
A. In relation to vital teeth
B. In relation to submental space
C. In relation to pulpless maxillary teeth only
D. Supernumerary non vital teeth

36. Eruption cysts should be treated:


A. .Immediately with enucleation
B. By marsupialisation
C. With no active treatment
D . With antibiotics.
37. The protein content of keratocyst is found to be:
A. < 4 gm/ dl
B. 6 gm / dl
C. Equal to serum protein
D. More than serum protein
38. Treatment of keratocyst is:
A. Marsupialisation
B. En ucleation
C. Excision
D. Enucleation with secondary closure
39. A 36-year-old patient presented with an asymptomatic
swelling on left side of body of mandible, radiograph
shows small radiopaque specks within the bone cav-
ity and on aspiration straw colored fluid was present.
It is a typical picture of:
A. Amaeloblastic adenomatoid tumour
B. Calcifying epithelial odontogenic cyst
C. Keratocyst
D. Cystic odontoma

35 A 36 c 37 .A 38 c 39 8
154 MCQs in Oral Surgery

40. A 26-year-old male patient presented with multiple


keratocysts, basal cell carcinoma on right cheek and
Dyskeratosis with bifid rib. Diagnosis would consti-
tute:
A. Godin's cyst
B. Carlin's syndrome
C. Marfan's syndrome
D. Pierre Robin syndrome
41. Globulomaxillary cyst occupies bony region between:
A. Maxillary central incisor and lateral incisor
B. Maxillary lateral incisor and canine
C. Maxillary canine and premolar
D. 1st and 2nd premolar of maxilla
42. A 18-year-old boy presented with swelling in labial
sulcus, difficulty in breathing, skiagram shows no
radiolucent lesion of the bone and on aspiration straw
coloured fluid was present. It could be:
A. Medial palatine cyst
B. Nasoalveolar cyst
C. Nasal polyp
D. Nasopalative duct cyst
43. A patient 14 years of age presented with swelling on
right mandible, the adjacent teeth were vital. Radio-
logically there was an extensive radiolucent lesion
with scalloped margin extending between the roots
and lamina dura was intact. On aspiration golden yel-
low coloured fluid was present, a tentative diagnosis
can be:
A. Aneurysmal bone cyst
B. Haemorrhagic bone cyst
C. Stafne's bone cyst
D. Static bone cyst

40 B 41 B 42 B 43 B
Cyst/Tumour/Surgical Pathology 155

44. Traumatic bone cyst is also known as:


A. Solitary bone cyst
B. Haemorrhagic bone cyst
C. Static
D. A and B
45. Stafne's bone cyst is a:
A. True cyst
B. Bony depression above inferior alveolar canal
C. Bony depression below inferior alveolar canal
D. Radiolucent area below inferior alveolar canal
46. The static bone cyst should be treated by:
A. Enucleation
B. Marsupialisation
C. No active treatment
D. Exploration and -closure
47. An example of retention cyst is:
A. Mucocele
B. Ranula
C. Dermoid cyst
D. Branchial cyst
48. Mucocele should be treated by:
A. Marsupialisation
B. Enucleation of cyst
C. Enucleation of the cyst and the minor salivary gland
D. Decompression only
49. Trauma to the excretory ducts of sublingual salivary
glands causes:
A. Ranula B. Mucocele
C. Solitary cyst D. Sialolithiasis

44 D 45 C/0 46 C 47 B 48C 49 A
156 MCQs in Oral Surgery

50. Following cysts occur on lateral side of neck:


A. Dermoid cyst
B. Thyroglossal cyst
C. Branchial cyst
D. Epidermoid cyst
51. A patient presented with a small cystic swelling in
anterior region of neck which moved on swallowing
and on protrusion of tongue, it could be:
A. Dermoid cyst
B. Th yroglossal cyst
C. Epidermoid cyst
D. Branchial cyst
52. A 33-year-old patient reported w ith an extensive
ameloblastoma of mandible but the lower border was
not involved, the treatment should be:
A. Curettage
B. En-block resection
C. Segmental resection
D. Hemimandibulectomy
53. The recommended treatment modality for Pindborg's
tumour is:
A. Curettage
B. Enucleation
C. Excision/resection
D. Marsupialisation with secondary closure
54. Adenoameloblastoma should be managed by:
A. Enucleation
B. En-block resection
C. Segmental resection
D . Hemi/partial mandibulectomy

50 c 51 B 52 B 53 c 54 A
Cyst/Tumour/Surgical Pathology 157

55. When performing curettage in aggressive tumours


such as ameloblastoma or Pindborg's tumour, one
should :
A. Not sacrifice vital structures present in the area
B. Sacrifice the vital structures present in the area
C. Both tumours are not aggressive
D. None of the above
56. The surgical management of Brown's tumour of man-
dible is:
A. En-block resection
B. Segmental resection
C. Parathyroidectomy
D. No treatment
57. A patient presented with a radiolucent lesion and bi-
opsy report shows ·giant cells the lesion could be:
A. Giant cell granuloma
B. Brown tumour
C. Cherubism
D. All of the above
58. A 8-year-old boy presented with bilateral swelling of
mandible which was asymptomatic and slowly pro-
gressive in nature, radiologic picture had extensive
bilateral multilocular radiolucencies in posterior man-
dibular angle and body, this is a characteristic pic-
ture of:
A. Hand-Schi.i.ller-Christian disease
B. Letterer-Siwe disease
C. Cherubism
D. Eosinophillic granuloma

55 B 56 c 57 D 58 c
158 MCQs in Oral Surgery

59. A hypertensive patient having cafe au lait spots, hy-


poplastic maxilla and signs of mental deficiency pre-
sented with a soft tissue mass on buccal mucosa, this
should insist surgeon to investigate for:
A. Neurofibromatosis
B. Neurilemoma
C. Down syndrome
D. Pierre Robin syndrome
60. A 35-year-old patient with history of trauma com-
plained of intermittent pain in mandible with .areas of
paraesthesia. X-ray picture revealed a radiolucent area
extending into the inferior alveolar canal, on aspira-
tion no fluid/ gas was present, one can suspect:
A. Haemorrhagic bone cyst
B. Aneurysmal bone cyst
C. Traumatic neuroma
D. None of the above
61. Incision for removal of a palatal torus should be
placed:
A. In the midline
B. Paramedian
C. From crevices of teeth
D. As an envelope flap
62. Following method/methods can be used for treating
oral leukoplakia:
A. Excision
B. Cryotherapy
C. Fulguration
D. All of the above

59 A 60 c 61 A 62 D
Cyst/Tumour/Surgical Pathology 161

73. The commonest etiological factor in the development


of T M Joint ankylosis in children is:
A. Infection
B. Tetanus
C. Trauma
D. Rheumatoid arthritis
74. The preferred treatment of an giant cell lesion 2.5 em
in diameter, in the mandibular anterior region is:
A. Electrocauterization
B. Wide radical excision
C. Curettage
D. Block excision
75. The following cyst has the highest rate of recurrence:
A. Radicular cyst
B. Primordial cyst
C. Keratocyst
D. Dentigerous cyst
76. Radiographic examination is useful in detecting all
the following cysts, except the:
A. Follicular cyst
B. Nasopalatine cyst
C. Residual cyst
D. Nasoalveolar cyst
77. "Partsch No. II operation" is the name given to:
A. Enucleation
B. Marsupialization
C. Excision of a ranula
D. Combination of (A) and (B) above

73 c 74 c 75 c 76 0 77A
160 MCQs in Oral Surgery

68. A 14-year-old female patient presented with swelling


on (right) side of face in maxillozygomatic area. The
swelling has been slowly progressive in nature, ra-
diograph shows diffuse radiopaque mass involving
maxillary sinus and zygoma, with a typical 'ground
glass' appearance this is characteristic picture of:
A. Osteoma
B. Ossifying fibroma
C. Fibrous dysplasia
D. Osteosarcoma
69. The recommended treatment for fibrous dysplasia is:
A. Curettage for contouring
B. Resection en-block
C. Radiotherapy
D. Excision
70. The tumors which are poorly differentiated are:
A. Radiosensitive B. Radioresistant
C. Radioatropic D. Radiorefractive
71. A protein level between 5-11 g/dl In a cystic fluid is
indicative of:
A. A dentigerous cyst
B. A keratocyst
C. A radicular cyst
D. A mucocele
72. The Reharmann flap is used to :
A. Close an oronasal fistula
B. Gain access to the TM Joint
C. Close an oro-antral fistula
D. None of the above

68 c 69 A 70 A 71 c 72 A
Cyst/Tumour/Surgical Pathology 165

91. Fordyces disease is due to:


A. Hyperkeratosis
B. Capillary fragility
C. Aberrant sebaceous glands
D. Cholesterol deposits
92. Rodent ulcer or basal cell carcinoma is usually present
in the:
A. Intraorally lateral border of the tongue
B. Upper third of the face
C. Middle third of the face
D. Lower third of the face
93. Needle aspiration of a central bone lesion is useful
to:
A. Rule out a vascular lesion
B. To determine thickness of buccal plate
C. To diagnose traumatic bone cyst
D. Feel for root surfaces
94. 60-year-old women with a past history of adenocarci-
noma of the breast complaining of an ulcer on the
right lateral border of her tongue. Fractured cusp on
the mandibular right molar which exposes a sharp
margin of dental amalgum is found during routine
clinical examination. Treatment of choice is:
A. Immediate biopsy of the lesion
B. Restoration of the tooth with biopsy in 2 weeks if
necessary
C. Restoration of the tooth
D. Restoration of the tooth with biopsy in 2 months, if
the lesion does not heal

91 c 92 c 93 A 94 B

~ -~- --
162 MCQs in Oral Surgery

78. A 25-year-old lady reports to you with a diffuse, non


progressive and a non tender swelling of the right
maxilla, approximately 2 x 1.5 em, extending from the
canine to the first molar region. The X-ray shows a
"ground glass" appearance of the bone in the con-
cerned area. The surgical treatment will be:
A. Total excision of the lesion
B. Curettage of the area with extraction of the involved
teeth
C. Surgical cosmetic recontouring only
D. Surgical excision followed by radiotherapy
79. " Tt in the TNM staging of the oral malignant lesions
represents:
A. Carcinoma in situ
B. Tumour 2 em or less in greatest diameter
C. Tumour > 2 em but < 4 em in greatest diameter
D. Tumour > 4 em in greatest diameter
80. While performing a submandibular sialography the
cannula is Inserted in the following direction to the
Wharton's duct:
A. Upward and medical
B. Medial to lateral
C. Lateral to medial
D. Lateral and downward
81 . Factors leading to difficulties in cannulation while
performing sialography are, except:
A. Short or blunt bevel on tubing
B. Rough bevel on tubing
C. Too large a caliber tubing
D. Excessive dilation of the duct orifice

78 c 79 D 80 B 81 D
Cyst/Tumour/Surgical Pathology 169

110. Which of the following may cause a painless fluid filled


retention cyst appearing in the area of recent dental
treatment?
A. Allergic reaction to some agent employed
B. Infection
C. An injury obstructing a minor salivary gland
D. Failure of absorption of the anaesthetic gland
111 . A bluish soft and nontender swelling on the anterior
floor of the mouth is noted. It has been present for
several months. Most likely diagnosis is:
A. Mixed salivary gland tumor
B. An infected periodontal abscess
C. A retention cyst
D. Carcinoma of the floor of the mouth
112·. All of the following are seen in cleidocranial dysosto-
sis except:
A. Absence or hypoplasia of clavicles
B. Pseudo or false prognathism
C. Premature eruption of teeth
D. Bulging forehead and prominent cranial bones
113. One might see Cafe au lait spots in the following con-
ditions except:
A. Tuberous sclerosis
B. Sjogren's syndrome
C. von Reckling Hausen's disease
D. Albright's syndrome
114. Most likely cause of tender swelling in the subman-
dibular triangle is:
A. Obstruction to stenen's duct
B. Lipoma
C. Lymphadenopathy
D. All of the above

110 c 111 c 112 c 113 B 114 c


164 MCQs in Oral Surgery

86. Hormonal disturbance is the primary cause of jaw


deformity in patients suffering with:
A. Paget's disease
B. Achondroplasia
C. Acromegaly
D. None of the above
87. Ground glass appearance in X-rays may suggest the
patient is suffering ·with:
A. Hyperparathyroidism
B. Fibrous dysplasia
C. Paget's disease
D. None of the above
88. Which of the following condition associated with EB
virus?
A. Common cold B. Measles
C. Burkitt's lymphoma D. Poliomyelitis
89. 16-year-old patient has a painful swelling which is firm
in consistency in the mandibular premolar area. Teeth
near the swelling are vital. X-ray reveals a discrete
radiolucency in the premolar area. The next diagnos-
tic step is:
A. Aspiration
B. Biopsy
C. Complete RBC and WBC count
D. Bone scans
90. Bifid ribs, multiple radiolucent lesions of the jaws, mul-
tiple basal cell nevi and falx cerebri calcification are
found in:
A. Basal cell nevus syndrome
B. Sturge-Weber syndrome
C. Homers syndrome
D. Hereditary intestinal polyposis

86 c 87 8 88 c 89 A 90 A
Cyst/Tumour/ Surgical Pathology 171

120. A 60-year-old man complaining some loss of hearing,


some loss of visual acuity and enlargement of his
maxilla. The above findings suggest he is most prob-
ably suffering with:
A. Paget's disease
B. Multiple myeloma
C. Ossifying fibroma
D. Ameloblastoma
121. In routine radiological examination one notices a pear
shaped radiolucency located between the maxillary
lateral incisor and canine. Most probably it would be:
A. Nasoalveolar cyst
B. Median palatine cyst
C. Globulo maxillary cyst
D. Naso palatine duct cyst
122. Which of the following may cause fixation of a
nontender soft tissue mass to surrounding tissues?
A. Benign tumor
B. An inflammatory process
C. Malignancy
D. Lesion arising from bone
123. In routine radiological examination one found radi-
olucency near angle of the mandible and below the
inferior alveolar canal. Most probably it is:
A. A cyst
B. A develop mental defect
C. A cavernous hemangioma
D. Granuloma
124. Globulornaxillary cyst is a type of:
A. Dentigerous cyst B. Idiopathic
C. Fissural cyst D. Primordial

120 A 121 c 122 c 123 B 124 c


166 MCQs in Oral Surgery

95. Of the following regions where ameloblastoma fre-


quently occur?
A·. Mandibular molar region
B. Maxillary molar region
C. Mandibular premolar region
D. Maxillary premolar region
96. During routine radiographic examination one found
pear shaped radiolucency maxillary canine and lat-
eral incisors region. The teeth are vital most probable
diagnosis is:
A. Radicular cyst
B. Incisive canal cyst
C. Globulomaxillary cyst
D. Primordial cyst
97. The best treatment for cementoma is:
A. No treatment
B. Endodontic treatment
C. Extraction of the teeth
D. Resection of the involved area
98. During the examination of radiograph of a 60-year-
old male patient, one finds periapical radiolucencies
near mandibular incisors. The diagnosis include:
A. Periapical cyst B. Early cementomas
C. Paget's disease D. A and B

99. In the clinical evaluation, the most significant finding


of a parotid mass may be accompanying:
A. Slow progressive painless enlargement
B. Nodular in consistency
C. Submental and preauricular lymphadenopath y
D. Facial paralysis

95 A 96 c 97 A 98 0 99 0
Cyst/Tumour/Surgical Pathology 173

130. Among the following which is fissural cyst and


entirely located in soft tissue?
A. Nasoalveolar cyst
B. Globulomaxillary cyst
C. Median alveolar cyst
D. Primordial cyst
131. Osteitis deformans is the condition in:
A. Which roots have hypercementosis
B. Which anodontia seen
C. Which supernumerary roots are more common
D. None of the above

130 A 131 A
168 MCQs in Oral Surgery

105. Nikolsky's sign helps in the diagnosis of:


A. Herpetic stomatitis
B. Erythema multiforme
C. Pemphigus vulgaris
D. Drug allergy
106. Which of the following is most common site for
sialoliths?
A. Parotid duct
B. Submandibular duct
C. Minor salivary gland
D. Parotid gland
107. Of the following where sialoliths are most common?
A. Parotid gland
B. Parotid duct
C. Submandibular duct
D. Sublingual gland
108. Of the following which is true ? "Port wine stain" is a:
A. A type of stain occasionally used by histopathologist
B. Commonly used contrast media for better image by
MRI
C. A type of hemangioma
D. None of the above are correct
109. Which of the following is correct regarding Bowen's
disease of the skin?
A. A type of dermatitis, some times affects the eye
B. A pre malignant condition/lesion
C. Is due to some abnormalities in sVveat gland
D. Occur in mercury poisoning

105 c 106 8 107 c 108 c 109 8


Facial Space Infection/Osteomyelitis 175

5. If one treats an abscess with antibiotics without I and


D it may cause:
A. Non-suppurative inflammatory reaction
B. Formation of antibioma
C. Healing with scar formation
D. None of the above
6. Of the following which organism is commonly asso-
ciated with sub acute bacterial endocarditis of dental
origin?
A. Staphylococcus aureus
B. Streptococcus viridans
C. Beta hemolytic streptococcus
D. Lactobacillus acidophilus
7. Usually acute pyogenic bacterial infection produce
A. Neutropenia
B. Lymphocytic leukocytosis
C. Neutrophilic leukocytosis
D. Eosinophilia
8. Koplik's spots are oral manifestations of
A. Smallpox
B. Rubeola
C. Rubella
D. Scarlet fever
9. Classical Ludwig's angina has bilateral involvement
of the following spaces except:
A. Submandibular
B. Sublingual
C. Submental
D. Lateral pharyngeal

5 B 6 B 7 c 8 B 9 D
170 MCQs in Oral Surgery

115. Of the following which denotes the inflammation of


the salivary duct?
A. Pulmonary embolism
B. Acute myocardial infarction
C. Thrombophlebitis
D. All of the above
116. A patient is under oral anticoagulants such as war-
farin therapy. Most probably he have:
A. Pulmonary embolism B. Acute sialadenitis
C. Chronic sialadenitis
D. Sialodochitis
117. Of the following which is characteristic of the trau-
matic (hemorrhagic) bone cyst?
A. It causes root resorption
B. Rarely expands cortices or displaces teeth
C. Devitalized involved teeth
D. Is usually found in the maxilla
118. Ameloblastoma is commonly seen in which of the fol-
lowing areas?
A. Antrum and floor of the nose
B. Symphysis area of the mandible
C. Molar area of the ramus of the mandible
D. Mandibular premolar area
119. In a routine examination of an IOPA one noticed the
following :
1. Ground glass appearance of bone
2. Complete loss of lamina dura
3. Loss of trabecular pattern of bone
4. The most probable diagnosis is
A. Hypercalcemia
B. Hyperparathyroidism
C. Fibro-osseous lesion
D. Hypocalcemia

115 D 116 D 117 8 118 c 119 8


Facial Space Infection/Osteomyelitis 177

15. Fascial spaces are filled by:


A. Loose connective tissue
B. Elastic fibres
C. Loose adipose tissue
D. Dead space
16. The characteristic features of infection of masticator
space is:
A. Swelling B. Draining pus intraorally
C. Trismus D. High grade fever

17. The infections of masticator space do not enter into


neck because:
A. The fascia is tenaciously adherent to mylohyoid line
B. The fascia is firmly adherent to periosteum lower
border of mandible
C. Before it reaches the neck it follows path of least
resistance to open extraorally or intraorally
D. Masticator space is not continuous with spaces in neck
18. The infection of masticator space can spread to
except:
A. Temporal pouches
B. Lateral pharyngeal space
C. Sublingual space
D. Submandibular space
19. Swellings of masticator:- space and lateral pharyngeal
space are similar. The distinctive difference is that
masticator space infection:
A. Is of dental origin
B. Is not pushed towards the midline
C. Is more diffuse and visible from outside
D. Has a tendency to spread to temporal pouches

15 A
'--------~ ·
16 c 17 B 18 D 19 B I
172 MCQs in Oral Surgery

125. Radiographs of the maxillary central and lateral inci-


sors disclose a heart shaped radiOlucency in the mid-
line. Teeth next to the radiolucency are vital, this is
most probably:
A. Nasopalatine duct cyst
B. Globulomaxillary cyst
C. Nasoalveolar cyst
D. Median palatine duct cyst
126. Fibrous dysplasia can be treated by:
A. Irradiation of the lesion
B. Surgical excision
C. Conservative surgery
D. Removal of the adjacent teeth
127. Among the following which is most frequent compli-
cation of subapical orthodontic surgery?
A. Non union of segments
B. Devitalization of teeth
C. Avulsion effect
D. Vertical frac ture of the roots
128. Ameloblastoma of the jaw can best be treated by:
A. A excision
B. Surgical resection followed by cauterisation
C. Enucleation
D. Irradiation
129. Which of the following lesion has no epithelial lining?
A. Nasopalatine cyst
B. Nasolabial cyst
C. Aneurysmal bone cyst
D. Follicular cyst

125 A 126 c 127 B 128 B 129 c


Facial Space Infection/Osteomyelitis 179

24. In Ludwig's angina the incision should be placed deep


uptill:
A. Mylohyoid muscle
B. Anterior belly of digastric
C. Geniohyoid
D. Mucous membrane of floor of mouth
25. Infections of lateral pharyngeal space travel usually
from:
A. Temporal pouches
B. Masticator space
C. Sublingual space
D. None of the above
26. Infections of lateral pharyngeal space are life-threat-
ening because there may be dangers of:
A. Thrombosis of lJV
B. Erosion of ICA
C. Oedema of larynx
D. All of the above
27. Infections from lateral pharyngeal space can traverse
to: ·
A. Anterior mediastinum
B. Middle mediastinum
C. Posterior mediastinum
D. Only superior mediastinum
28. Infections from submandibular space and sub-men-
tal space usually traverses to:
A. Anterior mediastinum
B. Middle mediastinum
C. Posterior mediastinum
D. Only superior mediastinum

24 D 25 B 26 D 27 c 28 A
FASCIAL SPACE
INFECTION/
OSTEOMYELITIS

1. The hard, firm condition of the tissues is termed:


A. Cellulitis
B. Purulence
C. Coagulum
D. Abscess
2. Treatment at the first appointment for patient with
cellulitis should include:
A. Antibiotics and heat only
B. Extraction of the offending tooth
C. Antibiotics, heat and fluids
D. Incision and drainage
3. The patient with cellulitis after giving heat, antibiot-
ics, and fluids returns 24 hours later. On palpation,
the area is soft painful to touch, and the tissue re-
bounds when palpated. This condition is termed:
A. Abscess B. Cellulitis
C. Resorption D. Induration
4. Treatment for the patient with abscess should include:
A. Aspiration
B. Antibiotics and heat only
C. Incision and drainage
D. Antibiotics, heat and fluids

1 A 2 c 3 A 4 c
Facial Space Infection/Osteomyelitis 181

34. Dissecting subperiosteal abscess develops:


A. Immediately after 3rd molar extraction on lingual side
B. Several weeks later and distant to site of 3rd molar
extraction
C. In association of post extraction infection in buccal
area of extracted 3rd molar
D. When extensive dissection is done while extracting
an impacted tooth
35. An acute alveolar abscess should be treated with:
A. First -antibiotics for three days and then incision and
drainage
B. Incision and drainage with broad spectrum antibiotic
C. Broad spectrum antibiotics and analgesics
D. Antibiotics and proteolytic drugs as chymotrypsin
36. Osteomyelitis begins as an inflammation of:
A. Cortical bone
B. Periosteum
C. Medullary bone
D. Periosteum and inner cortex
37. Which of the following conditions are susceptible to
osteomyelitis?
A. Paget's disease
B. Fibrous dysplasia
C. Radiation
D. All of the above
38. Osteomyelitis is more common in:
A. Maxilla
B. Mandible
C. Zygoma
D. Nasal complex

34 B 35 B 36 c 37 0 38 B
176 MCQs in Oral Surgery

10. Osteoradionecrosis occurs due to damage to which


of the following structures?
A. Blood vessels
B. Salivary glands
C. Muscle
D. Nerves
11. In a 19-year-old patient with a swelling over the left
angle of the mandible, temperature 38° C and nega-
tive history of trauma, one should suspect:
A. Spontaneous fracture of the mandible
B. Pericoronal infection
C. Parotid tumor
D. Submaxillary gland tumor
12. Infection from a maxillary first molar spreads most
often to which space:
A. Infratemporal
B. Retropharyngeal
C. Submandibular
D. Buccal
13. Cavernous sinus thrombosis following extraction of
acutely infected maxillary anterior teeth is most often
due to spread of infection along:
A. The anterior facial, angular, ophthalmic veins
B. The pterygoid plexus and inferior ophthalmic vein
C. The pterygoid plexus and superior ophthalmic vein
D. Not likely by any of these routes
14. The greatest deterrent to normal wound healing is:
A. Trauma
B. Infection
C. Metabolic disease
D. Nutritional deficiency

10 A 11 B 12 D 13 A 14 B
Facial Space Infection/Osteomyelitis 183

44. In treating osteomyelitis with hyperbaric oxygen:


A. 30% oxygen is used at 1 atrn
B. 70% oxygen is used at 2 atm
C. 100% oxygen is used at 3 atm
D. 80% oxygen is used at 2 atrn
45. A40-year-old patient presented with multiple extraoral
sinuses with yellowish discharge and with history of
intermittent remission after antibiotics treatment two
months ago. The disease started after extraction of
2nd mandibular premolar. It is suggestive of:
A. Tubercular osteomyelitis
B. Actinomycosis
C. Subperiosteal Garre's osteomyelitis
D. Dissecting subperiosteal abscess
46. Treatment of "alveolar osteitis" consists of:
A. Curettage of the socket to induce fresh bleeding in
the socket
B. Prescription of antibiotics specific for gram-negative
microorganisms
C. Gentle removal of debris, irrigation of the socket and
placement of an obtundent dressing
D. Irrigating the socket with 5% povidone-iodine
47. Surgical management of Ludwig's angina includes:
A. Incision and drainage
B. Incision and decompression
C. Drainage through closed dwelling catheters
D . None of the above

44C 45 B 46 c 47 B
178 MCQs in Oral Surgery

20. Incision and drainage of masticator space should be


attempted:
A. At region anterior to masseter muscle
B. lntraorally from buccal sulcus
C. Extraorally in subangular region
D. From pterygomandibular raphe
21. Infections from mandibular 1st molar would travel to:
A. Submandibular space
B. Sublingual space
C. Masticator space
D. Digastric space
22. The major structures present in the submandibular
space are:
A. Deep part of submandibular gland, branches of facial
artery, lingual nerve
B. Superficial part of submandibular gland, branches of
facial artery and lingual nerve
C. Superficial part of submandibular gland, branches of
facial artery, mylohyoid nerve
D. Submandibular d uct, lingual nerve and hypoglossal
nerve
23. In Ludwig's angina the classical sign is:
A. Tongue is raised and falls back causing respiratory
embarrassment
B. That submandibular, sublingual and submental
spaces are involved though tongue may not be raised
C. That s ubmandibular, sublingual and sub-mental
spaces are involved bilaterally
D. Board-like brawny induration of mandible with
tongue falling back and causing respiratory
embarrassment

20 c 21 B 22 c 23 c
Facial Space Infection/Osteomyelitis 185

52. Infection from a horizontally impacted mandibular


third molar will initiaUy involve the:
A. Submandibular space
B. Sublingual space
C. Pterygomandibular space
D. Masseteric space
53. While draining a submandibular abscess, the surgeon
should:
A. Increase the overlying tissues with a no. 10 blade
B. Aspirate the contents, and put in a vacuum drain to
evacuate the remaining pus
C. Use the Hilton's method to completely evacuate the
pus
D. Always perform the procedure under general
Anaesthesia
54. In Marx's protocol for the management of osteoradi-
onecrosis of the mandible using hyperbaric oxygen
if tissue dehiscence occurs after the initial 30 dives
and then:
A. The mandible is resected and 30 more dives are given
B. The mandible is resected and the dives are
discontinued
C. The full course of 60 dives is given followed by the
resection of mandible
D. The full course of 60 dives is given followed by
additional20 dives after 10 weeks
55. According to BIRN's hypothesis the severe pain as-
sociated with dry socket is due to:
A. Release of plasmin
B. Release of kinin from the degenerative clot
C. Thermal irritation of the exposed nerve ending of the
alveolar bone
D. BandC

52 c 53 c 54 A 55 D
180 MCQs in Oral Surgery

29. Infections which travel from masticator space to pa-


rotid space are very painful because:
A. Facial nerve is irritated
B. The capsule of parotid does not give way for the
developing infection to spread
C. Auriculotemporal nerve is irritated by infection
D. None of the above
30. While giving a inferior alveolar nerve block, infection
is transposed to :
A. Pterygopalatine fossa
B. Pterygomandibular space
C. Submandibular space
D. Masticator space
31 . While giving posterior superior alveolar nerve block,
infection may be instituted into:
A. Pterygomandibular space
B. Infratemporal fossa
C. Temporal pouches
D. Pterygopala tine fossa
32. A patient, presented with ophthalmoplegia and signs
of meningitis after extraction of upper central incisor,
could be diagnosed as due to:
A. Tumour of pituitary
B. Tuberculous meningitis
C. Cavernous sinus thrombosis
D. No relation
33. The diagnostic sign/s which Eagleton characterised
for cavernous sinus thrombosis is/are:
A. Known site of infection
B. Paresis of III, IV, VI nerves
C. Proptosis of eye and (B)
D. All of the above

29 B 30 A 31 A 32 c 33 D
Facial Space Infection/Osteomyelitis 187

61. Of the following which is not a component of masti-


cator space?
A. Submandibular space
B. Sub masseteric space
C. D~ep temporal pouch
D. Superficial temporal pouch
62. Dental infection from the mandibular first molar
spreads to the:
A. Submental space
B. Buccal vestibular space
C. Infratemporal space
D. Pterygomandibular space
63. Specimen for bacteriologic examination and diagno-
sis should be collected:
A. Prior to the development of the symptoms
B. One or two days after the antibiotic treatment
C. During the acute stage of the disease
D. During the convalescence
64. Local factors that predispose bone to osteomyelitis
are related mainly to:
A. Reduced autibody formation
B. Potent endotoxins
C. Reduced blood supply
D. Increased lymphatic circulation
65. Osteomyelitis of the TMJ can be treated by:
A. Condylectomy
B. Irradiation
C. Incision and drainage, if needed condylectomy
D. None of the above

61 A 62 B 63 c 64C 65 c
182 MCQs in Oral Surgery

39. Osteomyelitis is caused most commonly by:


A. Streptococcus B. Staphylococcus
C. M. tuberculosis D . E. coli
40. In osteomyelitis how much bone should have been
destroyed before It manifests radiologically:
A. 10-12% B. 15%
C. 30-60% D. 80%
41 . Moth eaten appearance so characteristically seen in
radiographs of osteomyelitis is due to:
A. Presence of sequestrum
B. Enlargement of medullary spaces
C. Reduced medullary spaces
D. Narrowing of Volkmann's canals
42 . Treatment of chronic osteomyelitis consists of:
A. Culture sensitivity and prolonged antibiotic therapy
B. Culture sensitivity with antibiotic therapy and
hyperbaric oxygen therapy
C. Sequestrectomy, surgical exploration and prolonged
antibiotic therapy after culture sensitivity
D. Sequestrectomy, antibiotics after sensitivity and
hydrocortisone therapy
43. Saucerisation as a surgical treatment for osteomyelitis
connotes:
A. Complete removal of decayed bone with primary
closure of wound
B. Trimming or excision of margins of necrotic bone
overlying focus of osteomyelitis and allowing
secondary healing
C. Creating a saucer shaped defect by excision of the
defect with primary closure
D. None of the above

39 B 40 c 41 B 42 c 43 B
Facial Space Infection/Osteomyelitis 189

71. Pterygomandibular space infection may lead to in-


volvement of the:
A. Submaxillary space
B. Parapharyngeal space
C. Infratemporal space
D. All of the above
72. Which of the following sign/symptom does not sug-
gest postoperative infection?
A. Firm, tender swelling
B. Localized pitting oedema
C. Temperature elevation
D. Increasing or persisting pain
73. After I and D of an abscess, the infectious process
has failed to regress inspite of the patient being on
high doses of an antibiotic, It would be wise to:
A. Repeat culture and sensitivity tests
B. Insert a large drain
C. Augmenting antibiotic action by administration of
parenteral proteolytic enzymes
D. Debride and irrigate the area with a fibrinolylic agent
74. Teeth in line of fire means:
A. Teeth in the area of planned therapeutic radiation
B. Teeth in the line of fracture
C. Teeth within the cancerous lesion
D. None of the above
75. Scrub technique refers to the proper method of scrub-
bing the:
A. Operator's hands and forearms before donning gown
and gloves
B. Patients body surface before surgical incision
C. Operating room after each infected operation
D. All of the above

71 D 72 8 73 A 74 B 75 A
184 MCQs in Oral Surgery

48. When a canine space infection has proceeded from


cellulitis to fluctuance which is apparent visibly and
digitally and the patient is in great pain, the initial treat-
ment should be:
A. Administering parentral antibiotics
B. Application of hot packs over the region
C. Incision and drainage
D. Giving an infraorbital nerve block to provide pain
relief
49. Management of ecchymosis following oral surgery
includes:
A. Application of cold packs over the area
B. Administration of steroids and enzymes
C. Aspiration with a w ide bore needle
D. None of the above
50. The major local factor that predisposes mandibular
bone to osteomyelitis is:
A. Reduced cellular proliferation
B. Reduced lymphatic circulation
C. Reduced vascular supply
D. Reduced antibody formation
51. Complications of lateral pharyngeal space infection
include:
A. Thrombosis of the internal jugular vein
B. Erosion of the internal carotid artery
C. Respiratory paralysis
D. All of the above

48C 49 A 50 c 51 0
Is I TMJ AND MAXILLARY
SINUS

1. Paranasal sinus view (or) Water's view is advised for


a sinusitis patients. Features of sinusitis include:
A. Fluid levels.
B. Clouding of the antra
C. Clouding and fluid levels
D. Erosion of the bone
2. Of the following which method is commonly used to
treat ankylosed TMJ?
A. Steroid injection into the joint.
B. Muscle relaxants and occlusal equilibration
C. Surgical procedure involving condylectomy
D. Systemic steroids
3. Of the following which is not a structural element of
the TMJ?
A. Joint cavities.
B. Condylar process
C. Sigmoid notch
D. Articular disc
4. The Al-kayat and Bramley approach to the TM joint is
a modification of the:
A. Hemicronal approach
B. Retroauricular approach
C. Preauricular approach
D. Risdon's approach

1 c 2 c 3 c 4 c
186 MCQs in Oral Surgery

56. Which is the potential primary hazard to a patient un-


der propylthiouracil therapy and whose laboratory
data indicates hematocrit value of 45 percent, reverse
neutropenia, reduced myeloblasts in bone marrow,
clotting time of 8 min?
A. Thyroiditis B. Haemorrhage
C. Hypoxia D. Infection
57. The following are the characteristic features of
Ludwig's angina:
A. Raised tongue
B. Bilateral submandibular swellings
C. Dysphagia
D. All of the above
58. A teenage girl presents with pain, trismus swelling
associated with a partially erupted lower third molar.
The most likely diagnosis is:
A. Peritonsillar abscess
B. Pericoronitis
C. Ameloblastoma
D. Fracture of the mandible near angle region
59. Which ofthe following may result from acute pyogenic
bacterial infections?
A. Lymphopenia
B. Leukopenia
C. Lymphocytosis
D. Leukocytosis
60. After radiotherapy bone is more prone for infection.
This is probably related to:
A. Decalcification
B. Anaemia
C. Invasion by m alignant cells
D. Endarteritis of small blood vessels

56 D 57 D 58 B 59 D 60 D
TMJ and Maxillary Sinus 193

10. In surgical management of TMJ ankylosis, one can


encounter excessive bleeding from:
A. Inferior alveolar artery
B. Internal maxillary artery
C. Pterygoid plexus of veins
D. All of the above
11. In unilateral TMJ ankylosis the chin is deviated to:
A. The affected side
B. The contralateral side
C. No deviation seen
D. Side where growth is occurring
12. In a bilateral TMJ ankylosis case the chin would be
deviated to:
A. Side of intense ankylosis
B. Side where more movement is present
C. No deviation
D. None of the above
13. Dautrey procedure is a treatment modality for:
A. TMJ clicking
B. TMJ dislocation
C. TMJ arthritis
D. TMJ ankylosis
14. A patient complains of pain in TMJ area on mas-
tication, his muscles of mastication are tender and
an audible click is there, these features are charac-
teristic of:
A. MPDS
B. Traumatic subluxation
C. Rheumatoid arthritis
D. Rheumatic arthritis

10 D 11 A 12 c 13 B 14 A
188 MCQs in Oral Surgery

66. Among the following which anaerobic organism is


frequently responsible for oral and facial infection?
A. Staphylococcus aureus
B. Staphylococcus albus
C. Bacteroids fragilis.
D. Streptococcus viridans
67. Among the following which variable has the greatest
significance in antibiotic management of dental infec-
tion?
A. Susceptibility of the organism
B. Route of administration
C. Antigenicity of the antibiotics
D. Specific antibody titre of the host
68. All the spaces mentioned below are involved in clas-
sic Ludwig's angina except:
A. Submandibular
B. Space of the body of the mandible
C. Submental
D. Sub lingual
69. To drain pus from an abscess of the pterygomandibu-
lar space from an intraoral approach, the muscle most
likely to be incised is the:
A. Buccinator B. Temporal
C. Medial pterygoid D. Lateral pterygoid
70. Contamination from a patient with a recent serum
hepatitis?
A. Wear gloves
B. Wear a mask
C. Sterilize all instruments and drapes after treatment
D. All of the above

66 c 67 A 68 B 69 A 70 D
TMJ and Maxillary Sinus 195

19. A patient who reports with bilateral dislocation of TMJ


should be managed:
A. Manually without LA
B. Manually with LA
C. Under GA only
D. Surgically under GA
20. The management of recurrent TMJ dislocation is:
A. High condylotomy
B. Eminectomy
C. Dautrey procedure
D. All of the above
21. A patient of MPDS with typical psychosomatic aetiol-
ogy should be prescribed:
A. Carbamethaxamol
B. Diazepam
C. Fomentation and cold compresses
D. All of the above
22. Berger's flap for OAF closure utilises a:
A. Palatal flap
B. Buccal flap
C. Only a mucosal mobilisation
D. None of the above
23. A palatal flap has high success rate in management
of OAF because:
A. Abundance of tissue
B. Branch of palatal artery is also mobilized
C. It is resistant to infection
D. Of fatty layer there are less chances of tear

19 B 20 D 21 D 22 B 23 B
190 MCQs in Oral Surgery

76. Acute pyogenic bacterial infections produce:


A. Leukopenia B. Lymphopenia
C. Neutropenia D. Leukocytosis
77. The tendency to infection of bone after radiotherapy
is probably related mainly to:
A. Anaemia
B. Decalcification
C. Invasion by malignant cells
D. Endarteritis of small blood vessels
78. A patient has received a tumoricidal course of cobalt
therapy for a lesion of the pharynx. His mandibular
first molar has developed a periapical abscess related
to caries. This tooth would be treated by:
A. Administering an antibiotic and extracting
B. Performing root canal therapy if at all possible
C. Prescribing an analgesic compound plus an antibiotic
D. Incising the patient to the radiotherapist
79. Fourty-eight hours following the removal of a left im-
pacted mandibular third molar, the patient returns to
your office complaining of moderate pain radiating to
the left ear. His emperature is 99°F and swelling is
minimal. The most probable diagnosis is:
A. Traumatic injury to the inferior alveolar nerve during
the injection procedure
B. Postoperative infection involving the masticator
fascial space
C. Postoperative infection involving the parotid space
D. Postextraction alveolitis

76 D 77 D 78 B 79 D
TMJ and Maxillary Sinus 197

29. The treatment of parotid abscess is:


A. Antibiotics only
B. Dilation of duct and (A)
C. Incision parallel to facial nerve branches and drainage
with (A)
D. Fomentation with (A)
30. The stone in anterior submandibular salivary gland
duct should be removed by placing the incision:
A. Medial to plica sublingualis
B. Lateral to plica sublingualis
C. Never in anterior region
D. Just superficially in 2nd-3rd molar region
31. Once the stone in submandibular salivary gland duct
has been identified the incision should be placed:
A. Longitudinally and duct sutured
B. Transversely and duct sutured.
C. Longitudinally and surgical wound closed without
suturing the duct .
D. Transversely and surgical wound closed without
suturing the duct
32. While removing a submandibular gland one encoun-
ters:
A. Facial artery, facial vein, cervical branch of facial nerve
and lingual nerve
B. Facial artery, facial vein, cervical branch of facial nerve
only
C. Facial artery, facial vein, hypoglossal nerve only
D. Facial artery, facial vein, marginal mandibular branch
of facial nerve only

29 c 30 A 31 c 32 A
192 MCQs in Oral Surgery

5. Mention the blood supply to the flap that has been


used to close an oroantral fistula in the area of tooth:
A. Nasopalatine
B. Greater palatine
C. Posterior superior alveolar
D. Facial
6. .The treatment of unilateral TMJ ankylosis in a 8-year-
old child would be:
A. Simple gap arthroplasty
B. Condylectomy
C. Gap arthroplasty with costochondral grafting
D. High condylotomy with costochondral grafting
7. The ideal surgical approach to TMJ ankylosis is:
A. Endaural
B. Submandibular
C. Postauricular
D. Preauricular
8. Interposition of temporal muscle and fascia in treat-
ment of TMJ ankylosis is advocated:
A. To prevent reankylosis
B. To prevent erosion of glenoid fossa due to movement
of ramal end
C. To provide soft pad for easy movement of ramal end
D. None of the above
9. Which of the following is/are cause/s of TMJ ankylo-
sis?
A. Trauma
B. Middle ear infection
C. Rheumatoid arthritis
D. All of the above

5 B 6 c 7 D SA 9 D
TMJ and Maxillary Sinus 199

38. Early movement following surgery for temporoman-


d ibular joint ankylosis is:
A. Harmful B. Desirable
C. Unimportant
D. Indicated only when ankylosis is limited to one side
39. To reduce a dislocation of the mandible, the move-
ment employed is:
A. Downward and backward
B. Downward and forward
C. Upward and backward
D." Upward and forward
40. The most common disorder causing pain about the
masticatory apparatus including the TMJ , is:
A. Myofacial-pain-dysfunction
B. Trigeminal neuralgia
C. Degenerative arthritis
D. Traumatic arthritis
41. Radiographic features of sinusitis include:
A. Fluid levels
B. Erosion of bone
C. Clouding of the antra
D. Cloud ing and fluid levels
42. Extraction of a maxillary second molar has resulted
in a perforation of the maxillary antrum 0.5 em in di-
ameter. An acceptable procedure would be:
A. Caldwell-Luc procedure
B. Creation of a nasal antrostomy w indow for proper
aeration and drainage
C. Smoothing bone margins of the socket and placement
of sutures across the socket
D. Insertion of iodoform gauze packing into the socket
to allow healing by second intention

38 B 39 A 40 A 41 D 42 c
194 MCQs in Oral Surgery

15. The hypertonic saline or sclerosing solution is used


for conservative management ofTMJ sub-luxation and
dislocation. These injections are given:
A. In superior compartment
B. In inferior compartment
C. Paracapsular
D. In the articular disc
16. In TMJ osteoarthritis which medicament is injected
in TMJ?
A. Sodium morrhuate
B. Hydrocortisone
C. Sodium salicylate
D. Hypertonic saline
17. Myofacial pain dysfunction syndrome can be precipi-
tated by:
A. High filling or malocclusion
B. Psychogenic factors
C. Bruxism
D. All of the above
18. Arthroscopy is a technique by which:
A. The inside of joint can be seen from outside but for
treatment open surgery is required
B. The inside of joint can be seen and operated from
outside, without any open surgery
C. Dye is injected into the joint and serial radiographs
are taken to see movement of disc in the joint
D. Dye is injected into the joint and outline of joint cavity
is delineated to see any bony erosion spur formation,
etc

15 c 16 B 17 D 18 B
TMJ and Maxillary Sinus 201

49. A fractured mandibular condyle is displaced forward


and medially by the action of the following muscle:
A. Temporalis
B. External pterygoid
C. Internal pterygoid
D. Masseter
50. Following an oral Sl;Jrgical procedure the surgical site
is covered with a gauze dampened with saline be-
cause the:
A. Saline promotes haemostasis at the site
B. Saline decreases the tendency of the clot to become
embedded in the gauze mesh
C. Saline has localised anti-inflammatory action
D. Saline accelerates the healing of the wound
51. "Drum splints" are used in the management of
patients with:
A. Myofacial pain dysfunction syndrome
B. Non displaced mandibular angle fractures
C. Fibrous TMJ ankylosis
D. Atypical facial pain
52. The amount of dye that can be injected into ductal
system of the parotid glands during sialography var-
ies between:
A. 0.5 to 0.75 ml B. to 1.5 ml
C. 1.5 to 2.0 ml D. to 2.5 ml
53. The following agent.is now commonly used for chemi-
cal capsulorrhaphy in patients with recurrent dislo-
cation of the TMJ:
A. Carnoy's solution
B. Sodium psylliate
C. Sodium tetradecyl sulphate
D. Sodium mourrhate

49 B 50 B 51 A 52 B 53 c
196 MCQs in Oral Surgery

24. If nasal antrostomy is planned after OAF closure open-


ing should be made in:
A. Middle meatus
B. Inferior meatus above nasal floor level
C. Inferior meatus at nasal floor level
D. Just above middle concha
25. OAF should never be closed if:
A. Palatal mucosa is deficient
B. Signs of infection are present
C. Opening is too large
D. Patient is to have a complete denture
26. If on removing a tooth, one realises that a large OAF
has been formed:
A. Immediate primary closure should be done
B. Closure should be done after 7 days
C. The sinus should be irrigated, lavaged for 2/3 days
then closed
D. None of the above
27. Nasal decongestants are prescribed in management
of OAF to:
A. Allow drainage
B. Shrink antral lining
C. Prevent infection
D. Make breathing easier
28. Sialoangiectasis denotes:
A. Salivary gland and duct system as vastly dilated
B. A sialolith is present
C. A stricture in duct is present
D. Chronic inflammation of salivary gland

24 c 25 B 26 A 27 '8 28 A
TMJ and Maxillary Sinus 203

58. The clinical sign of acute maxillary sinusitis are all


except:
A. Mucopurulent exudates
B. Tenderness over the anterolateral sinus wall
C. Tenderness to percussion of maxillary molar teeth
D. Loss of vitality of maxillary molar teeth
59. Among the following which is best radiograph to ex-
amine maxillary siriuses?
A. Orthopantamogram
B. Intraoral periapical films
C. Occlusal view
D. Water's view
60. Commonly advised extra oral radiograph to see the
maxillary sinuses is:
A. AP skull
B. Submentovertex
C. Occipitomental
D. Lateral skull
61. Hypertrophy of the mandibular condyle may cause the
following:
A. An anterior cross bite
B. Ipsilateral posterior open bite
C. Unilateral class III malocclusion
D. All of the above
62. Of the following which is most common disorder caus-
ing pain about the ·masticatory apparatus including
the TMJ?
A. Traumatic arthritis
B. Trigeminal neuralgia
C. Myofacial pain dysfunction syndrome
D. Degenerative arthritis

58 D 59 D 60 c 61 D 62 c
198 MCQs in Oral Surgery

33. The early manifestation of sialadenitis on a sialogram


is:
A. Terminal acini are dilated
B. The acinar system is dilated
C. The ductal system is dilated
D. Constriction of ductal and acinar system
34. Warthin's tumour is:
A. Malignant parotid tumour
B. Benign submandibular tumour
C. Benign parotid tumour
D. Any tumour of salivary glands which can be benign
or malignant
35. Mucoepidermoid tumour is:
A. Malignant
B. Benign
C. Squamous cell tumour of salivary gland
D. Same as adenocarcinoma
36. Cylindroma:
A. Is malignant tumour
B. Is slow growing but metastases
C. Shows extensive invasion
D. All of the above
37. Ankylosis of the temporomandibular joint is best
treated via:
A. X-ray therapy
B. Arthroplasty
C. Cortisone injection
D. Exercises
E. Antibiotic therapy

33 A 34C 35 A 36 D 37 B.
MAXILLOFACIAL
INJURY

1. Of the following which view is best to visualize


zygomatic arches?
A. Submentovertex or jug handle view
B. Occipitomental view
C. Orthopantamogrum
D. Skull P A view
2. Open reduction and internal fixation of fracture frag-
ments in the older patients is risky because:
A. Of their old age. they less likely tolerate the major
procedure under general anesthesia
B. Fixation is difficult because bone will become more
dense as age advances
C. Delayed or non-union may occur because of over all
decrease in reparative abilities of the body
D. None of the above are correct
3. Of the following which Is the best method of treating
a green stick fracture of the mandible?
A. Allow normal masticatory movements
B. Bringing the teeth into occlusion with interdental
wiring
C. Extraskeletal fixation
D. None of the above

1 A 2 D 3 B
200 MCQs in Oral Surgery

43. The most common cause of temporomandibular joint


ankylosis is:
A. Infection B. Rheumatoid arthritis
C. Trauma D. Congenital malformations
44. The "hanging drop" appearance in the maxillary si-
nus radiograph indicates:
A. A nasal polyp
B. A blow out# of the orbit
C. A radiographic artifact
D. An antrolith
45. The most critical period for a child after having a
condylar fracture would be the age group between:
A. Five to ten B. Ten to twelve
C. One to five D. Twelve to fifteen
46. The following movement is used to reduce an ante-
rior dislocation of the condyles:
A. Downward, forward and upward
B. Downward, backward and upward
C. Upward, forward and medial
D. Upward, backward and lateral
47. The Denker's procedure is an approach to the:
A. Pterygomandibular space
B. Ethmoid sinus
C. Maxillary sinus
D. Medial wall of the orbit
48. Hydrocortisone acetate is injected in a painful arthritic
TM Joint to:
A. Increase the blood supply
B. Lubricate the synovial lining
C. Anaesthetise the nerve supply
D. Decrease the inflammation

43 c 44B 45 c 46 B 47 c 48 D
Maxillofacial Injury 207

10. Placing a nasal pack during nasal bleeding and CSF


leak carry the danger of:
A. Fracture of ethmoidal plates
B. Redirecting the CSF to oropharynx
C. Meningitis
D. Redirecting CSF to orbit
11. Hypovolemic shock develops after loss of:
A. 10% blood B. 20% blood
C. 30% blood D. 40% blood
12. Facial wounds can be considered for primary closure
when they report within:
A. 24 hrs B. 72 hrs
C. 36 hrs D. 48 hrs
13. Failure of primary suturing occurs in facial wounds
when:
A. Fine silk had not been used
B. Catgut has been used
C. Dead space develops
D. Continuous suturing is done
14. Diplopia would result if fracture line around
zygomatlcofrontal suture passes:
A. Below the Whitnall's tubercle
B. Above the Whitnall's tubercle
C. Through zygomaticofrontal suture
D. Tearing the periosteum of orbital surface of zygomatic
bone
15. Battle's sign is associated with:
A. Fracture zygoma
B. Fracture anterior cranial fossa
C. Fracture middle cranial fossa
D. Fracture nasoethmoid

10 c 11 D 12 A 13 c 14 B 15 c
202 MCQs in Oral Surgery

54. According to Sawhney's classification of TM joint


ankylosis, type II cases present:
A. A bridge of bone between the remus and the
zygomatic arch
B. A joint entirely replaced by a mass of bone
C. Flattening of the condyles with no joint space seen on
the radiograph
D. Bony fusion of the outer edge of the articular surface
of the joint
55. A rib harvested for growth center transplantation fol-
lowing TMJ arthoplasty, should have at least the fol-
lowing amount of cartilage attached to it:
A. Smm B. lOmm
C. lSmm D. 25mm
56. A reciprocal click occuring during movement of the
TM joint is indicative of:
A. Anterior disc displacement with reduction
B. Complete anterior disc dislocation of the disc
C. Disc displacement with perforation
D. Normal joint function
57. The canfiled operation to explore the maxillary sinus
utilizes assess through:
A. Inferior angle of the anteroinferior angle of the antrum
B. The canine fossa above the premolar teeth
C. An opening created by removal of lower portion of
the angle formed by the junction of antral and nasal
walls
D. An opening made below the inferior turbinate

540 55 B 56 A 57 A
Maxillofacial Injury 209

21. In replanting an avulsed tooth:


A. It should be thoroughly made sterile
B. Root filling with apicoectomy should be done
C. There is failure que to external resorption
D. All of the above
22. Pathognomonic sign of fracture mandible Is:
A. Deranged occlusion
B. Tenderness and swelling at site
C. Sublingual haematoma
D. Inability to open mouth
23. Fracture of coronoid process can occur due to:
A. Trauma at chin region
B. Trauma from posterior region
C. Reflex muscular contraction
D. Lateral trauma
24. The term vertical in 'vertical favourable' fractures con-
notes:
A. The fracture line running in vertical direction
B. The displacement of fracture is in vertical plane
C. The direction of view of the observer is in vertical
direction
D. Fracture can be reduce vertically
25. Submentovertex view is an ideal view for diagnosing
fracture of:
A. Zygoma
B. Zygomatic arch
C. Horizontal fracture of mandible
D. Nasoethmoid region

21 c 22 c 23 c 24 c 25 B
204 MCQs in Oral Surgery

63. Early movements of TMJ following surgery for TMJ


ankylosis is:
A. Desirable
B. Harmful
C. Contraindicated
D. BandC

64. Which of the following incisions best exposes TMJ:


A. Submandibular
B. Preauricular
C. Risdon
D. Intraoral
65. The following clinical disease process affect the TMJ
directly:
A. Ankylosis
B. Arthritis
C. Dislocation
D. All of the above
66. Best treatment for a small opening of a disease free
maxillary sinus is:
A. Not treating socket but advising the patient proper
home care
B. Packing tile socket with a hemostatic agent to
encourage clotting
C. Leaving the socket undisturbed but prescribing nasal
vasocontrictor and antibiotics.
D. Employ primary closure of the socket using sutures

63 A 648 65 D 66 A
Maxillofacial Injury 211

30. Indirect reduction of fracture zygoma can be done by:


A. Gillies approach
B. Intraoral approach
C. Percutaneous approach
D. All of the above
31. The optimum length of screw, for fixation of plate in
mandible is:
A. 2mm B. 3mm
C. 4mm D. 6mm
32. The most commonly injured tooth during the place-
ment of mini plate for the fracture of mandible in ante-
rior region may be:
A. Central incisor B. Lateral incisor
C. Canine D. 1st premolar
33. In fractures of mandible in elderly patients, fixation of
plate is: ·
A. Submucosal B. Supraperiosteal
C. Subperiostal D. None of the above
34. The contraindication to miniplate along the line of
osteosynthesis would be:
A. A comminuted fracture
B. An infected fracture site
C. A fracture in a 10-year-old
D. When more than one fracture site exists in mandible
35. Stress shielding effect is seen in:
A. Miniplating
B. Compression bone plating
C. Lag screw
D. Transosseous wiring

30 0 31 c 32 c 33 B 34C 35 B
206 MCQs in Oral Surgery

4. Radiographic examination following chin trauma to a


patient discloses a unilateral fracture of the mandible.
Clinical examination discloses a deviation of· the jaw
to the right side on opening. One would suspect frac-
ture of the:
A. Symphysis
B. Left condyle of the mandible
C. Right condyle of the mandible
D. Left body of the mandible
5. The most common site of fracture of the mandible is
the:
A. Angle B. Symphysis
C. Coronoid process D. Midbody
6. Fat embolism may result following:
A. Nephrosis B. Burn
C. Crush injury D. Diabetes
7. A LeFort I fracture is a:
A. Transvers fracture of the maxilla
B. Pyramidal fracture of the maxilla
C. Craniofacial dysjunction
D. Fracture of the zygomatic arch
8. A patient with maxillofacial injuries should be carried
in:
A. Supine position B. Lateral position
C. Prone position D. Sitting position

9. Immediate management of nasal bleed in facial inju-


ries is:
A. Reduction of nasal bones manually
B. Paraffin gauze packing
C. Positioning the patient in supine position
D. Positioning the patient in prone position

4 c 5 A 6 c 7 A 8 B 9 B
Maxillofacial Injury 213

41 . Tongue-tie is indicated in:


A. Bilateral parasymphysis fracture
B. Unconscious patient
C. Chin has been destroyed in gw1shot
D. All of the above
42. Glasgow coma ~cale is used:
A. To ascertain motor resp!Jnsiveness
B. Verbal responsiveness
C. Eye response
D. To ascertain level of consciousness
43. Examination of pupils is of paramount importance in
maxillofacial injuries because it indicates:
A. Trauma to brain
B. Trauma to op tic tract
C. Progress of patient after trauma
D. All of the above
44. The method commonly used to differentiate nasal dis-
charge from CSF in fracture of middle third of face:
A. Examining level of glucose
B. Examining level of chlorides
C. Drying the discharge on a piece of cloth
D. Examining the level of p roteins
45. Patient with maxillofacial injury complains of regur-
gitation, absence of gag reflex and weakening of voice,
he may have:
A. Laryngeal trauma
B. Injury to middle cranial fossa
C. These symptoms are d ue to acute pain
D. Paralysis of IX N

41 D 42 D 43 D 44C 45 B
208 MCQs in Oral Surgery

16. The differentiating feature of bleeding due to black


eye and that due to fracture of orbit is/are:
A. Circumorbital ecchymosis in black eye develops
rapidly
B. Posterior lirni t of subconjunctival haemorrhage cannot
be seen in black eye
C. Posterior limit of subconjunctival haemorrhage can
be seen in black eye
D. None of the above
17. Guerin type fracture is same as fracture:
A. Le Fort I B. Le Fort II
C. Suprazygomatic.D. LeFort III
18. 'Moon face' appearance is not present in fracture:
A. Le Fort I B. Le Fort II
C. Zygomatic complex D. LeFort III
19. 'Dish face' deformity commonly seen with fractures
of middle third of face is because of:
A. Posterior and downward movement of maxilla
B. Anterior and forward movement of maxilla
C. Anterior and downward movement of maxilla
D. Nasal complex fracture
20. In a crown-root fracture of the tooth, if fracture is not
below alveolar bone and pulp is not exposed the tooth
should be:
A. Endodontically restored
B. Extracted
C. Only jacket crown given
D. Observed for 3-6 weeks

16 c 17A 18 c 19 A 20 c
Maxillofacial Injury 215

51 . A patient presents with bilateral infraorbital step, para-


esthesia on left cheek region, with posterior gagging,
and mobility of maxillary complex at nasal bones, it
indicates:
A. Bilateral fracture zygoma
B. Bilateral fracture LeFort II
C. Fracture zygoma left side with bilateral LeFort II
D. Fracture bilateral subcondylar and fracture zygoma
left side
52. A patient complains of diplopia following fracture zy-
goma, this is because of:
A. Fracture of orbital floor
B. Entrapment of medial rectus
C. Entrapment of superior oblique
D. All of the above
53. Traumatic telecanthus is associated with:
A. Bilateral Le Fort II fracture
B. Nasoethmoidal injury
C. Fracture nasal bones
D. Bilateral fracture zygoma with enophthalmos
54. Guerin sign Is presence of:
A. Ecchymosis at mastoid area
B. Ecchymosis at greater palatine foramen area
C. Ecchymosis in zygomatic butress area
D. Ecchymosis in subling ual area
55. The typical 'cracked pot' sound on percussion of up-
per teeth is indicative of fracture:
A. LeFort I
B. LeFort II
C. Le Fort III
D. AandB

51 B 52 A 53 B 548 55 D
210 MCQs in Oral Surgery

26. Gunning type splints are used when patient is:


A. Edentulous in one jaw
B. Edentulous in both jaws
C. When vertical relation is not known
D. All of the above
27. A 32-year-old female patient reported with bilateral
subcondylar fracture with anterior open bite, the treat-
ment would constitute:
A. Intermaxillary Fixation for 6 weeks
B. Distraction with rubber stoppers and anterior traction
followed by Intermaxillary fixation for 4-6 weeks
C. Intermaxillary fixation for 4 weeks
D. Distraction with rubber stoppers and posterior
traction followed by intermaxillary fixation for 4-6
weeks
28. If fracture angle result following extraction of man-
dibular impacted 3rd molar the immediate treatment
should be:
A. IMFonly
B. Bone plating (under GA)
C. Superior border transosseous wiring and IMF
D. Transosseous wiring at the lower border and IMF

29. Walsham's forceps are used for:


A. Disimpaction of maxilla
B. Reduction of maxilla fractures
C. Reduction of fracture nasal bones
D. Ash septal force

26 D 27 B 28 c 29 c
Maxillofacial Injury 217

61. Step and mobility at infraorbital margin and step at


zygomatico-frontal region would indicate fracture:
A. Le Fort III and Le Fort II
B. Le Fort II and zygoma
C. Le Fort III and zygoma
D. None of the abov
62. Fracture Le Fort II involves the following bones:
A. Frontal process of maxilla, nasal, lacrimal
B. Frontal process of maxilla, lacrimal, ethmoidal
C. Frontal, maxilla and nasal
D. Maxilla, frontal process of zygoma, nasal and lacrimal
63 . A patient presents with open bite on left side and with
tenderness at nasal bones, it could be fracture:
A. Unilateral Le Fort I on right side
B. Subcondylar on left side and zygoma on right side
C. Le Fort II on right side
D. Zygoma on right side and subcondylar on right side
64. If there is root fracture in apical third of tooth without
mobility:
A. Tooth should be extracted
B. Treated endodontically
C. No treatment and periodic review
D. None of the above
65. High rate of fractures at canine region of mandible is
due to:
A. Change of direction of forces occurring here
B. Long canine root
C. Lower border is thin in this area
D. Alveolus is thin in this area

61 B 62 A 63 c 64C 65 B
212 MCQs in Oral Surgery

36. During compression bone plating which type of heal-


ing would be observed:
A. Contact healing
B. Gap healing
C. Primary healing
D. Secondary healing
37. In Luhr system of plating, the two individual compres-
sion screws move through :
A . 1 mm B. 1.6mm
C. 3.2 mm D. 4 mm
38. The whole of middle third of face can be approached
by:
A. Infraorbital incisions
B. Bicoronal flap
C. Alkayat and bramley approach
D. Transconjunctival approach
39. Acceptable treatment modality for fracture mandible
in a 8-year-old is:
A. Transosseous wiring since IMF cannot be done
B. IMF for 3 weeks
C. Circum-mandibular splinting
D. AO bone plating
40. A patient with maxillofacial injuries should be carried
in a supine position only when there is:
A. Spinal, cervical injury
B. Bilateral parasymphysis fracture
C. Unconsciousness
D. Excessive mobility of fractured maxilla

36 c 37 B 38 B 39 c 40 A
Maxillofacial Injury 219

71. To find if fracture of angle mandible is vertically


favourable or unfavourable the radiograph advised:
A. P A view mandible
B. Lateral oblique 30° mandible
C. Occipitomental view
D. Lateral oblique 15° mandible
72. There is absolute indication for extraction of a tooth
which Is present in the fracture line when there is:
A. Longitudinal fracture of tooth involving the root
B. Infected fracture line
C. Dislocation of tooth from its socket
D. All of the above
73. A 7 -year-old boy presented with fracture of left
subcondylar region with occlusion undisturbed, the
treatment would be:
A. lmmobilisationfor 7 days
B. Immobilisation for 14 days with intermittent active
opening
C. No immobilisa tion with restricted mouth opening for
10 days
D. No immobilisation and active movement
74. While doing circum-mandibular wiring there are
chances of injuring:
A. Facial nerve B. Facial artery, vein
C. Epiglottis D. Lingual nerve
75. The submandibular incision for approaching angle
fracture is placed one finger breadth below the lower
border of mandible:
A. To keep the incision line masked
B. To prevent injury to facial vessels
C. To prevent injury to marginal mandibular nerve
D. Access becomes easy

71 A 72 0 73 0 74 8 75 c
214 MCQs in Oral Surgery

46. Treatment of choice of a linear nondisplaced fracture


of the body of the mandible with a full complement of
teeth is:
A. Kirchner's wire
B. Circumferential wiring
C. External pin fixation
D. Closed reduction w ith intermaxillary fixation
47. Following a bilateral fracture of the mandible in the
canine region, the anterior fragment of the mandible
is displaced posteriorly by the action of the:
A. Anterior belly of the digastric muscles, geniohyoid
and genioglossus muscles
B. Thyrohyoid, genioglossus and geniohyoid muscles
C. Mylohyoid, genioglossus and geniohyoid muscles
D. Mylohyoid, geniohyoid and thyrohyoid muscles
48. The best radiographic view for evaluation of fracture
of the middle face is:
A. PA skull ·B. Lateral skull
C. Towne's view D. Water's view
49. The most important step in suturing lacerated lip:
A. Apposition of muscular layer
B. Apposition o(vermilion border
C. Apposition of mucosal layer
D. All of the above
50. A patient presents with lateral subconjunctival
haemorrhage. Infraorbital step and diplopia on right
side with inability to open mouth, he can be having:
A. Fracture subcondylar right side
B. Fracture zygoma right side
C. Fracture Le Fort II right side
D. Fracture of floor of the orbit

46 D 47 A 48 D 49 D 50 B
Maxillofacial Injury 221

81. A true open bite is seen in which of the following frac-


tures:
A. Horizontal fracture of the maxilla
B. Fracture of the angle of the mandible
C. Unilateral condylar fracture
D. Fracture of the zygomatic bone
82. Which of the following mouthguards are most rec-
ommended for high impact sports such as rugby?
A. Type I (stock) mouthguards
B. Type II (boil and bite) mouthguards
C. Type III (custom) mouthguards
D. All are equally effective
83. The "Epi-Tek" catheter is used for:
A. Drainage of pus from a fascial space
B. Administration of fluids
C. Sialography
D. Control of nasal haemorrhage
84. Endotoxic shock is seen In patients of maxillofacial
trauma with related:
A. Abdominal injuries
B. Cardia-thoracic injuries
C. Crush injuries
D. Spinal cord injuries
85. Indications for neurosurgical referral In a patient with
maxillofacial trauma include all, except:
A. Meningitis
B. Large aerocoele
C. Excessive oral/nasal bleeding
D. Young patient w.ith CSF leak

81 A 82 c 83 D 84A 85 c
216 MCQs in Oral Surgery

56. In Le Fort I fracture infraorbital rim is:


A. Bilaterally involved
B. Not involved
C. Involved medially
D. May or may not be involved
57. Ecchymosis at zygomatic buttress would indicate
fracture:
A. Le Fort I B. Le Fort II
C. Zygoma D. All of the above
58. On palpation there is a step at bilateral infraorbital
margins and mobility of midface is detectable at na-
sal bridge a possible diagnosis would be fracture:
A. Le Fort I B. Le Fort II
C. Le Fort III D. Le Fort III and II
59. There is tenderness at zygomatico-frontal suture, with
hooding of eyes and step at zygomatic arches with
disturbed occlusion, a possible diagnosis would be:
A. Fracture zygoma and zygomatic arch
B. Fracture zygoma with paralysis of III nerve causing
hooding
C. Fracture Le Fort II and fracture zygoma
D. Fracture Le Fort III
60. On moving the maxilla bimanually, movement is felt
at zygomatico-frontal suture area In a case of middle
third fracture of face, it is indicative of fracture:
A. LeFort I
B. LeFort II
C. Le Fort III
D. Zygoma

56 B 57 D 58 B 59 D 60 c
222 MCQs in Oral Surgery

86. As per the Glasgow Coma Scale, motor responsive-


ness score 3 denotes:
A. Obeying
B. Localising
C. Extending
D. Flexing
87. Acrylated arch bars are also known as:
A. Krupps arch bars
B. Winter's arch bars
C. Wipla arch bars
D. Schuchardt arch bars
88. Contraindications for use of gunning splints in
patients with fractures of the mandible include all,
except:
A. Grossly comminuted fractures
B. Fractures of the atrophic mandible
C. Bilateral fractures of the edentulous mandible where
proximal fragments can be controlled by IMF
D. Posterior displacement of the anterior part of the
mandible
89. Following intermaxillary fixation, a patient can lose
weight on an average upto:
A. 5 kg B. 5.5 kg
C. 6.5 kg D. 7 kg

90. Log rolling a trauma patient into the recovery posi-


tion minimizes the risk of:
A. Damage to the spinal cord
B. Aspiration of blood or foreign bodies
C. Obstruction of the airway from the displaced tongue
D. None of the above

86 D 87 D 88 c 89 c 90 A
218 MCQs in Oral Surgery

66. Respiratory embarrassment can occur in fracture:


A. Angle
B. Parasymphysis
C. Bilateral parasymphysis
D. Bilateral subcondylar
67. A patient reported with deviation of jaw to the right
side on opening and bleeding from the right ear, is a
typical picture of:
A. Left-subcondylar fracture
B. Right-subcondylar fracture
C. Right-subcondylar with fracture of anterior cranial
fossa
D. None of the above
68. A patient with bilateral subcondylar fracture presents
with:
A. Inability to op17n mouth
B. On opening mandible moves forward
C. Anterior open bite
D. Closed bite
69. A horizontally unfavourable fracture of angle of man-
dible runs from:
A. Lingual plate anteriorly backward through buccal
plate posteriorly
B. Upper border downward and forward
C. Upper border downward and backward
D. None of the above
70. Best radiograph for fractures of middle third of face:
A. Submentovertex
B. Reverse Towne's view
C. OPG
D. Occipitomental view

66 c 67 B 68 c 69 c 70 D
220 MCQs in Oral Surgery

76. In old patients, open reduction and fixation should be


done with great care to:
A. Prevent iatrogenic fracture of atrophic mandible
B. Detach minimum of periosteum
C. Prevent dislocation of condyle
D. None of the above
77. Following are the examples of rigid fixation:
A. Lateral frontal suspension
B. Extraskeletal pin fixation
C. Bone plating
D. B andC
78. To fix a zygomatic fracture by open reduction follow-
ing sites have to be approached:
A. Zygomatic, frontal and infraorbital
B. Infraorbital and zygomaticotemporal
C. A and B
D. Zygomaticofrontal, zygomatic prominence and, floor
of orbit
79. Tetanus immunization of patients with facial injuries
includes:
A. Administration of antibiotics
B. Administration of 0.5 ml of absorbed toxoid
C. Administration of 250 units of TIG
D. (B) and (C) above
80. The 'Golden Hour of Trauma' refers to:
A. The period of time between minutes and hours after
the trauma
B. The period of time in seconds or minutes after the
traumatic incident
C. The period of time exactly one hour after the trauma
is sustained
D. The period of time during which the patient regains
consciousness after trauma

76 B 77 c 78 c 79 0 80 c
Maxillofacial Injury 223

91. The "parade-ground" fracture refers to:


A. Bilateral fractures of the mandibular angles
B. Midline symph yseal and undisplaced bilateral
condylar fracture
C. Bilateral fracture dislocation of the condyles
D. Midline symphyseal and bilateral fracture dislocation
of the condyles
92. A patient with severe maxillofacial trauma has a sys-
tolic pressure less than 100 mm Hg. The expected
amount of blood loss should be in the range of:
A. 500 ml B. 750 ml
C. 1000 ml D. 1250 ml
93. Replantation technique of avulse mature teeth
requires:
A. Removal of any soil contamination by rinsing with
water followed by sterilization
B. Leaving the pulp tissue intact but sealing the apex
with an alloy prior to replantation
C. Removal of any blood clot in the socket by curettage
D. None of the above
94. "Matchbox" injuries of the maxillo mandibular region
refer to:
A. Burns of the soft tissues overlying the facial skeleton
B. Fracture of the nasal bones
C. Fractures of the middle third of the facial skeleton
D. Crush injuries of the maxilliary sinus
95. Paresthesia is one of the commonest finding in which
of the following fractues?
A. Subcondylar fracture of the mandible
B. Zygomaticomaxillary complex fracture
C. Fractured coronoid and displacement of the fracture
D. Symphysis fracture associated with bilateral
subcondylar fracture (parade ground fracture)

91 c 92 c 93 0 94 c 95 B
224 MCQs in Oral Surgery

96. Clinical findings of a subcondylar fracture on the right


side include:
A. Trismus and crepitus bilaterally
B. Inability to deviate the mandible to the left
C. Deviation of the mandible on protrusion towards the
left side
D. Bleeding intraorally. most of the times moderately
some times severely
97. Clinical findings of a subcondylar fracture on the left
side include:
A. Trismus and crepitus bilaterally
B. Inability to deviate the mandible to the left
C. Deviation of the mandible on protrusion towards the
left side
D. Bleedmg intraorally, most of the times moderately
some times severely
98. After reduction of a mandibular dislocation that
occurred for the first time, treatment should be to:
A. Inject sclerosing solution into the joint, so further
dislocation is less likely
B. Inject corticosteroids into the joint which reduces the
inflammation
C. Immobilize with IMF for S-6 weeks
D. Advise the patient to limit opening of the mo uth for
2-3 weeks
99. Among the following which Is the least common site
of mandibular fracture?
A. Body
B. Coronoid
C. Condyle
D. Angle

96 B 97 c 98 0 99 B
Maxillofacial Injury 225

100. CSF rhinorrhoea commonly seen in patient with:


A. Le Fort I fracture
B. Zygomatic complex fracture
C. Le Fort III fracture
D. Bilateral condylar fracture with symphysis fracture
of mandible
101. Forward displacement of the condyle in condylar frac-
tures is due to:
A. Medial pterygoid
B. Lateral pterygoid
C. Masseter
D. Temporalis
102. The movement employed in the reduction of displaced
mandible is:
A. Downward and backward
B. Upward and forward
C. Upward and backward
D. Downward and forward
103. After a blow to the chin, patient has pain and tender-
ness over the right TMJ. Open bite, and deviation of
the chin to the right side. This suggests:
A. Fracture mandible right angle region
B. Fracture mandible left angle region
C. Fracture mandible right condyle
D. Fracture mandible left condyle
104. Fractures of mandible where full complement of teeth
are present is best treated by:
A. Intraoral open reduction
B. Circumferential wiring
C. Full cast splints
D. Closed reduction and intermaxillary fixation

100 c 101 B 102 A 103 c 104 0


226 MCQs in Oral Surgery

105. In which of the following anterior open bite occur?


A. Bilateral condylar fractures
B. Symphysis fracture on one side and angle fracture on
the other side of the mandible
C. Horizontal fracture of the maxilla
D. AandC
106. For an oral surgery·patient undergoing closed reduc-
tion of a fractured mandible which of the following
procedures should be performed?
A. Medical history and physical examination
B. CBC
C. Urine analysis
D. All of the above
1 07. Bones usually fracture at the sites of:
A. Compressive strain
B. Tensile strain
C. Rich blood supply
D. Thin periosteal covering
108. Energy range required to fracture the mandible is in
the range of:
A. 44.6- 74.4 kg/m
B. 79.2-98.1 kg/ m
C. 10.1-28.5 kg / m
D. 100.2-150.9 kg/ m
109. In case of subcondylar fracture, the condyle moves
in:
A. Anterior lateral direction
B. Posterior medial direction
C. Posterior lateral direction
D. Anterior medial direction

105 D 106 0 107 B 106 A 109 0


Maxillofacial Injury 227

110. Geurin's fracture is synonymus to:


A. Le Fort I level fracture
B. Le Fort II level fracture
C. Le Fort III level fracture
D. Le Fort IV level fracture
111 . The following fracture is usually pyramidal in shape:
A. Le Fort I fracture
B. Le Fort II fracture
C. Le Fort III fracture
D. Mandibular symphysis fracture
112. Craniofacial dysjunction commonly occurs in:
A. Le Fort I fracture
B. Le Fort III fracture
C. Mandibular symphysis fracture
D. Mandibular condyle
113. Gilli's approach is:
A. Used to block inferior alveolar nerve
B. Used to reduce the fractured zygoma
C. Placed just anterior to the ear
D. One of the frequent approach for condylar surgery
114. Lowering the pupillary level of eyeball occur if:
A. The orbital volume increases
B. Detachment of suspensory ligament of lockwood
occurs
C. In case of blow out fracture
D. None of the above
115. How many weeks of fixation are required for fracture
mandible?
A. 8 - 10 weeks B. 6-8 weeks
C. 4-6 weeks D. 2 -4 weeks

110 A 111 B 112 B 113 B 114 B 115 c


228 MCQs in Oral Surgery

116. During the Gillies approach, the structure of anatomic


significance is:
A. Superficial temporal artery
B. Marginal mandibular nerve
C. Internal jugular vein
D. Inferior alveolar nerve
117. An unfavorable displaced fracture of the mandibular
angle is difficult to treat because of:
A. Muscle pull causes d istraction
B. Malocclusion secondary to the injury
C. Injury to nerves and vessels
D. Bone in that region is very thick
118. Among the following which may produce respiratory
obstruction?
A. Bilateral condylar fracture
B. Symphysis frac ture of mandible
C. Bilateral fracture of mandible in the second premolar
area
D. Fracture of the angle of the mandible
119. In a patient with bilateral dislocated fractures of the
necks of the mandibular condyles one can expect the
following clinical signs:
A. Anterior open bite
B. Inability to protrude the mandible
C. Inability to bring posterior molars into contact
D. AandB
120. Of the following which facial bone is most frequently
fractured?
A. Mandible B. Maxilla
C. Nasal D. Zygomatic

116 A 117 A 118 c 119 D 120 D


Maxillofacial Injury 229

121 . Ecchymosis in the post auricular region over the mas-


toid process is called as:
A. Battle's sign
B. Murphy's sign
C. Guiren's sign
D. None of the above
122. "Hanging drop" sign in radiograph usually indicate:
A. Nasal bone fracture
B. Orbital floor fracture
C. Isolated coronoid fracture fragment hanging by
temporalis muscle
D. Condylar fracture
123. Of the following wh-ich is almost pathognomonic of a
mandibular fracture?
A. Deep laceration near the area of trauma
B. Ecchymosis in the lingual sulcus
C. Anterior open bite
D. None of the above are correct. There is no pathogenic
fea ture of mandibular fracture
124. Among the following where one might see "Bucket
handle" displacement of fracture segments:
A. Bilateral mandibular body in a 60-year-old
B. Isolated nasal bone fracture in a 35-year-old female
who is pregnant
C. Unilateral zygomatic maxillary complex in a 40-year-
old patient w ho is wrestler by occupation
D. Pure blow out fracture of floor of the orbit in a
20-year- old tennis player

121 A 122 B 123 B 124 A


230 MCQs in Oral Surgery

125. A patient came to the trauma center who had a blow


over his lower jaw. Intraoral examination reveals he-
matoma near lingual side of lower second molar. The
proximal fragment is medially displaced. Which of the
following might explain the above findings?
A. Vertically favorable fracture of angle of the mandible
and displacement is due to masseter action
B. Vertically unfavorable frac ture of angle of the
mandible and displacement is due to internal
pterygoid action
C. Horizontally favourable fracture at the angle and
displacement is due to medial pterygoid action
D. Horizontally unfavourable fracture at the angle and
displacement is due to masseter action
126. In case of pure symphysis fracture of mandible, frac-
ture segments are usually displaced:
A. Lingually and downward by the pull of genioglossus
mylohyoid musCles
B. Mainly lingually and lingual and downward
movement due to the pull of geniohyoid and
myohyoid musle
C. Mainly upwards movement. Lingual and upward
movement due to the pull of geniohyoid and
mylohyoid muscle
D. None of the above a re correct. A little (or) no
displacement occur
127. Excessive muscular contraction is one ofthe frequent
cause of:
A. Unilateral condylar fracture
B. Cornoid fracture
C. Fracture of angle of the mandible
D. Bilateral condylar fracture

125 B 126 0 127 B


Maxillofacial Injury 231

128. Which of the following is right regarding Guardsman


fracture?
A. It is nasal fracture associated w ith unilateral
zygomatic maxillary complex fracture
B. CSF rhinorrhoea is severe in this cases
C. Commonly seen in epileptics
D. It is a type of comminuted fracture due to bullet (or)
missile injures in the war field
129. "Panda Facies" is one of the term to describe the pa-
tient faceafter mid face trauma. The appearance is due
to:
A. A-gross swelling of the face
B. CSF rhinorrhoea and bleeding from the nose and
laceration results in red and white streaks on the face
C. Edema and ecchymosis around the eyes
D. Sub conjunctival hemorrhage (Bilateral)
130. Tram line pattern on the face is due to:
A. Sutures placed with tension
B. CSF rhinorrhea
C. Circumorbital ecchymosis !
D. Sub conjunctival hemorrhage
131. Of the following which is weakest part of orbit:
A. A Medial wall B. Lateral wall
C. Floor of the orbit D. Roof of the orbit
132. Posterior displacement of the fractured anterior seg-
ment In the bilateral fracture of the mandible in the
canine region is due to the action of the:
A. Thyrohyoid, genioglossus and geniohyoid
B. Mylohyoid, genioglossus and geniohyoid
C. Geniohyoid and genioglossus and anterior belly of
digastric muscles
D. Mylohyoid, geniohyoid and thyrohyoid muscle

128 c 129 c 130 B 131 c 132 c


232 MCQs in Oral Surgery

133. Which of the following is complication often open frac-


ture?
A. Malunion
B. Nonunion
c. Infection
D. Crepitation
134. A patient with suspected cervical fracture should, be
kept in:
A. Body and neck extended
B. Prone position
C. Body extended and neck flexed
D. Both body and neck flexed
135. Principles in treating fractures include:
A. Reduction of fracture
B. Fixation of fracture and restoration of occlusion
C. Immobilisation
D. All of the above
136. Depressed fracture of the zygomatic area may be clini-
cally recognised by:
A. Concavity of the overlying tissue in the zygomatic
arch area
B. Interference w ith movements of the mandible
C. Subluxation of condyles
D. A and B
137. Which of the following is characteristic of Lefort frac-
ture?
A. CSF rhinorrhoea
B. Bleeding from the ear
C. Bleeding into the antrum
D. Aand B

133 c 134 A 135 0 136 0 137 c


Maxillofacial Injury 233

138. Most common anatomic site of fracture mandible is:


A. Angle
B. Condyle
C. Coronoid
D. Body
139. Of the following, which is the immediate treatment for
a patient with comminuted fracture and in the state of
shock?
A. Ringer's lactate solution by IV route
B. Normal saline by IV route
C. Blood transfusion
D. Plasma expanders
140. A Gunning's splint is used in the treatment of:
A. Fracture of the condylar neck of a child
B. Fracture of the edentulous mandible
C. Anterior dentoalveolar fracture
D. None of the above
141. Presence of ecchymosis in the sulci, the floor of the
mouth and hard palate usually suggest there is a:
A. Laceration
B. Fracture
C. Abrasion
D. None of the above
142. Among the following which is not used in the fixation
of bone grafts?
A. A Bone plates
B. Titanium mesh
~· Gut
D. Tranosseous wires

138 A 139 A 140 B 141 B 142 c


234 MCQs in Oral Surgery

143. Which is most frequently performed to correct a skel-


etal mandibular retrognathia?
A. 'C' type osteotomy
B. Horizontal osteotomy or the ramus
C. Oblique sub condylar
D. Sagittal split osteotomy of the ramus
144. Which of the following procedure best suited to cor-
rect bimaxillary protrusion?
A. Extraction of four premolars and anterior alveolar
segment repositioning
B. Mandibular body osteotomy and posterior maxillary
osteotomy
C. Subcondylar osteotomy
D. None of the above
145. What are the principles in the treatment of mandibu-
lar fractures to ensure rehabilitation of jaw function?
A. Reduction of the fracture
B. Fixation of the fracture
C. Restoration of occlusion
D. All of the above
146. Among the following which muscle plays least role to
displace fractured mandibular angle?
A. Temporalis B. Triangularis
C. Medial pterygoid D. Lateral pterygoid
147. The proximal segment of mandibular angle ·f racture
usually displaced in which direction?
A. Anterior and superior
B. Posterior and inferior
C. Inferior only
D. Posterior and superior

143 0 144 A 145 D 146 B 147 A


Maxillofacial Injury 235

148. After a depressed fracture of zygomatic arch mandibu-


lar movement is restricted. The most probable rea-
son is:
A. Disruption of TMJ
B. Spasm of the lateral pterygoid muscle
C. Mechanical impingement of the fracture fragment on
the coronoid process
D. Splinting action of masseter and medial pterygoid
muscle
149. Among the following which is compound fracture?
A. Fracture with many small fragments
B. Fracture in a "star" shaped appearance
C. Fracture with communition with the oral cavity
D. Fracture with bleeding into the masticator space
150. Most common disorder causing pain about the mas-
ticatory apparatus including the TMJ is:
A. Trigeminal neuralgia
B. MPDS
C. Degenerative arthritis
D. Traumatic arthritis
151. Among the following which extra oral radiograph best
demonstrates the subcondylar fracture?
A. Towne's projection
B. AP Mandible
C. Submento vertex
D. Occipitomental

148 c 149 c 150 B 151 A


236 MCQs in Oral Surgery

BONE PLATING
152. The mini-bone plate system is a:
A. Compressive bone plating system
B. Monocortical system
C. Bicortical system
D. None of the above
153. The best and most effective position (in mandible) of
miniplate as proved by various experimental studies
is:
A. Lower border of mandible
B. Buccoalveolar region
C. Linguoalveolar region
D. At a height midway between superior alveolar region
and lower border of mandible
154. The minimum number of mlniplates required in frac-
tures anterior to canine in mandible is:
A. No plate is required since anterior region develops
less amount of tension forces than in molar region
B. Only one plate as in molar region
C. Two plates
D. Three plates
155. Minimum number of screws required for fixation of
miniplate are:
A. One screw on each side of fracture site
B. Two screws on each side of fracture site
C. Three screws on each side of fracture site
D. Two screws in smaller fragment and three screws in
larger fragmen t

152 B 153 c 154 c 155 B

/
Maxillofacial Injury 237

156. In a fracture of mandible at the angle-region the place-


ment of screws in proximal segment is in:
A. Sagittal plane
B. Horizontal plane
C. Such a close relation to teeth that injury to molar
invariably occurs
D. No relation to teeth
157. To prevent injury to the apices of the teeth in man-
dible, the placement of miniplate is:
A. At the lower border of mandible
B. At a distance; twice the height of the clinical crown
below the alveolar crest
C. Below the inferior alveolar canal
D. Not possible since alveolar bone bears the apices of
the teeth
158. The spherical gliding principle is a feature of:
A. Miniplates
B. Luhr plating
C. ASIF plating
D. Lag screws
159. Epiphora results due to:
A. Blockage of lacrimal gland canaliculi
B. Blockage of nasolacrimal duct
C. Overactivity of lacrimal glands
D. Evulsion of palpebral conjunctiva
160. The most common site of mandible, which shows non
union or delayed union after IMF is:
A. Angle B. Body
C. Symphysis D. Ramus

156 A 157 B 158 c 159 B 160 c


238 MCQs in Oral Surgery

161. Risdon wiring is indicated for:


A. Body fracture
B. Angle fracture
C. Symphysis frac ture
D. Subcondylar fracture
162. Facial paresis following maxillofacial fractures is most
common in:
A. # of the condylar neck
B. # of the nasal bones
C. #of the zygomatico-maxillaryconiplex
D. # of the mandibular symphysis
163. The most common complication of maxillofacial inju-
ries requiring immediate attention is:
A. Haemorrhage
B. Airway obstruction
C. Infection
D. Shock
164. The safest initial approach to opening the airway of a
patient with maxillofacial trauma and suspected neck
inj ury is:
A. Head tilt-chin lift
B. Jaw thrust technique
C. Head lift-neck lift
D. Heimlich procedure
165. The Gillies approach is used to gain access to the
following bone:
A. Nasal bone
B. Zygomatic bone
C. Maxilla
D. Temporal bone

16 1 c 162 c 163 B 164 B 165 B


Maxillofacial Injury 239

166. Ecchymosis in the mastoid region seen after a frac-


ture of the petrous bone is known as:
A. Chovstek's sign
B. Battle's sign
C. Guerin's sign
D. Tinel's sign
167. Direct interdental wiring is also known as:
A. Risdon's wiring
B. Gilmer's wiring
C. Eyelet wiring
D. Col. Stout's wiring
168. The " 2.7" in the 2.7 mm plating system denotes:
A. The bone plate thickness
B. The bone screw diameter
C. The diameter of the plate hole
D. The distance between the plate
169. The proximal segment of an horizontally unfavourable
mandibular angle fracture Is displaced:
A. Inferiorly
B. Posteriorly and medially
C. Anteriorly and superiorly
D. Anteriorly and laterally
170. The outermost holes of an " eccentric dynamic com-
pression plate" are angled at :
A. 75 Degree
B. 85 Degree
C. 60 Degree
D. 55 Degree

166 B 167 ·a 168 B 169 c 170 A


240 MCQs in Oral Surgery

171 . CSF rhinorrhea follwing a Le Fort Ill frontal bone frac-


tu re is because of:
A. # of the cribriform plate of the ethmoid
B. # of the posterior wall of the frontal sinus
C. # of the roof of sphenoid air sinus
D. All of the above
172. 'Panda facies' is commonly seen after:
A. Le Fort I fractures
B. Le For t II fractures
C. Zygomatic arch fractures
D. Orbital blow-out fractures
173. A subconjunctival haemorrhage remains bright red
in colour for a long time because of the:
A. Permeability of the conjunctiva to oxygen
B. Natural colour of blood
C. Lack of d rainage of the pooled blood
D. None of the above
174. The following structures are divided when the angle
of the mandible is exposed through a submandibular
incision:
A. Skin and superficial fascia only
B. Skin, superficial fascia, platysma, deep cervical fascia
and medial p terygoid m uscle
C. Skin, su perficial fascia , deep cervical fascia and
masseter muscle '
D. Skin, superficial fascia, platysma, deep cervical fascia
and masseter muscle

171 D 172 B 173 A 174 D


Maxillofacial Injury 241

17 5. The most common pathognomonic sign of a mandibu-


lar fracture is:
A. Malocclusion
B. Trismus
C. Deviation of the jaw on opening
D. Paraesthesia of the mental nerve
176. A protein supplement is provided for a 70 kg adult
patient who has been treated with intermaxillary fixa-
tion following a non displaced mandibular angle frac-
ture. The amount of protein that he requires per day
is:
A. 100 mg/kg/day' B. 100 gm/kg/day
C. 1000 mg/kg/ day D. 500 mg/kg/ day
177. Clinical union of fractures occurring in the region of
the middle third of the facial skeleton takes place, on
an average within :
A. 3 to 4 weeks B. 4 to 6 weeks
C. 6 to 8 weeks D. 2 to 4 weeks
178. Reduction of a fractured malar bone is best carried
out after:
A. Peri-orbital oedema has subsided
B. Three to five days after injury
C. Chemosis has subsided
D. All of the above
179. A Class Ill fracture of the tooth is:
A. A fracture of only the enamel portion of the crown of
the tooth
B. An injury extending into the dentin but w ith no p ulpal
exposure
C. An extensive injury to the coronal portion of the tooth
with pulp exposure
D. A fracture occurring at or below the cementoenamel
junction of the tooth

175 A 176 c 177 A 178 D 179 c


242 MCQs in Oral Surgery

180. Elastic traction used commonly to reduce facial frac-


tures, does so by overcoming:
A. The active muscular pull that distracts the figments
B. The organized connective tissue at the fracture site
C. The malposition caused by the direction and force of
trau ma
D. All of the above
181 . The gap created between the base of the skull and
the ramus of the mandible during a TMJ arthroplasty
to prevent reankylosis should be at least:
A. 05. to 1.0 em
B. 1.0 to 1.5 em
C. 1.5 to 2.5 em
D. 2.5 to 3.5 em
182. Gun shot fractures of the facial bones should not be
treated via open reduction because:
A. Infection w ill definitely occur
B. Closure of the wound due to soft tissue loss is difficult
C. The n umerous small fragments w ill lose their vitality
when the periosteum is reflected
D. All of the above
183. While doing circumferential wiring around a mandibu-
lar gunning splint, care must be taken not to damage
the:
A. Mandibular b ranch of the facial nerve
B. Facial artery as it crosses the anteroinferior of the
masseter
C. The lingual nerve
D. The submandibular gland and its d uct

180 D 181 B 182 c 183 B


1. A transplant of bone from one human to another's is
termed:
A. Autogenous B. Homologous
C. Heterogenous D. Alloplastic
2. Which of the following surgical procedures are com-
monly used for treatment of maxillary retrognathia?
A. Le Fort I osteotomy
B. C-osteotomy
C. Inverted L osteotomy
D. Anterior maxillary osteotomy

3. Which of the following materials is best to restore a


missing portion of the mandible?
A. Silastic B. Tantalum
C. Chrome cobalt D. Autogenous bone

4. A patient with class II div I malocclusion is operated


for genioplasty his anterior teeth after tt)e operation
would be:
A. In edge to edge bite
B. Without any change
C. Having normal overjet of 2 mm
D. Having no overbite

1 B 2 A 3 D 4 B
244 MCQs in Oral Surgery

5. Jumping genioplasty Is a term which connotes:


A. Movement of chin posteriorly
B. Double step genioplasty
C. Single step advancement
D. Advancement after set back of mandibular body
6. Sagittal split osteotomy Is a procedure carried out for:
A. Mandibular deformities
B. Maxillary deformities
C. Deformities in which occlusion in not involved
D. Condylar repositioning
7. Sagittal split osteomy was first advocated by:
A. Obwegesser B. Dal Pont
C. Wunderer D. Moose
8. Apertognathia is a condition in which there is:
A. Retrogenia
B. Maxillary hypoplasia
C. Open bite deformity
D. Maxillary and mandibular prognathism only
9. During genioplasty there are chances of injuring:
A. Inferior alveolar nerve
B. Marginal mandibular nerve
C. Mental nerve
D. Lingual nerve
10. Wassmund and Wunderer procedures are:
A. Mandibular segmental osteotomies
B. Maxillary segmental osteotomies
C. Maxillary subap ical osteotomies
D. Multiple subapical osteotomy procedure of maxilla
and mandible respectively

5 B 6 A 7 A 8 c 9 c 10 B
Reconstructive and Orthognathic Surgery 245

11. White graft are:


A. Nerve rejected
B. Are immunologically biocompatible
C. Are rejected without evidence of vascularization
D. Behave in same manner as autogenous grafts
12. The best bone graft which can be utilized for recon-
struction of large mandibular defect is:
A. Chostochondral graft
B. Calvarial graft
C. Iliac crest graft
D. Metatarsal bone graft
13. Iliac crest graft should ideally be taken from:
A. Lateral aspect
B. Medial aspect
C. Posterosuperior aspect
D. Anteroinferior aspect
14. A patient in whom iliac crest graft has been taken for
mandibular reconstruction, should be kept nil orally
postoperatively:
A. For 6 hrs
B. Till bowel sounds appear
C. For 12 hours
D. Till patient is ambulatory
15. The graft of choice in a 30-year-old patient of
ameloblastlc resection would be:
A. Free iliac crest graft
B. Free vascularized iliac crest graft
C. Medullary bone graft
D. 6th rib

11 c 12 c 13 B 14 B 15 B
246 MCQs in Oral Surgery

16. Alveoplasty should be carried out:


A. When multiple extractions are done in one quadrant
B. When entire arch extraction is there
C. To remove undercuts
D. All of the above
17. The Kazanjian's. technique of vestibuloplasty leaves:
A. Lip surface to re-epithelialise
B. Alveolar surface to re-epithelialise
C. Depth of sulcus periosteum to re-epithelialise
D. None of the above
18. The Lipwitch procedure is used for:
A. Ridge augmentation
B. Sulcoplasty
C. Tuberoplasty
D. Chiroplasty
19. When there is high crestal attachment of muscle and
tissue the indicated method of vestibuloplasty is:
A. Kazanjian's B. Clark's
C. Obwegesser's D. Howe's lipwitch
20. In a patient with class Ill facial profile one would think
of which type of genioplasty:
A. Reduction G B. Advancement G
C. Straightening G D. Rotational G
21. A patient reported with class Ill skeletal deformity the
ideal choice would be:
A. Inverted L osteotomy
B. Segmental osteotomy
C. Reverse sagittal split osteotomy
D. Sagittal split osteotomy

16 D 17A 18 B 19 c 20 A 21 D
Reconstructive and Orthognathic Surgery 247

22. The basic advantage of sagittal split osteotomy is/are:


A. It is carried out intra-orally as well as extra-orally
B. No bone grafting is required when defect is less than
8mm
C. There are no chances of paresthesia
D . All of the above
23. In a patient in whom SNA is 82° and SNB is 96° indi-
cates he would require:
A. Maxillary surgery with setback
B. Mandibular surgery
C. Mandibular advancement
D. Maxillary advancement
24. Allografts are grafts taken from:
A. Same species and individuals are genetically related
B. Different species
C. Sam e species but individuals are genetically not
related
D. Same species and between genetically identical
individuals
25. Composite grafts consist of:
A. Bone only
B. Medullary bone only
C. Bone and soft tissue
D. Particulate bone mixed with resins
26. lntercortical alveoloplasty is done by:
A. Removing margins of cortical plates
B. Removing interseptal bone entirely and collapsing
labial and palatal cortical plates
C. Removing septa till upper third of socket compressing
the cortical plates
D. None of the above

22 B 23 B 24 c 25 c 26 B
248 MCQs in Oral Surgery

27. The principle problem with tuberosity reduction is:


A. Poor access
B. Formation of OAF
C. Infection
D. Damage to posterior superior alveolar nerve
28. Incision for operation of tongue-tie should be placed:
A. Transversely on lingual frenum
B. Longitudinally along lingual frenum on both sides
C. On crest of frenum longitudinally
D. None of the above
29. Surgical treatment of blmaxillary protrusion consists
of:
A. Le Fort I osteotomy with sagittal split osteotomy in
the mandible
B. Subcondylar osteotomy with Le Fort II osteotomy
C. Vertical ramus osteotomy only
D. Anterior maxillary osteotomy with anterior subapical
osteotomy of the mandible
30. The ideal time for repair of a cleft lip is:
A. Immediate after birth
B. 3 weeks to 3 months
C. 3 months to 3 years
D. After puberty
31. The blind subcondylar osteotomy for the correction
of mandibular prognathism
A. May damage the internal maxillary artery and cause
profuse haemorrhage
B. May damage the branches of the facial nerve
C. May be used for cases requiring less than 7 mm
correction
D. All of the above

27 B 28 B 29 D 30 B 31 B
Reconstructive and Orthognathic Surgery 249

32. The extraoral subcondylar ramus osteotomy is indi-


cated for the following, except:
A. Major setback of the mandible(> 10 mm)
B. Minor setback of the mandible (< 10 mm)
C. Vertical shortening of the mandibular ramus
D. Asymmetrical mandibular setback
33. The disadvantage of a full thickness graft is:
A. Pigmentary changes
B. Lack of depth for contour
C. Decreased survival chance
D. Marked tendency to contract
34. The scalpel blade that should be preferred for inci-
sion and drainage of abscess is:
A. No. 15 B. No. 12
C. No. 11 D. No. 10
35. The following are the basic indications for anterior
subapical mandibular surgery, except:
A. To correct a mandibular dentoalveolar protrusion
B. To alter the lower third facial height and increase chin
projection
C. To level an excessive curve of Spee
D. To correct mandibular dental arch asymmetry
36. The medial bony cut given while performing a modi-
fied sagittal ramus osteotomy extends:
A. 15 to 20 mm posteriorly from the anterior border of
the ramus
B. 5 mm above the inferior alveolar neurovascular
bundle
C. Through the entire length of the medical rcitmus upto
the posterior border
D. 5 mm below the sigmoid notch and 10 mm posterior
to the anterior border

32 B 33 B 34 c 3~ B 36 A

- -----
250 MCQs in Oral Surgery

37. A-50-year-old male is undergoing radiotherapy for


carcinoma left mandibular body. He requires hemi-
mandibulectomy and radical neck dissection. The
Ideal time for the surgery would be:
A. Six months after radiotherapy
B. Immediately upon completion of radiotherapy
C. Four to six weeks after completion of radiotherapy
D. Any time during the course of radiotherapy
38. Among the following which is advantage of sagittal
split osteotomy over transoral vertical subcondylar
osteotomy?
A. Greater mandibular movement is achieved
B. Because it is intraoral procedure. No external scar is
produced
C. Alone may be used to correct a mandibular
retrognathism and prognathism.
D. Less chance of injuring the inferior dental canal
39. Goals of orthognathic surgery includes:
A. Improve periodontal stability and periodontal
prognosis.
B. Shorten orthodontic treatment time and improve
orthodontic results.
C. To correct jaw relationships prior to major restorative
procedures
D. All of the above
40. Generally bone marrow for grafting the defects is ob-
tained from:
A. The iliac crest
B. The mandible
C. Maxillary tuberosity
D. Rib

37 c 38 c 39 D 40 A
Ittl MEDICAL
EMERGENCY

1. The surgical risk for a patient with organic heart dis-


ease depends upon his:
A. Cardiac reserve
B. Blood pressure
C. Respiration
D. Pulse rate
2. When a localized clot forms intravascularly at the site
of operation or Injury, It Is called:
A. Embolus
B. Thrombus
C. Coagulated hematoma
D. Platelet agglutination
3. A patient Is wide-eyed , very nervous with an Increased
systolic pressure (with widening of the pulse pres-
sure), increased pulse rate, fine skin and hair and loss
of body weight. He is probably suffering from:
A. Diabetes
B. Hypothyroidism
C. Hyperthyroidism
D. Hyperpituitarism
4. Preoperative sedation with short-acting barbiturates
(Numbutal®, Seconal ®) Is valuable because It:
A. Prevents respiratory depression
B. Produces drowsiness and amnesia
C. Is easily administered and has no side effect
D. Is without hazard and is compatible with narcotics

1 A 2 B 3 c 4 B
252 MCQs in Oral Surgery

46. To get the best long term results, which of the


following procedures for augmentation genioplasty
are recommended?
A. Injection of silas tic gel
B. Onlay bone graft
C. Pedicled horizontal sliding osteotomy
D. Insertion of silastic rubber implant

46 c
Medical Emergency 255

10. In administering artificial ventilation, It Is recom-


mended that the rescuer deliver a resting tidal vol-
ume that Is:
A. Normal
B. Twice the normal
C. Three times the normal
D. Four times the normal
11. A patient who Is currently taking Coumadln ~ay
recently have had:
A. Thrombophlebitis
B. A pulmonary embolism
C. Acute myocardial infarction
D. Any of the above
12. Patients suffering from uncontrolled juvenile onset
diabetes mellitus are usually poor candidates for oral
surgery because of:
A. The high incidence of wound infection and poor
healing
B. Their inability to follow postoperative instruction
C. Their tendency to hemorrhage profusely
D. Abnormal protein metabolism
13. Which of the following always indicated obstruction
to the airway?
A. Slow pounding pulse
B. Stertorous breathing
C. Increase in pulse rate
D. Decrease in blood pressure

10 8 11 D 12 A 13 8
254 MCQs in Oral Surgery

5. A patient who is on a regimen of steroid therapy and


has need for extraction of chronically infected teeth
requires premedication with:
A. Atropine to reduce the hazard of vagal stimulation
and cardiac arrest
B. Antihypertensives to combat tendency toward shock
C. Antihistamines to prevent" allergic reactions
D. Antibiotics
6. Differential white blood cell counts In the laboratory
are useful in the diagnosis of:
A. Anemia B. Eosinophilia
C. Spherocytosis D. Vitamin deficiency

7. Postoperatively, a patient may develop serum sick-


ness as a reaction to a non protein drug or to a bio-
logic product. The reaction occurs after a sensitiza-
tion period of:
A. 1 day B. 3 days
C. 4 days D. 1-3 weeks
8. Complete respiratory obstruction is first manifested
by:
A. Prolonged expiration
B. No movement by the patient
C. Information given by the patient
D. Pronounced retraction of the intercostal and
supraclavicular spaces
9. The immediate postoperative complication following
aspiration of liquid vomitus into trachea and bronchi
is:
A. Pleurisy B. Bronchitis
C. Atelectasis D. Lung abcess

5 D 6 B 7 D 8 D 9 B
Medical Emergency 257

19. Two minutes following cessation of a two-hour


treatment session using 80 percent nitrous oxide,
20 percent oxygen and local anesthesia, the patient
becomes cyanotic and tachycardic:
, A. Over-oxygenation
B. Under-oxygenation
C. Diffusion hypoxia
D. Malignant hyperthermia
20. In order to carry cardiac compression effectively, the .
sternum should be depressed:
A. 2 inches every second
B. 5 inches every second
C. 2 inches every 5 second
D. 1 inch every 10 second
21. Which of the following drugs used for premedication
produce a detached serenity without clouding con-
sciousness?
A. Ataractics B. Antiemetics
C. Analeptics D. Anesthetics
22. Which of the following laboratory studies would be
expected to be abnormal in a patient with hemophilia
A?
A. Partial thromboplastin time
B. Prothrombin time
C. Platelet count
D. Bleeding time
23. Narcotics are used primarily:
A. To control anxiety
B. To counteract depression
C. For the treatment of pain
D. For their ataractic effect

19 c 20 A 21 A 22 A 23 c
256 MCQs in Oral Surgery

14. Which of the following procedures is correct for treat-


ing an unconscious patient with no pulse and dilated
pupils?
A. Start closed chest massage at once
B. Start artificial ventilation at once
C. Start closed chest massage in conjunction with
artificial ventilation
p, Start closed chest massage if the pupils do not constrict
with light
15. Hemostasis depends upon each of the following
except:
A. Vessel wall B. Prothrombin
C. Blood platelers D. Albumin

16. The earliest sign of hemorrhagic shock is


A. Dyspnea B. Hypotension
C. Tachycardia D. Vasoconstriction
17. In shock, vasopressor drugs are preferably given:
A. Sublingually
B. Intravenously
C. Intracardially
D. Subcutaneously
18. The prothrombin time that would cause the practitio-
ner the greatest concern before a surgical extraction
is:
A. 20 percent of normal
B. 40 percent of normal
C. 50 percent of normal
D: 80 percent of normal

14 c 15 D 16 c 17 B 18 A
Medical Emergency 259

30. Hepatitis that commonly occur with multiple transfu-


sions is due to:
A. Hepatitis A B. Hepatitis B
C. Non A: Non B D. All of the above
31 . Which of the following condition can be diagnosed
by differential white blood?
A. Anaemia
B. Spherocytosis
C. Thrombocytopenic purpura
D. Eosinophilia
32. After intravenous administration of a short acting
barbiturates which ·tissue is saturated last as a result
of redistribution:
A. Liver B. Brain
C. Muscle D. Fat
33. After the injection of a test dose of IV sedation a
patient complains of immediate severe burning pain
at the site of injection, radiating distally in the involved
arm. Simultaneously the colour of the arm appears
blacky with several blanched areas. The pulse is regu-
lar but weak compared to the opposite side likely
diagnosis is:
A. Injection into the radial nerve
B. Injection into biceps tendon
C. Injection into an artery
D. Impeding anaphylatic reaction
34. Which of the following is best treatment for hypogly-
cemia in an unconscious diabetic patient?
A. Intravenous administration of 50% dextrose in water
B. Administration of oral carbohydrates
C. Sublingual injection of 50% dextrose
D. None of the above. No treatment is necessary

30 B 31 D 32 D 33 c 34A
258 MCQs in Oral Surgery

24. A patient with a history of ankel edema, shortness of


breath and orthopnea Is probably suffering from:
A. Asthma
B. Emphysema
C. Rhinophyma
D. Cardiac decompensation
25. Ultrashort-acting barbiturates produce loss of con-
sciousness by depression of the:
A. Thalamus
B. Spinal cord
C. Limbic system
D. Ascending portion of the reticular activating system
26. Use of which of the following antibiotics is most re-
stricted because of its side effects?
A. Penicillin B. Tetracycline
C. Cephalexin D. Chloramphenicol
27. Abnormal coagulation time will occur in a patient with
which of the following conditions?
A. Hemangioma B. Hemophilia
C. Thalassemia D. Pernicious anemia
28. When vasoconstrictors produce constriction of
afterioles, the site of action is at which receptor?
A. Alpha B. Beta
C. Gamma D. Delta
29. External cold application to achieve hemostasis
causes:
A. A transient/ vasoconstriction
B. Increased chemotaxis
C. Active component which assist in hemostasis
D. Band C

24 D 25 D 26 D 27 B 28 A 29 A
Medical Emergency 261

40. AU of the following tissues contain alkaline phos-


phatase except:
A. Bone B. Muscle
C. Prostate D. Kidney
41. Which of the following indicates a localised clot for-
mation intravenously at the site of Injury?
A. Thrombus
B. Platelet agglutination
C. Embolus
D. Organised hematoma
42. In artificial ventilation it is commonly recommended
that the rescuer deliver a resting tidal volume that is:
A. Four times the normal
B. Three times the normal
C. Twice the normal
D. Normal
43. In an elective tracheostomy the entry should be made:
A. Above the cricoid
B. Below the cricoid
C. Through the cricothyroid membrane
D. Laterally below the thyroid cartilage
44. Patients who are suffering with uncontrolled.juvenile
onset diabetes mellitus are usually poor candidates
for surgical procedure because of:
A. Their inability to follow postoperative instructions
B. The high incidence of wound infection and poor
healing
C. Their tendency to bleed profusely
D. Their oral hygiene

40C 41 A 42 c 43 8 448
260 MCQs in Oral Surgery

35. Which of the following may occur due to hyperventi-


lation in an anxious patient?
A. Acidosis
B. Elevated pC02 > M
C. Carpopedal spasm
D. Cyanosis
36. Of the following which is treatment of sustained con-
vulsive reaction to a local anaesthesia?
A. Oxygen and IV fluids
B. Diazepam and oxygen
C. Phenytoin
D. Slow acting barbiturates and oxygen
37. Of the following which nerve is affected in Saturday
night palsy?
A. Ulnar
B. Lingual
C. Radial
D. Median cephalic
38. A patient came to the emergency ward who needs
immediate transfusion of blood. There is no time for
blood grouping and cross matching which of the fol-
lowing group of blood can be transfused?
A. Rh positive
B. ABRh positive
C. Rh negative
D. ABRh negative
39. Risus sardonlcus Is a sign of:
A. Rabies
B. Tetanus
C. Poliomyelitis
D. Diphtheria

35 c 36 B 37 c 38 c 39 B
Medical Emergency 263

51. All of the following conditions characterized by neu-


trophilic leukocytosis (Increased neutrophlls) except:
A. Neonatal period
B. Corticosteroid therapy
C. Typhoid
D. Periapical abscess
52. Of the following which condition Is characterised by
decreased eoslnophils?
A. Aplastic anemia
B. Periarteritis nodosa
C. Amoebiasis
D. Psoriasis
53. Which Is factor IV?
A. Thromboplastin
B. Calcium
C. Christmas factor
D. Fibrinogin
54. Of the following which is present in both intrinsic and
extrinsic pathway?
A. Factor VIII
B. Factor XII
C .. Factor V
D. None of the above
55. Of the following which is screanlng test for scurvy?
A. Tourniquet test
B. Immunoassay for total body Vitamin C
C. Melkerson-Rosenthal test
D. None of the above

51 c 52 A 53 B 54C 55 A
262 MCQs in Oral Surgery

45. All of the following play a role in hemostasis except:


A. Vessel wall B. Blood platelets
C. Albumin D. Prothrombin
46. Universal distress signal, characterizing the ob-
structed airway in a conscious adult is:
A. Rapid heavy breathing
B. Victims's hand at his throat
C. Violent choking
D. Violent thrashing of the victim's arm
47. Hemophilia-A is:
A. X-Linked recessive
B. X-Linked dominant
C. Autosomal recessive
D. Autosomal dominant
48. Normal factor VIII levels in the blood is:
A. 5.0-6.5l.u/ rnl B. 3.0-4.51.u/ ml
C. 1.5-2.5l.u/ ml D. 0.5-l.Sl.u / ml
49. Epsilon amino caproic acid (EACA) is one ofthe agent
useful in the treatment of hemophilia. A mechanism
of action of EACA is:
A. It is rich in factor VIII, which is deficient in hemophilia
B. Antifibrinolytic activity
C. It accelerates the coagulation process which is usually
delayed in hemophilia
D. By unknown mechanism
50. Because of many side effects of (EACA) it Is largely
replaced by:
A. Desmopressin
B. AC A (Amino caproic acid)
C. Tranexamic acid
D. Cryoprecipitate

45 c 46 B 47 A 480 49 8 50 c
Medical Emergency 265

61. Of the following which indicates early oxygen want?


A. Cyanosis
B. Increased pulse rate
C. Bradycardia
D. A and B
62 . Among the following where is tetanus likely to occur?
A. Avulsive wounds
B. Perforating wounds
C. Deep puncture wounds
D. Contaminated laceration
63. Cold packs commonly used:
A. To produce local vasodilation
B. Immediately after surgery
C. To increase permeability
D. To prevent the spread of infection
64. Which of the following is a constant finding in sys-
temic infection:
A. Fever B. Swelling
C. Bacteremia D. Lymphadenopathy
65. Immediate post-surgical swelling can be best reduced
by:
A. Gentle manipulation of soft and hard tissue and early
application of cold packs
B. Judicious administration of antihistamines
C. Jud icio us administration of n o nsteroidal anti-
inflammatories
D. None of the above

61 D 62 c 63 B 64A 65 A
264 MCQs in Oral Surgery

56. Prolonged prothrombin time is seen in all of the fol-


lowing except:
A. Factor VII deficiency
B. In patient with coumarin therapy
C. Factor V deficiency
D. Factor XI deficiency
57. Serum Alkaline phosphatase increase in all of the fol-
lowing conditions except:
A. Scurvy
B. Hyperparathyroidism
C. Paget's disease
D. None of the above
58. Of the following what would be the WBC range in mild
infections:
A. 4000- 8000 cells/ mm3
B. 24,000-30,000 cells/ mm3
C. 15,000-20,000 cells / mm3
D. None of the above
59. Of the following which is earliest sign of hemorrhagic
shock?
A. Vasoconstriction
B. Hypotension
C. Tachycardia
D. Dyspnoea
60. Near accurate body temperature can be obtained if
one measures:
A. Orally
B. Axillary
C. Rectally
D. Temperature over forehead

56 D 57 A 58 c 59 c 60 c
Medical Emergency 267

71. On examination it is noted that a patient requires


5 to 6 seconds to rapidly and completely exhale after
a deep Inspiration. He may be suffering with:
A. Upper air way disease
B. Advanced pulmonary disease
C. Severe cardiovascular disease
D. Normal exhalation time
72. Which of the following may result In Interruptions In
cardiac compressions?
A. Little changes in blood flow and blood pressure·
B. A reduction of the blood flow and blood pressure to
zero
C. Carbon dioxide buildup in the lungs
D. None of the above
73. Which of the following is true of cardiopulmonary re-
suscitation?
A. Compression to ventilation ratio in two person CPR
is 5:1
B. Compression to ventilation ratio in single person CPR
is 15:2
C. Compression should be 60-80/minute in adults
D. All of the above
74. In a patient presenting for dental surgery with the his-
tory of chest pain upon exertion, which Is relieved by
rest and nitroglycerin, one would suspect:
A. Pneumothorax
B. Fractured rib
C. Myocardial infarction
D. Angina pectoris

71 B 72 B 73 D 74 D
266 MCQs in Oral Surgery

66. Surgical risk for a cardiac patient depends upon his:


A. Blood pressure
B. Respiration
C. Pulse rate
D. Cardiac reserve
67. Hemorrhagic shock is characterized by:
A. Hypotension
B. Low blood volume
C. Increased pulse rate
D. All of the above
68. Of the following which is most useful in evaluating a
patient for an abnormal bleeding tendency?
A. History
B. Bleeding time
C. Prothrombin time
D. Rumple-Leede test
69. Treatment: Syrup Brufen 2 teaspoon TID for 2 days.
What is the volume of teaspoon and what is the mean-
ing of TID?
A. 5 ml and twice in a day
B. 5 ml and thrice in a day
C. 15 ml and thrice in a day
D. 15 ml and twice in a day
70. Which of the following is first manifestation of com-
plete respiratory obstruction?
A. Pronounced retraction of in tercosta l and
supraclavicular spaces
B. Prolonged expiration
C. Cyanosis
D. None of the above. No changes in the patient

66 0 67 0 68 A 69 B 70 A
Medical Emergency 269

79. Before any oral surgical procedure Is carried out,


prophylactic antibiotic cover Is Indicated for patients
with the following conditions, except:
A. Rheumatic heart disease
B. Chronic renal failure
C. Prosthetic heart valve
D. Coronary artery bypass
80. A patient requiring tooth extraction Is taking anti-co-
agulant drugs for the past six months. His prothrom-
bin time is 21 seconds and the control is 15 seconds.
The treatment under these circumstances should In-
clude:
A. Preoperative administration of vitamin K
B. Postoperative administration of vitamin K
C. Discontinuation of the anticoagulant drugs for one
week prior to the extraction
D. Extraction of the tooth with proper local care to control
bleeding
81. When two rescuers are performing cardiopulmonary
resuscitation on an adult, how many compressions
are to be given per minute?
A. 20 B. 40
C. 60 D. 80
82. Inhaled aromatic spirits of ammonia act against syn-
cope by:
A. Direct stimulation of the respiratory center
B. Direct stimulation of the vasomotor center
C. Irritating the sensory endings of the olfactory nerves
D. All of the above
83. The minimal acceptable value for a haematocrit for
elective oral surgery Is:
A. 20 B. 30
C. 40 D. SO

79 D 80 D 81 c 82 c 83 B
268 MCQs in Oral Surgery

75. A patient on dicumerol therapy should have which of


the following prior to dental surgery?
A. Vitamin~
B. A sedative
C. Prothrombin time test
D. An antibiotic
76. In Pierre-Robin syndrome, a complete upper airway
obstruction may occur when the infant is in a supine
position or during feeding. Due to the nature of this
obstruction, it may be handlel by putting the patient
into a prone position. If this fails, the next procedure
should be:
A. Intubation
B. Nasal airway
C. Tracheostomy
D. Displacing mandible forward
77. A patient in shock should:
A. Be placed in Trendelenburg's position
B. Be placed in reverse Trendelenburg's position
C. Have his legs elevated 20 degrees with trunk and head
remaining level
D. Have his arms elevated 20 degrees with trunk and
head remaining level
78. In Pierre-Robin syndrome, a complete upper airway
obstruction may also occur when the patient is in a
prone position. This may be handled by:
A. Intubating the patient
B. Performing a tracheotomy
C. · Displacing the mandible forward
D. Inserting a nasal catheter with oxygen under pressure

75 c 76 D 77 c 78 c
Medical Emergency 271

88. The importance of chronic alcoholism prior to oral


surgery is:
A. Predispoistion to syncope
B. Relative contraindication to local anaesthesia
C. Prolonged bleeding secondary to liver dysfunction
D. Contraindication for use of NSAIDs

89. An odour of acetone in one's breath should direct


suspicion to:
A. Heart disease
B. Liver damage
C. Diabetes mellitus
D. Kidney disease
90. Extraction of four teeth is required in a 65-year-old
diabetic female who has had her morning insulin, pre-
operative instruction should include:
A. Do not eat or take any medication by mouth prior to
surgery
B. Increase sugar intake prior to surgery
C. Maintain normal diet
D. Increased premedication
91. A patient on recent prolonged steroid therapy may
have:
A. Relative adrenal insufficiency
B. Intolerance to sedatives
C. Prolonged post extraction haemorrhage
D. Relative analgesic potential
92. Normal bleeding time by Duke's method:
A. 1 toS min
B. 3 toS min
C. 3 to 8 min
D. 10 to 30 sec

88C 89 0 90 c 91 A 92 A
270 MCQs in Oral Surgery

84. For the ligation and exposure of the external carotid


artery in the carotid triangle, the incision is made
from:
A. The angle of the mandible to the cricoid cartilage
B. The tip of the mastoid process to one inch below the
mandibular angle
C. The angle of the mandible to the hyoid bone
D. The level of the hyoid bone to the cricothyroid
cartilage
85. Intraoperative venous haemorrhage can be controlled
by the following methods except:
A. Electrocoagulation
B. Ligation with haemostats
C. Bone wax temponade
D. Pressure packing with gauze
86. In order to carry out cardiac .compression effectively,
the sternum should be depressed:
A. 1 inch every second
B. 1.5 to 2 inches per second
C. 2 inches per five seconds
D. 1 inch every ten seconds
87. A pregnant woman in her third trimester loses
consciousness on the dental chair, she should
immediately be:
A. Placed in the Trendelenburg's position
B. Lowered in the horizontal position and turned toward
her right side
C. Placed in the supine position with the feet elevated
slightly
D. Lowered in the supine position and turned toward
her left side

84A 85 c 86 B 87 D
Medical Emergency 273

99. Which of the following are symptoms of digitalis tox-


icity?
A. Resting pulse less than 60 beats/mm
B. Arrhythmias and mental disturbances
C. Nauseas
D. All of the above
100. Which drug is contraindicated in patients taking so-
dium warfarin daily?
A. Aspirin
B. Morphine
C. Benadryl
D. Pentobarbital
101. But mainly in the liver, procaine reduces the
effectivenss of the following drug group:
A. Sulfonamides
B. Pencilline
C. Aminoglycosides
D. Cephalosporins
102. One of your patient stated that he is narcotic addict.
After the extraction which of the following analgesics
are contraindicated?
A. Aspirin
B. Pentazocin
C. Acetaminophen
D. None of the above
103. Prophylaxis for tetanus in a previously immunized
person involves the use oftetanus toxoid as a booster
in which manner?
A. 5 ml subcutaneously
B. 5 ml orally
C. 1 ml intramuscularly
D. 3 ml intravenously

99 0 100 A 101 A 102 B 103 c


272 MCQs in Oral Surgery

93. Treatment required for post operative cutaneous ec-


chymosis is:
A. Aspiration of blood
B. Use of topical steroids
C. Application of cold
D. None
94. The rate of infusion of Intravenous diazepam is:
A. 3 ml / min B. 1 mg/ min
C. 1 ml/ min D. 3 mg/min
95. The following elements interfere with the activity of
tetracycline:
A. Ca and Bi B. Na and K
C. Mgand Na D. Mgand Ca
96. Class of drugs are currently, the best oral sedative
drugs for dentistry:
A. Narcotics
B. Barbiturates
C. Benzodiazepenes
D. Ethanols
97. Among the following which is contraindicated in pa-
tients taking sodium warfarin therapy?
A. Acetaminophen B. Diphenhydramine
C. Ibuprofen D. Codeine
98. Intravenous diazepam administration some times
causes phlebitis. This usually attributed to the pres-
ence of:
A. Ethyl alcohol
B. Benzoic acid
C. Propyleneglycol
D. Methyl paraben

93 c 94 c 95 D 96 c 97 c 98 c
Medical Emergency 275

109. Mechanism of action of penicillin Is due to:


A. Damage to the cytoplasmic membrane
B. Interference with cell wall synthesis
C. Antimetabolite action
D. None of the above
110. Which of the following antibiotic mainly effective
against gram-negative bacteria:
A. Lincomycin
B. Vancomycin
C. Kanamycin
D. Clindamycin
111. Among the following which drug Is commonly
administered to a patient with an allergic reaction who
is demonstrating urticaria and wheezing:
A. Steroid
B. Atropine
C. Dexamethasone sodi urn
D. Adrenalin
112. The following drugs can be used for conscious seda-
tion:
A. Diazepam
B. Phenobarbital
C. Chloral hydrate
D. All of the above
113. Among the following which drug Is least rapidly act-
ing in treatment of anaphylaxis:
A. Adrenaline
B. Hydrocortisone
C. Diphenhydramine
D. Aminophyline

109 B 110 c 111 D 112 D 113 B


274 MCQs in Oral Surgery

104. A RHO patient requires preoperative prophylaxis with


penicillin. An appropriate regimen Is:
A. 2 grn penicillin V 1 hour before surgery
B. 6,00,000 ·units procaine penicillin IM 12 hourly day
before surgery
C. 250 rng p henoxyrnethyl penicillin 6 hrly day before
surgery
D . All of the·above are correct
105. Glucocorticoids :
A. Decreases inflammation
B. Decreases pain threshold
C. Increases the rate of wound healing
D. Enhances the repair
106. Among the following which analgesic is contraindi-
cated in patients with G6PD deficiency:
A. Aspirin
B. Paraceternol
C. Opioid analgesic
D. None of the above
107. How many postoperative days one should continue
antibiotic coverage for subacute bacterial endocardi-
tis?
A. One B. Two
C. Three D. Four
108. Among the following which is " Broad spectrum" an-
tipiotlc:
A. Sulphanarnides
B. Penicillin
C. Tetracycline
D . None of the above

104 A 105 A 106 A 107 B 108 c


Medical Emergency 277

119. All of the following are important in the management


of a patient depressed from known over dose of mor-
phine except:
A. Assistance of ventilation
B. Provision of a patent airway
C. Narcotic antagonist
D. Tranquilizer
120. Which of the following is the drug of choice for treat-
ment of an Infection which Is caused by a gram- posi-
tive, non penicillinase staphylococci organism?
A. Clindamycin B. Erythromycin
C. Penicillin G D. Cephalosporin
121 . Usual adult dosage of oral codeine is:
A. 0.5 - 1.0 grains B. 0.5 - 1.0 grains
C. 1.0-2.0 grains D. 2.0 grains
122. Intra articular injections of steroids into painful joints
such as arthritic TMJ, have palliative effects from:
A. Providing an analgesic effect
B. Providing better lubricating effect
C. Providing an anti inflammatory action
D. Increasing the blood supply ·
123. Penicillin is not effective against certain organisms
because the causative organism:
A. Produces an enzyme inhibiting the activity of the drug
B. Produces catalase
C. Is not a gram-negative organism
D. None of the above are correct. Penicillin works on
almost all the organisms

119 D 120 c 121 A 122 c 123 A


276 MCQs in Oral Surgery

114. Among the following which is correct regarding oral


route of drug administration ?
A. Its effect is most predictable
B. Quickest way to administer drug
C. It is usually a pleasant route
D. AandC
115. Of the followi ng which group of drugs commonly
cause respiratory depression:
A. Antihistamines
B. Tranquilizers
C. Synthetic narcotics
D. Non barbiturate sedative hypnotics
116. In which of the following conditions corticosteroid
administration is contraindicated?
A. Peptic ulcer
B. Psychosis
C. Tuberculosis
D. All of the above conditions
117. Administration of short acting barbiturates for pre-
operative sedation is valuable because it:
A. Is easily administered and has no side effect
B. Produces drowsiness and amnesia
C. Prevents respiratory depression
D. Is compatible with narcotics and has no hazards
118. Of the following which is effective agent for the treat-
ment of respiratory depression which is the result of
overdosage of barbiturates?
A. Caffeine
B. Oxygen
C. Metronidazole
D. None of the above

114 c 115 c 116 D 117 A 118 8


Medical Emergency 279

129. Clinical hemophilia is characterised by (BT - bleed-


ing time, CT- clotting time, PT- Prothrombin time, PTT
- Partial thromboplastin time:
A. Prolonged PT and normal BT
B. Prolonged BT, PT and PTT
C. Normal BT and PT and prolonged PTT
D. Prolonged BT and normal CT; PT and PTT
130. Severe liver disease may cause the following:
A. Decreased quality of circulating platelets
B. Decreased prothrombin production
C. Increased drug detoxification and decreased drug
effects
D. None of the above
131. Characteristic featured of hypovolaemlc shock In-
clude:
A. Hypotention
B. Low pulse pressure
C. Tachycardia
D. All of the above
132. "Vital signs" include which of the following?
A. TPR (temperature, pulse, respiration)
B. BP (blood pressure)
C. All of the above
D. None of the above
133. Among the following in which condition one might
see Hutchinson's incisors?
A. Acquired syphilis
B. Congenital syphilis
C. Secondary syphilis
D. Tertiary syphilis

129 c 130 B 131 c 132 c 133 B


278 MCQs in Oral Surgery

124. Which of the following drug administration produces


the most rapid relief of the symptoms of angina pec-
toris?
A. Pentobarbital
B. Amyl nitrate
C. Oxygen
D. Phenobarbital
125. History of bruising easily, nocturia, excessive thirst
and low resistance to infections indicate the patient
is most likely suffering from:
A. Diabetes mellitus
B. Glomerulonephritis
C. Lupus erythematous
D. Thrombocytopenic purpura
126. Spider telangiectatic spots on the skin of a patient
are noted. He should have an examination to deter-
mine the condition of his:
A. Kidneys B. Liver
C. Lungs D. Gallbladder
127. A 13-year-old girl has a history of severe sore throat,
migratory type of joint pains and swollen joints of the
extremities. This history is suggestive of:
A. Gout B. Serum sickness
C. Rheumatic fever D. Osteoarthritis
128. Laboratory data of patient indicates white blood cell
count more than 1,00,000/ec most likely the patient is
suffering from:
A. Leukopenia
B. Polycythemia
C. Leukemia
D. Anemia

124 B 125 A 126 B 127 c 128 c


Medical Emergency 281

138. Which of the following preoperative hematological


analysis may reflect von Wlllebrand disease?
A. Bleeding time
B. Factor X level assay
C. Platelet count
D. Factor IX level assay
139. Among the following which is absolute contraindica-
tion to the teeth extraction?
A. Heart murmur
B. Patient on anticoagulant therapy
C. Patient taking steroids
D. None of the above
140. One patient states he has recently become excessively
thirsty (polydipsia], hungry (polyphagia] and arises
at night several times to urinate [polyuria). The most
probable diagnosis Is:
A. Acute dehydration
B. Congective heart failure
C. Diabetes mellitus
D. Renal failure
141. Among the following in which condition eosinophilia
is not seen?
A. Parasitic infection B. Allergy
C. Hodgkin's disease D. Cushing's syndrome
142. Among the following which physical sign would de-
ter you from proceeding with multiple extraction and
alveoloplasties without further evaluation?
A. Hepatomegaly
B. Seleral icterus
C. Ascites
D. All of the above

138 A 139 D 140 c 141 D 142 B


280 MCQs in Oral Surgery

134. Among the following which is absent in an asthmatic


patient?
A. Hyperactive bronchi
B. Inspiratory wheezes
C. Bronchi that rapidly and spontaneously change caliber
in response to various stimuli
D. History suggestive of allergy
135. Oral examination reveals unilateral lesions along the
distribution of nerve, localised hyperemic and vesicle
formation vesicles filled with yellow fluid and when it
ruptures, it leaves an eroded area surrounded by a
red areola. The lesion most likely is:
A. Lichen planus B. Herpes zoster
C. Pemphigus D. Fordyce diseases
136. Acetone odour in one's breath usually indicate the
person is suffering with:
A. Heart disease B. Diabetes mellitus
C. Kidney disease D. Liver damage
137. A patient came to oral surgery department who has
spontaneous bleeding from the gums, acute gingival
hyper trophy, along with complaining of weakness and
anorexia. His blood picture showing the:
HB% 14gm
RBC count: 4.5 million/ mml
WBC 1,10,000
DC - Neutrophils- 88%
Lymphocyte- 10%
Monocytes - 0%
Eosinophils- 2%
Likely diagnosis of the above findings
A. Infectious mononucleosis
B. Thrombocytopenic purpura
C. Myelogenous leukemia
D. Marked gingivitis due to local cause

134 B 135 B 136 B 137 c


Medical Emergency 283

147. Of the following which represent complete respira-


tory obstruction?
A. Prolonged expiration
B. No movement by the patient
C. Pronounced retraction of the intercostal and
supraclavicular spaces
D. Cyanosis
148. Of the following which is a feature of acromegaly?
A. Micrognathia
B. Hypoglycemia
C. Crowded teeth
D. Large tongue
149. In which of the following conditions one can see neck
· swelling?
A. Hodgkin's disease
B. Irlfectious mononucleosis
C. Tuberculosis
D. All of the above
150. Among the following which bone lesion is most fa-
tal?
A. Paget's disease
B. Osteochondroma
C. Multiple myeloma
D. Odontogenic myxoma
151. Increased Bence-Jones proteinuria and multiple ra-
diolucent areas in the skull indicate which of the fol-
lowing conditions?
A. Hodgkin's lymphoma
B. Burkitt's lymphoma
C. Multiple myeloma
D. Adeno carcinoma

147 c 148 D 149 0 150 c 151 c


282 MCQs in Oral Surgery

143. If one consider BT (bleeding time), PT (prothrombin


time) PTT (partial thromboplastin time) which of the
following represent vascular hemophilia?
A. Normal BT P1T and prolonged BT
B. Prolonged BT, PT and P1T
C. Prolonged PT, moderately prolonged P1T and normal
BT
D. Prolonged BT, moderately prolonged P1T and normal
PT
144. Macroglossia seen in all of the following conditions
except:
A. Cretinism
'B.Von Reck Linghausen's disease
C. Scurvy
D. None of the above
145. Patient history reveals dyspnea, orthopnoea, edema
of the ankle and palpitation most likely diagnosis is:
A. Respiratory problem
B. Hepatic failure
C. Uremia
D. Congestive heart failure
146. An anxious, nervous patient states that he has had
recent weight loss and is easily fatigued. Tremors,
tachycardia and tremors, sweaty palms are noted in
physical examination most likely diagnosis is:
A. Renal disease
B. Hyperthyroidism
C. Diabetes
D. Cushing's disease

143 0 144 0 145 0 146 B


Medical Emergency 285

157. Which Is the minimal acceptable value for a hemat-


ocrit for elective surgery?
A. 30 B. 25
C. 15 D. 5
158. If one uses normal cuff of Sphygmomanometer on an
adult obese patient it will yield diastolic pressure read-
ings that are:
A. Falsely high
B. Falsely low
C. Accurate but systolic pressure readings are falsely
high
D. Accurate but systolic pressure readings are false
159. Among the following which is true regarding phlebi-
tis:
A. High doses of antibiotics are necessary to accelerate
resolution
B. Application of ice packs to the area during the first 2
days is indicated
C. Local application of heat and immobilisation for
several days is indicated
D. Abscess formation is quite common
160. The primary airway hazard for an unconscious patient
In a supine position is:
A. Tongue obstruction B. Bronchospasm
C. Laryngospasm D. Aspiration
161. The best blood product administered preoperatively
to patients with hemophilia A Is:
A. Fresh frozen plasma
B. Factor IX concentrate
C. Whole blood
D. Factor VIII concentrate

157 A 158 A 159 c 160 A 161 D


284 MCQs in Oral Surgery

152. All of the following are signs of thyrotoxicosis except:


A. Slow pulse rate
B. Exophthalmos
C. Tremor
D. Temperature elevation
153. Cyanosis of the lip may suggest:
A. Carbon monoxide poisoning
B. Anemia
C. Congenital heart disease
D. All of the above
154. The asthmatic patient is characterised by:
A. Recurrent pulmonary infections
B. Multiple pulmonary emboli which may be life
threatening
C. Hyperactive bronchi, that rapidly and spontaneously
change calibre in response to various stimuli
D. None of the above
155. An obese patient falls while jogging and is unconcious
which of the following should be considered to dif-
ferentiate between cardiac arrest and other causes:
A. Pulse (carotid (or) femoral)
B. Pupil constriction/ dilatation
C. Respirations present (or) absent
D. All of the above
156. Of the following which is possible sign of subacute
bacterial endocarditis?
A. Enlarged tongue
B. Discharge from the ear
C. Splinter hemorrhage
D. Intestinal obstruction

152 A 153 c 154 c 155 0 156 c


Medical Emergency 287

167. All of the following are various methods to control


intraoperative hemorrhage except:
A. Gauze sponge pressure
B. Artery application to the open vessels
C. Infiltration with lidocaine
D. Bone compression for surface bleeders
168. Pin point haemorrhages on skin are called as:
A. Purpura B. Hematoma
C. Petechiae D. Ecchymosis
\
169. When one suspects tetanus organisms In a wounded
patient. Antitetanus prophylaxis given in the form of:
A. Tetanus antitoxin (if the patient is previously not
immunized)
B. Penicillin (if there is no allergy)
C. Tetanus toxoid
D. All of the above
170. All patients admitted to the hospital for surgery must
A. Give verbal permission
B. Sign an operative permit after receiving
premedication
C. Sign an operative permit before the discharge
D. Sign an operative permit at the time or admission to
the hospital
171. One suspected rheumatoid arthritis of the TMJ help-
ful positive laboratory study would be:
A. Hematocrit elevation
B. GTT
C. Increased erythrocyte sedimentation rate
D. Tourniquet test (or) hess test

167 c 168 c 169 D 170 D 171 c


286 MCQs in Oral Surgery

162. Cardinal symptom of dehydration due to disturbance


of fluid and electrolyte balance is:
A. Hypothermia
B. Nausea
C. Polydipsia
D. None of the above
163. Emotional stress affect the rate of absorption of oral
medication. Which of the following is correct?
A. It decreases the rate of absorption
B. It increases the rate of absorption
C. The above statement is wrong there is no effect on
the rate of absorption
D. It increases the rate of absorption of few drugs and
decreases the rate of absorption of few drugs
164. If efforts in c;;~rdiopulmonary resuscitation are
effective there will be
A. Constriction of pupils
B. Dilatation of pupils
C. Hypertension immediate
D. None of the above
165. Patient with idiopathic thrombocytopenic purpura
(ITP) is most likely to have which of the following post-
operative complication?
A. Infection
B. Hemorrhage
C. Edema
D. Localised alveolar osteitis (dry socket)
166. Complete blood count does not include:
A. Haemoglobin B. Haematocrit
C. Total WBC D. ESR

162 c 163 A 164 A 165 B 166 D


Medical Emergency 291

187. Frequent site of occurrence for ameloblastomas:


A. Mandibular premolar area
B. Maxillary molar area
C. Antrum and floor of the nose
D. Molar and ramus area of the mandible
188. Among the following which is short acting barbitu-
rate?
A. Pentobarbital B. Phenobarbital
C. Valium D. Codeine
189. Site of action of valium (Diazepam) is:
A. Cerebell urn
B. Limbic systems
C. Cerebral cortex
D. Pyramidal system
190. Among the following which drug is contraindicated
in hyperthyroid patients, because the subjects are
extra-ordinarily sensitive to the drug:
A. Salicylates B. Barbiturates
C. Adrenaline D. Digitalis
191. Among the following which frequently causes infec-
tive endocarditis?
A. Staphylococcus aureus
B. Streptococcus virid ans
C. Staphylococcus pyogens
D. None of the above
192. The emergency, most frequently encountered during
outpatient general anesthesia is:
A. Anaphylaxis
B. Bradycardia
C. Respiratory obstruction
D. Hypotension

187 0 188 A 189 B 190 c 191 B 192 C


288 MCQs in Oral Surgery

172. Which of the following is early sign of want of oxy-


gen?
A. Tachycardia
B. Cyanosis
C. Sternal relaxation
D. Constricted pupils
173. Treatment of syncope is most effectively accom-
plished by which of the following?
A. Ad ministering 100% oxygen for five minutes
B. Lowering the chair to place the patient's head lower
than his feet
C. Bending the patients head forward between his knees
and asking the patient to raise his head against hand
p ressure
D . It req uires treatment by a p hysician

174. Which of the following causes postoperative infec-


tion?
A. Lack of asepsis
B. Failu re to remove pathology
C. Inadequate debridement
D. Any of the above
175. Among the following, which is best hemostatic agent
for control of local hemorrhage?
A. Gel foam B. Surgicel
C. Bone wax D. Vit. K
176. Among the following which causes the practitioner
the greatest concern?
A. 80% of normal PTT
B. 50% of normal PTT
C. 30% of normal PTT
D. 20% of normal PTT

172 A 173 B 174 0 175 B 176 D


Medical Emergency 293

198. The average daily adult fluid Intake should be:


A. Approx. 4000 ml
B. Approx. 1800 ml
C. Approx. 1200 ml
D. Approx. 500 ml
199. The following space infection may cause severe res-
piratory difficulty and tracheostomy may require:
A. Pterygopalatine space
B. Canine space
C. Temporal space
D. Parapharyngeal space
200. Of the following which is most common site of
metastasis from mandible:
A. Heart
B. Liver
C. Lung
D. Pancreas
201. After prolonged steroid therapy patient may have:
A. Prolonged post extraction bleeding
B. Analgesic potentiation
C. Relative adrenaline insufficiency
D. Not tolerable to sedatives
202. Prophylactic antibiotic coverage for a patient with
rheumatic heart disease is required prior to:
A. Scaling and curretage
B. Extraction of maxillary fi rst molar (Single tooth
extraction)
C. Open reduction and internal fixation (ORIF)
D. All of the above required

198 B 199 0 200 c 201 c 202 D


290 MCQs in Oral Surgery

182. In patients on artificial ventilators the cycle of exhale


air ventilation should be repeated every:
A. 20 seconds
B. 10 seconds
C. 5 seconds
D. 1 second
183. The following factors influence the sedation dosage
of intravenous agents:
A. Weight of the patient
B. Age of the patient
C. Type of agent used
D. All of the above
184. Among the following which steps, followed in com-
pleting physical examination?
A. Inspection
B. Palpation and percussion
C. . Auscultation
D. All of the above
185. Which of the following antibiotics is commonly pre-
scribed for penicillin allergic patients?
A. Tetracycline
B. Erythromycin
C. Chloramphenicol
D. Cephalosporin
186. Which cardiac condition require antibiotic prophylaxis
before surgery?
A. Coronary insufficiency
B. Angina pectoris
C. Rheumatic carditis
D. Congestive heart diseases

182 c 183 0 184 D 185 B 186 c


Medical Emergency 295

208. Among the following all are Important measures to


prevent emergencies except:
A. Monitor the patient intraoperatively
B. Assess your patient's medical status preoperatively
C. Weigh your patient
D. Keep emergency kit upto date
209. Treatment of choice for postoperative cutaneous ec-
chymosis is:
A. Aspiration of blood
B. Application of cold
C. Use of topical steroids
D. None
210. After the administration of penicillin patient reports
itching in various areas of the body. One should:
A. Ignore the itching
B. Discontinue all medication
C. Call an ambulance and meet patient in hospital
D. emergency room
E. BandC
211. Allergic reactions may be characterized by:
A. Bronchospasm
B. Cardiovascular collapse
C. Angioneurotic edema
D. All of the above
212. Continuation of antibiotic medication in gram nega-
tive soft tissue infections is required:
A. At least 2 days after all clinical signs of infection
subside
B. Until acute symptoms of infection subside
C. for a minimum of 7-10 days
D. For a period of 21 days

208 c 209 D 210 D 211 D 212 c


292 MCQs in Oral Surgery

193. A patient came to dental clinic who has a pulse rate of


72, a respiratory rate of 15, a BP of 120/80, warm pink
extremities and pupils that constrict during near
accommodation. What is the most likely diagnosis?
A. Normal patient
B. Acute anxiety syndrome
C. Coronary ar tery disease
D. Myopia
194. An inhaled foreign body most probably enters:
A. Left bronchus
B. Righ t bronchus
C. Esophagus
D. N one of the above
195. Among the following which is an advantage of IV
administration of a drug?
A. Minimal skill is necessary
B. Sedative drugs are compatible with IV solution
C. It eliminates side effects
D. It allow s titration of the d r ug

196. Among the following which factor is strongest stimu-


lator to increase the respiration?
A. Decrease in venous oxygen
B. Increase in blood pH
C. Increase in arterial ca rbon dioxide
D. Decrease in arterial oxygen
197. During the treatment of shock jugular venous pres-
sure (JVP) should be maintained in the range of:
A. ·15-20 mm Hg
B. 5-10 mm Hg
C. 10-15 mm Hg
D. 0.2-5 mm Hg

193 A 194 B 195 D 196 c 197 c


IMPLANT

1. Osseointegrated implants have the following features:


A. They form junctional epithelium with the surrounding
tissues
B. They are anchored directly to living bone as
determined by radio graphic and light microscopic
analyses
C. They have direct structural and functional connection
with bone only at the radiographic level of detection
D. They form a pseudo-periodontal ligament
2. Osseointegration implies:
A. The process of inserting implants in bone
B. Integration of the bone and implant with fibrous tissue
between them
C. At least some direct contact of living bone and the
surface of the implant at the light microscopic level of
magnification
D. None of the above
3. An endosteal implant is an implant inserted in:
A. Periosteum
B. Bone
C. Root canal
D. None of the above

1 B 2 c 3 B
294 MCQs in Oral Surgery

203. The significance of chronic alcoholism prior to oral


surgical procedure is:
A. Prolonged haemorrhage secondary to liver disease
B. Intolerence to LA
C. Predisposition to syncope
D. Contraindication for N 20 analgesics
204. Which of the following is significant in a patient with
renal transplant?
A. Hematocrit 37%
B. Cholesterol -200 mg%
C. WBC -2000
D. BUN -21 %
205. At what diastolic pressure do you consider the pa-
tient to have significant hypertension?
A. 110 B. 90
C.- 85 D. 65
206. Most important single aspect of a patient's surgical
experience is:
A. Family history
B. Past medical history
C. Complete blood count
D. Suture technique
207. The hematocrit serves as an index for:
A. Monocyte volume
B. Patient red blood cell volume
C. Patient total blood cell volume
D. Patient white blood cell volume

203 A 204 D 205 A 206 B 207 B


Implant 299

9. While Inserting an implant a gap is observed between


the implant and the prepared site the implant will most
probably:
A. Fail
B. Be a success
C. The gap does not affect the success of the implant
D. None of the above
10. The most common types of implant in use today are:
A. Subperiosteal implant
B. Transosteal implants
C. Endosteal implants
D. All of the above
11. Which of these is an indication for implant placement?
A. Inability to wear a removable or complete denture
B. Unfavorable number and location of natural tooth
abutments
C. Single tooth loss which would necessitate pr~paration
of undamaged teeth for FPD
D. Any of the above
12. Ideal amount of bone under soft tissue is:
A. 8 mm vertical and 4 mm horizontal
B. 10 mm vertical and 6 mm horizontal
C. At least 12 mm vertical and 6 mm horizontal
D. Depends from patient to patient
13. The minimum space between implant should be:
A. 2mm
B. Smm
C. 3mm
D. 4mm

9 A 10 c 11 D 12 8 13 c
296 MGQs in Oral Surgery

213. Which of the following values should be taken if there


is excessive blood loss after a surgical procedure?
A. Hematocrit and platelet coun t
B. WBC and RBC co unt
C. Hemoglobin and Hematocrit
D. Platelet and RBC count

214. Among the following which patient will probably have


bleeding problems postoperatively?
A. Leukaemic pa tient
B. Hypertensive patient
C. Thrombocy to~nic patient
D. All of the above arc correct
215. If a normal patient losses approximately 1000 cc of
blood during surgery, the necessary colloid fluid re-
placement should be:
A. 1000 cc
B. 2000 cc
c. 3000 cc
D . 6000 cc
216. Blood transfusions may cause complications. Which
of the following are possible complications associ-
ated with blood transfusions?
A. Circulatory overload
B. Thrombophlebitis
C. Immediate and delayed hemolytic reactions
D. All of the above

213 c 214 0 215 c 216 D


Implant 301

20. The recommended time interval between surgery and


placing load in the posterior mandible is:
A. 2 months B. 3 months
C. 4 months D. 6 months
21. The recommended time interval between surgery and
placing load in the maxilla is:
A. 2 months B. 3 months
C. 4 months D. 6 months
22. Internal irrigation is used for implant surgery:
A. To clear the operative field
B. To cool the rotating bur so that. the temperature in
the bone does not increase
C. So that bone can absorb the water
D. None of the above
23. Whenever implants are placed in the posterior man-
dible they should:
A. Engage the superior cortical bone and medullary bone
B. Engage the superior cortical, medullary bone and
inferior cortical bone
C. Engage only the superior cortical bone
D. None of the above
24. Implants placed in the posterior maxilla should:
A. Leave 1 mm of bone between the floor of the sinus
and the implant
B. Leave 2 mm of bone
C. Can go into the antrum if required as long as lining is
not punctured
D. None of the above

20 c 21 D 22 B 23 B 24 A
298 MCQs in Oral Surgery

4. Endosteal implant can be:


A. Root form implant only
B. Plate form implant only
C. Can be either root form or plate form
D. Combination of both
5. The transfer coping In an implant is used:
A. To position an analog in the impression
B. To transfer the position of the implant in the mouth
to the cast
C. To gain exact implant positioning
D. All of the above
6. In case of endosseous implant:
A. A seal between implant surface and soft tissue is not
required
B. A permucosal seal of the soft tissue at the implant
surface is very essential
C. Whether the seal is established or not it does not really
make a difference to the success
D. None of the above

7. A permucosal seal in case of a dental implant is:


A. Possible
B. Absolutely impossible as tissue will not stick to the
implant surface
C. Not necessary at all
D. None of the above
8. Lack of osseointegration can be due to:
A. premature loading of the implant system
B. Placing the implant with too much pressure
C. Overheating the bone during preparation
D. All of the above

4 c 5 A 6 B 7 A 8 D
Implant 303

31. Among the following which indicates successful al-


logenic tooth implant?
A. The tooth survives for 12 months
B. The periodontal' ligament is re-established
C. Both the pulp and periodontal ligament survive
D. Ankylosis occur
32. Which of the following is true in regard to reconstruc-
tion of an edentulous mandible with implants?
A. Subperiosteal implant requires only single surgery
B. Staple implant is most useful for the posterior
mandible
C. Both blade and the osteointegrated cylinder are useful
as posterior abutments in patients with high mental
foramen
D. Osteointegrated concept of implant. Stabilisation has
the best documentation of long term success
33. Angle SNA in a cephalogram indicates:
A. Mandible - cranial bone relationship
B. Mandible - maxilla relationship
C. Maxilla- cranial bone relationship
D. Maxillary teeth to cranial bone

31 0 32 0 33 c
300 MCQs in Oral Surgery

14. This distance between the Implant and the superior


aspect of the Inferior alveolar canal should be:
A. lmm B. 3mm
C. 2mm D. 4mm
15. The distance between the implant and the mental fo-
ramen should be:
A. 2mm B. 3mm
C. lmm D. Smm
16. What should the distance between the implant and
the post ligament of the adjacent teeth be?
A. 2mm B. lmm
C. Smm D. 3mm
17. Time taken for Integration of implants in the maxilla
Is:
A. 4months B. 2months
C. 6 months D. 8 months
18. With respect to anatomic limitation the most straight-
forward area for Implant placement is:
A. Anterior mandible
B. Posterior mandible
C. Anterior maxilla
D. Posterior maxilla
19. A surgical guide template is used In Implant dentistry
for:
A. Delineate the embrasure
B. Locate the implant within the restoration contour
C. Align implants along the long axis of the completed
restoration and identify the level of the CEJ or tooth
emergency profile
D. Alloftheabove

14 c 15 B 16 B 17 c 18 A 19 A
Miscellaneous 305

5. Visible bleeding of an artery is best controlled via:


A. Pressure
B. Tea bag plus pressure
C. Tourniquet
D. Clamping and ligation
6. The purpose of giving antibiotics prior to oral sur-
gery in patients with a history of rheumatic heart dis-
ease is to help:
A. Eliminate all bacteria from the blood stream
B. Prevent recurrence of rheumatic fever
C. Prevent rheumatoid arthritis
D. Prevent subacute bacterial endocarditis
7. A graft that has been detrived from another species
of a different genetic disposition is also known as a:
A. Allograft B. Isograft
C. Homograft D. Heterograft
8. Vitamin K is used in the management of post-extrac-
tion bleeding when the cause is determined to be due
to:
A. Lowered platelet count
B. Thrombocytopenic purpura
C. Prothrombin deficiency
D. Factor VIII deficiency
9. Site selection for intravenous fluid therapy in a out
patient should be the:
A. Dorsal vein, at the back of the hand
B. Median cubital vein
C. Femoral vein
D. Cephalic vein

5 0 6 0 7 0 8 c 9 A
302 MCQs in Oral Surgery

25. Probing 1,1nder local anaesthesia Is used:


A. To check if any sharp bony edge is there
B. To check for caries
C. To check the length of the implant
D. To judge soft tissue thickness at the planned implant
site
26. The transfer coping In an Implant Is used:
A. To position an analog in the impression
B. To transfer the position of the implant in the mouth
to the cast
C. To gain exact implant positioning
D. All of the above
27. The minimum safe distance between an endosteal
Implant and any adjacent anatomical structure should
be:
A. 2mm B. 2.5mm
C. 3mm D. l.Smm
28. Advantage of root form implants over plate forms in-
clude:
A. Greater surface area
B. Fewer pontics
C. Greater bone density
D. All of the above
29. The decrease In bone width within the first one to three
years after tooth extraction Is:
A. 25 % B. 30 %
c. 35 % D. 40 %
30. The following division provides bone In all dimensions
for Implant placement:
A. Div. A B. Div. B
C. Div. C D. Div. D

25 D 28 A 27 A 28 D 29 D 30 A
Miscellaneous 307

16. Body fluids can be responsible for transmission of


AIDS except:
A. Blood B. Cervical secretions
C. CSF D. Tears
17. The facial development seen in HIV positive children
is characterized by:
A. Normal facial profile
B. Microcephalic-
C. Ocular hypotelorism
D. Narrow palpebral fissure
18. Detection of HIV antigen is:
A. Positive after 2 to 4 weeks of infection and then
becomes negative till the clinical disease sets in
B. Positive after 4 to 6 weeks till clinical disease sets in
C. Only positive when clinical disease sets in
D. Positive only after 1 week of infection
19. When blood of HIV patients is analyzed it would show:
A. Leukocytosis
B. Decreased ratio of T4/ T8 cells
C. Decreased level of IgG
D. Decreased level of IgA
20. The immunodeficiency characteristic of HIV infection
is due to:
A. Decreased humoral immunity
B. Decreased cellular immunity
C. Both of the above
D. None of the above

16 D 17 B 18 A 19 B 20 B
MISCELLANEOUS

1. Purpose of taping the eyes shut before surgery is to:


A. Prevent lacrimal secretions which may contaminate
the field
B. Prevent corneal abrasion
C. Limit ocular motility
D. None of the above
2. Of the following which is correct ? Thiersch graft is
in:
A. Full thickness graft
B. Pedicle graft
C. Partial thickness graft
D. None of the above
3. Of the following which is the best method of treating
a green stick fracture of the mandible?
A. Allow normal masticatory movements
B. Bringing the teeth into occlusion with interdental
wiring
C. Extraskeletal fixation
D. None of the above
4. Which one of the following disorders responds favor-
ably during using an occlusal separator?
A. Capsular fibrosis
B. Muscle spasm
C. Chronic dislocation
D. Unilateral condylar hyperplasia

1 B 2 c 3 B 4 B
Miscellaneous 309

26. Aspirator used for suction should be disinfected af-


ter using for HIV patient by:
A. Flushing with 2% glutaraldehyde and washing with
fresh water after 10 min
B. Flushing with 2% glutaraldehyde and leaving
overnight
C. Flushing with savlon and washing after 12 hours
D. None of the above
27. Impressions for maxillofacial trauma should be dis-
infected by: ·
A. Thorough rinsing with water
B. Immersing in 2% glutaraldehyde for 3 hours after
rinsing in water
C. Immersing in 2% glutaraldehyde for minimum of 10
mins after rinsing in water
D. Immersing in 2% glutaraldehyde for 12 hours
28. The dental infections commonly seen in AIDS patient:
A. Necrotizing ulcerative gingivitis
B. Rapidly progressive periodontitis
C. Horizontal bone loss
D. All of the above
29. Body fluid in which HIV is highly concentrated :
A. Semen, cervical fluids
B. Blood
C. Saliva
D. Tears
30. The most common pathogen isolated from pulmonary
system of HIV patient is:
A. Mycobacterium tuberculosis
B. Mycobacterium avium intracellulare
C. Pneumocystis carinii
D. None of the above

26 B 27 c 28 D 29 B 30 c
306 MCQs in Oral Surgery

10. 'Eagle's syndrome' is associated with the elongation


of the:
A. Mastoid process
B. Odontoid process
C. Styloid process
D. Palatine process
-
11. Dental laser technology is now available for:
A. Soft tissue surgery
B. Treatment of hard tissues
C. Non-surgical applications
D. All of the above
12. Viral infection which can be seen in oral cavity of pa-
tients with HIV is/are?
A. Hairy leukoplakia
B. Herpetic stomatitis
C. Papilloma warts
D. All of the above
13. The major criteria of WHO guide line for diagnosing
AIDS consists of all except?
A. Weight loss more than 10%
B. Prolonged fever for more than 1 month
C. Chronic diarrhea for more than 1 month
D. Generalized lymphadenopathy
14. The main target cells of HIV are:
A. Plasma cells B. T4 cells
C. T8 cells D. B cells
15. The cells from which HIV can be isolated:
A. Langerhan's cells
B. Dendritic follicles
C. Alveolar macrophages
D. All of the above

10 c 11 D 12 D 13 D .14 B 15 D
Miscellaneous 311

37. Following chemicals are used for inactivating HIV (ex-


cept):
A. 2% glutaraldehyde
B. 50% ethanol
C. 1% sodium hypochlorite
D. 2.5% cetamide
38. World's AIDS day is observed on:
A. 22nd November B. 26th February
C. 1st December D. 29th November
39. The recommended method for sterilizing impressions
for maxillofacial prosthesis is:
A. Impression in silicone and autoclaving
B. Impression in alginate and soaking in 2%
glutaraldehyde for 10 minutes
C. Impression in silicone and soaking in 2%
glutaraldehyde for 20 minutes
D. Impression in silicone and soaking in 2%
glutaraldehyde for three hours
40. The working surface in operation theatre should be
disinfected especially for HIV, by using:
A. Savlon B. Gamma radiation
C. Hypochlorite solution D. UV light
41. Best way to palpate submandibular gland is:
A. Bimanual extraloral palpation with the patient head
tipped forward and towards the same side
B. Monomanual extraoral palpation with patient's head
tipped forward and toward the same side
C. Bimanual, simultaneous intraoral and extraoral
palpation
D. Intraoral palpation with the patient head tipped
forward

37 D 38 c 39 D 40 c 41 c
308 MCQs in Oral Surgery

21. When T cell subset assay is done in HIV positive pa-


tients there Is:
A. Increased ratio ofT helper / T suppressor cells
B. Decreased ratio of T helper / T suppressor cells
C. Increased ratio of T4/T8 cells
D. Decreased production of TS cells
22. Besides autoclavlng, simple method of Inactivating
HIV Is all except:
A. Dry heat at 100 degree C
B. Boiling for 20 min
C. 1% sodium hypochlorite
D. 6% Hydrogen peroxide
23. WHO guide lines for suspected HIV patients suggests
that they should:
A. Be excluded from the main stream
B. Remain integrated within the society
C. Be excluded to isolation
D. Non of the above
24. HIV crosses:
A. Blood-brain barrier
B. Placental and vitreous humour
C. Both of the above
D. None of the above
25. The Impression material recommended for taking
impression for maxillofacial prosthesis in AIDS
patient:
A. Alginate
B.- Silicone
C. ZnOE
D. · Plaster of paris

21 8 22 A 23 8 24 c 25 8
Miscellaneous 313

48. Which of the following is non resorbable suture ma-


terial?
A. Dacron B. Catgut
C. Polyglycolic acid D. Polyglactin
49. Presence of suture usually increase the susceptibil-
ity to infection by a factor of:
A. 10 times B. 100 times
C. 1000 times D. 10,000 times

50. Langer's lines usually run:


A. Parallel with skin creases and parallel to the action of
underlying muscles
B. Perpendicular with skin creases and parallel to the
action of underlying muscles
C. Parallel with skin creases and perpendicular to the
action of underlying muscles
D. Perpendicular with skin creases and perpendicular to
the action of underlying muscles
51. Among the following suture material which one elicit
more tissue reaction:
A. Catgut B. Silk
C. Nylon D. Linen
52. Which of the following is commonly used as preser-
vative for Gut sutures:
A. Ethyl alcohol
B. Isotonic saline
C. Hypertonic saline
D. Isopropyl alcohol

48 A 49 D 50 c 51 A 52 D
310 MCQs in Oral Surgery

31. HIV virus is a:


A. DNA virus
B. Retrovirus
C. DNA, RNA virus
D. None of the above
32. Diagnosis of paediatric AIDS can be done by:
A. ELISA test B. Western blot test
C. Virus culture D. None of the above
33. The first antibody to HIV antigen appears in blood af-
ter:
A. 1-2 weeks of infection
B. 2-4 weeks of infection
C. 4-6 weeks of infection
D. 4-6 months of infection
34. ELISA test demonstrates:
A. HIV antigen
B. HIV antibodies
C. HIV
D. Nm1e of the above
35. The confirmatory test used for HIV infection is:
A. ELISA B. Immunodot test
C. RIPA test D. Western blot test
36. HIV can be inactivated by (except):
A. Autoclaving
B. 2% glutaraldehyde
C. Boiling for 20 minutes
D. Gamma radiation

31 B 32 c 33 c 34 B 35 D 36 D
314 MCQs in Oral Surgery

53. Surgical needle and sutures are usually sterlized in


manufacturing units by:
A. Gamma radiation
B. X-rays
C. Boiling
D. Dry heat sterilization
54. A patient admitted to the hospital for dental treatment
by a dental staff member is the responsibility of the:
A. Family physician
B. Admitting dentist
C. Kesident physician
D. Physician consultants
55. Completed patient hospital records or charts are the
legal property of the:
A. State
8. Patient
C. Hospital
D. Doctor in charge

53 A 54C 55 c
312 MCQs in Oral Surgery

42. In an Irradiated field, the optimal time for a surgical


procedure is:
A. A Immediately after completion of radiotherapy
B. Any time during radio therapy
C. 6 months to 1 year after the completion of
radiotherapy
D. 4-6 weeks after radiotherapy
43. Commonly used suture material for closure of in-
traoral wound is:
A. Nylon B. Black silk
C. Cat gut D. Chromium catgut
44. The following Is one of the advantage of chromic gut
over plain gut suture:
A. Greater ease of use
B. Non absorbability
C. Greater strength
D. Less expensive
45. Vertical mattress sutures are frequently used in clo-
sure:
A. To evert margins
B. In one tissue layer thick flaps
C. Because of the lack of sufficient blood supply
D. None of the above
46. Virus which causes AIDS is a:
A. Retrovirus
B. DNA virus
C. Reovirus
D. None of the above
47. The following suture material has the memory prop-
erty:
A. Silk B. Catgut
C. Nylon D. Linen

42 D 43 B 44C 45 A 46 A 47 c

You might also like